Sie sind auf Seite 1von 168

‫ملزمــــــــة‬

‫‪NURSING‬‬
1. .Diabetes mellitus is a _____ disorder?
A. Bleeding
B. Metabolic
C. Degenerative
D. Traumatic

2. The expected date of delivery is calculated from date of _____?


A. First love
B. Date of marriage
C. Date of honeymoon
D. Last menstruation

3. Which of the following blood group is universal recipient?


1. A
2. B
3. AB
4. O

4. The local anesthesia is given by:


a. I.V injection
b. I.M injection
c. I.D injection
d. S/C injection

5. A child, who is bleeding heavily, is in hypovolemic shock. The nurse determines that
the child is currently compensating for the loss of blood when the nurse notes which of
the following?
1. Tachycardia
2. Hypotension
3. Bradypnea
4. Cyanosis

6. How many times you should take vital signs in the recovery room at the first hour:
a. 4
b. 10
c. 12
d. 14

7. Streptomycin toxicity can cause ________ loss


a)hair
b)visual
c)hearing
d)a,b,c

8. The best cannuls used when you want to give blood IV to an adult Pt is :
A. Yellow cannula B. Blue cannula C. Green cannula
.

1
9. The nurse that is responsible for setting sterile tables and equipment and assist
surgeon is:
a. scrub nurse
b. circulating nurse
c. recovery nurse
d. none of the above

10. The first nursing intervention should be done in the recovery room is:
a. give the patient analgesics
b. put the patient in recovery position
c. take vital signs and record
d. do suction to the airway

11. what is atelectasis?


A. accumulation of fluid in lungs
B. collapse of lungs
C. inflamation of lungs
D. none of the above

12. The normal range of blood sugar in the newborn is…


a. 50 – 100 mg/100cm
b. 40 – 80 mg/100cm
c. 70 – 110 mg/100cm
d. 40 – 100 mg/100cm

13. The term placenta praevia is____


A) Early development of placenta
B) Early placental expulsion
C) Implantation of placenta in lower uterine segment
D) None

14. Spinal anesthesia is injected into the subarachnoid space at the lumbar level to
block nerves and suspend sensation to the:
a. lower extremities
b. perineum
c. lower abdomen
d. all of the above

15. Streptokinase injection is given to patient with M.I, this drug is


a. Analgesic
b. Narcotic
c. Thrombolytics
d. Anticoagulant

16. Inability of the heart to pump an adequate amount of blood to the systemic
circulation to meet the demands of the body is called...
a. aortic stenosis b. ASD c. CHF d. VSD

2
17. When there is an extra chromosome belonging to the pair 21, the following
symptoms will appear on the baby except one…
a. small head
b. long hands
c. mental retardation
d. large tongue protrudes from the mouth

18. How many fontanels in the newborn baby…


a. one fontanel
b. two fontanels
c. three fontanels
d. four fontanels

19. Cushings disease is due to _____ ?


Q. Pitutary hypersecretion.
A. Adrenal adenoma.
Z. Adrenal hyperplasia.
I. All of above.

20. You should not give digoxin to the patient if his pulse is less than:
a. 60 B/m
b. 70 B/m
c. 80 B/m
d. 90 B/m

21. Before administering digoxin to infant, nurse should take apical pulse for _____
minute(s)
a)30
b)5
c)3
d)1

22. When administering an antibiotic, the nurse must be alert for the possibility of:
a. overdoses and CNS depression
b. hypersensitivity and possible anaphylaxis
c. sings of increasing infection
d. orthostatic hypotension

23. After coming head of breech will have difficulty in delivery in all of the following
condition expect??
A Hydrocephalus
B placenta previa
C Incomplete dilation of cervix
D Extension of head

24. Dr. order:" give ranitidine 40 mg P.O ABT", and the available is 20 mg tablets, how
many tablet you should give to the patient in one day:

3
a. 1
b. 2
c. 3
d. 4

25. DTP is a vaccine that covers all of the followings diseases except:
a. Diphtheria
b. Tetanus
c. Pertussis
d. Typhoid

26. Dehydration in infants may result from:


a. Vomiting
b. Diarrhea
c. Bleeding
d. All of the above

27. Which of the following areas should the nurs try first to insert IV line ?
1_ back of the hand
2_ inner aspect of the forearm
3_ inner aspect of the elbow

28. When administering heparin , the substance the nurse would keep available as the
antidote is:
a. magnesium sulphate
b. protamine sulphate
c. calcium gloconate
d. vitamin k

29. Hemophilia B is the deficiency of clotting factor number:


a. VI
b. VII
c. VIII 8
d. IX

30. When assessing the unconscious adult victim for pulselessness, which of the
following is the best artery to check:
a. Radial
b. Femoral
c. Brachial
d. Carotid

31. In treating puncture wound, the first priority is to:


a. stop the bleeding
b. cleans the wound
c. give prophylactic treatment against tetanus

4
32. The physician has ordered nitroglycerin buccal tablets for a client with stable
angina. The nurse knows that nitroglyerin:
❍ A. Slows contractions of the heart
❍ B. Dilates coronary blood vessels
❍ C. Increases the ventricular fill time
❍ D. Strengthens contractions of the heart

33. Placenta previa :-


1- cervical blushing
2- painless vaginal bleeding
3- unengaged presenting part

34. The priority of nursing carr after a hemorrhoidectomy is ??


1- prevent infection
2- control pain

35. Low birth weight is defined as


1- birth weight < 2500gm
2- birth weight > 2500gm

36. Deferoxamine is administered in overdose of:


A. Iron
B. Calcium gluconate
C. Digoxin
D. Beta blockers

37. Which of the following blood tests should be performed before a blood transfusion?
A. Prothrombin and coagulation time
B. Blood typing and cross-matching
C. Bleeding and clotting time
D. Complete blood count (CBC) and electrolyte levels.

38. A client has a sucking stab wound to the chest. Which action should the nurse take
first?
A. Draw blood for a hematocrit and hemoglobin level.
B. Apply a dressing over the wound and tape it on three sides.
C. Prepare a chest tube insertion tray.
D. Prepare to start an I.V. line

39. In a 28-year-old female client who is being successfully treated for Cushing's
syndrome, the nurse would expect a decline in:
A. serum glucose level.
B. hair loss.
C. bone mineralization.
D. menstrual flow

5
40. When caring for a patient who has started anticoagulant therapy with
warfarin (Coumadin), the nurse knows not to expect therapeutic benefits for:
1.At least 12 hours
2. The first 24 hours
3. 2-3 days
4. 1 week

41. Jillian, a student nurse is assisting the staff nurse in performing a physical
examination on a newborn, which assessment, according to what she has learned
should be reported to the physician?
a) Head circumference of 40 cm
b) Chest circumference of 32 cm
c) Acrocyanosis and edema of the scalp
d) Heart rate of 160 and respirations of 40

42. Blood bank of body ?


A.Spleen
B.Liver
C.Heart
D.Red bone marrow

43. When administering an antibiotic or a vaccine , the nurse must be alert for the
possibility of :
a. overdoses and CNS depression
b. hypersensitivity and possible anaphylaxis
c. sings of increasing infection
d. orthostatic hypotension

Before irrigating a client’s NGT the nurse must first


a. assess breath sounds
b. instill 15 ml. Of normal saline
c. auscultate for bowel sounds
d. check the tube for placement

44. Common cause of deth in burns ??


A. Hemorrhage
B. Sepsis
C. Neurogenic shock
D. Hypovolemic shock

45. The most important preparation for a patient for a total laryngectomy includes which
of the following?
a. provide adequate humidity
b. aspirate secretions since cough mechanism is not as effective
c. elevate him to semi-Fowler's or sitting position
d. arrange for him to be visited by a laryngectomee

6
46. The best method of preventing the spread of infection is
a. isolating all patients suspected of having an infection
b. wearing rubber gloves when performing all nursing procedures
c. washing the hands thoroughly before & after each contact with a patient

47. Immediately after a child is admitted with acute bacterial meningitis , the nurse
should plan to :
a. assess the child’s vital signs every 3 hours
b. administer oral antibiotic medication as ordered
c. check the child’s level of consciousness every hour
d. restrict parental visiting until isolation is discontinued
48. Which technique is considered the gold standard for diagnosing DVT?
1. Ultrasound imaging
2. Venography
3. MRI
4. Doppler flow study

49. A drug must enter the bloodstream before it can act within the body. Which
parenteral administration route places a drug directly into the circulation, requiring no
absorption?
A. I.M.
B. Subcutaneous (subQ)
C. Intradermal
D I.V.

50. Deficiency of which of the following vitamin during pregnance may cause neural
tube defects ?
1 Folic acid
2 Riboflavin
3 Niacin
4 Thiamine

51. .Exceeding which of the following serum cholesterol levels significantly


increases the risk of coronary artery disease?
A. 100 mg/dl
B. 150 mg/dl
C. 175 mg/dl
D. 200 mg/dl

52. Ones the bleeding site has been determined , the first emergency
measure to institute during hemorrhage would be to :
a. apply a firm – pressure dressing
b. apply direct , firm – pressure over the bleeding area or the artery involved
c. apply a tourniquet just proximal to the wound

7
d. elevate the extremity

53. Which of the followng conditions is most commonly responsible for


myocardial infarction?
A. Aneurysm
B. Heart failure
C. Coronary artery thrombosis
D. Renal failure

54. Vitamin A is stored in?


A. Spleen
B. Pancreas
C. Liver
D. Gall bladder

55. A client is to have NPO for at least 2 hr before same-day surgery. A nurse
learns the client had half a glass of orange juice 3 hr prior to admission. The
nurse should:
Choose one of the following
A. Report the incident to the nursing supervisor.
B. Inform the surgery department.
C. Notify the anesthesiologist.
D. Reschedule the surgery.

56. The client arrives in the emergency department after a motor vehicle
accident. Nursing assessment findings include BP 80/34, pulse rate 120,
and respirations 20. Which is the client’s most appropriate priority nursing
diagnosis?
A. Alteration in cerebral tissue perfusion
B. Fluid volume deficit
C. Ineffective airway clearance
D. Alteration in sensory perception

57. when dealing with a client with aphasia , the nurse sWhould remember to :
a. wait for him to communicate
b. speak loudly to ensure that the massage is received
c. speak from the client’s side to avoid overload
d. encourage writing of massages

58. Which of the following diagnostic tools is most commonly used to determine the
location of myocardial damage?
A. Cardiac catheterization
B. Cardiac enzymes
C. Echocardiogram
D. Electrocardiogram

59. Ceftrixone belong to which generation of cepirosphorin..


A.1st generation C.3rd Gen
B.2nd Gen D.4rd Gen

8
60. Main complication of appendicitis is__
A) Perforation
B) Piles
C) Jaundice
D) Liver cirrhosis

61. Measles vaccine is given under the national lmmunization program.


a.6 weeks
b.6 months
C.9 months
D.18 months

62. Pressure sores involves all layers of skin and presence of septicemia and
fatal one is called.
a.stage l
B.stage ll
C.stage lll
D.stage lV

63. The vaginal fluid is strogly acid with ph 4.5 this mainly related to..
A.The fact that there are no gland in the vagina..
B.The mucus from the cervix.
C.The action of doderlein's bacili on glucogen found in the squamous
epithelium of the vagina
D.none of the above

60. A nurse is caring for a client in labor. The nurse determines


that the client is beginning in the 2nd stage of labor when which of the
following assessments is noted?
A. The client begins to expel clear vaginal fluid
B. The contractions are regular
C. The membranes have ruptured
D. The cervix is dilated completely

61. year-old patient presents in triage with left-sided chest pain, diaphoresis, and
dizziness. This patient should be prioritized into which category?
A. High urgent
B. Urgent
C. Non-urgent
D. Emergent

62. When Bp.elevated extremely rapid and endanger one of the vital organ is
known as.
1.primary hypertension
2 . secondary hypertension
3 malignant hypertension
4 . systolic hypertension

9
63. Clinical manifesations associated with smal bowl obstruction include all of the
following except.
1.persistent vomiting @ metabolic disturbance
2. passage of flatuse @stool.
3 .abd distribution @ paralyatic ileus
4 .Dehydration @ electrolytes imbalance

64. Early treatment of mild developmental dislocation of the hip joint is ..


a.spica cast
B.leg traction
C.Double diapering
D.open reduction

65. The mimimize pain associated with sickle cell anemia you should
administration.
1.fluid
2 .fresh frozen plasma
3.whole blood
4.calcium

66. The prevention measures of bacteremia as a compliction of Iv therapy. Include


all of the following except.
1. Use sterile techniques when inseting IV cannula.
2.change the inserted site at least every 96 hours.
3 . change intermittent administration sets every 24 hours.
4.never having solution longer than 24 hours

67. Most common home accidents in the home setting among toddlers in Gaze
strip are.
1.suffocation
2.drowning
3.poisoning
4.scalding and burns

68. All of the following are correct about hemophilia except.


1 . diseases characters by prolonged coagulation time.
2.Recessive sex diease usually affect females
3.manifest during infant and children
4.prevenation include avoid of aspirin and Im injection

Isolation of achild with measles belongs to what level of prevenation.


1.primary
2 . Secondary
3.Tertiary
4 . intermediate

10
69. All of the following are the characterstices of DKA except.
1.occures mainly in DM type two.
2 . associated with sever hyper glycemia.
3 present with metabolic acidosis.
4.Manifested by fruity .sweet breath.

70. Addison disease is result from.


1.adrrnocortical hyper function.
2.pituitary gland over stimulate
3.malignant tumor of pituitary gland.
4.Adernocortical hypo funaction

71. All of the following are clinical manifesations of premature infant except.
1.apneic attack
2.weak crying
3.excess vernix ceaeosa
4.Absent lanugo hair

72. Nurse teaches a client with heart failure to take oral


furosemide in the morning the reason for this to help.
1.Retard rapid drug absorpation.
2.Excrete excessive fluids accumulation at night.
3.prevent sleep disturbance during night
4.prevenation of electrolayte imbalance

73. s disorders characters by persistent depressive features not


amounting to depression disirders which last for more than 2 years.
1.cyclothymia
2.dysthymia
3.melancholia
4.Euthymia

74. The least incidence of breast cancer usually occurs among


women who have..
1.multiple pregnancies
2.Menarche occurs before age 12 years
3.first child after age 34 ye2
4.family history of breast cancer.

75. which one of the following is the most common initial finding
in client with a pulmonary embolus <PE?
A. sudden sever dyspnea and chest pain
B. chest pain with uneqal chest exapnasion
C. petechiae over the upper chest and shoulder
D.gradually ascending leg pain

11
76. Which of the following systems is the most likely origin of
pain the client describes as knifelike chest pain that increases in intensity with
inspiration?
A. Cardiac
B. Gastrointestinal
C. Musculoskeletal
D. Pulmonary

77. Before giving antibiotic to a patient which of the following


should be done first:
a. wound culture
b. blood tests
c. wound dressing
d. sensitivity testing

78. When the patient is diaphoretic , there is tachycardia and


decrease blood pressure , he is experiencing :
a. Neurogenic shock.
b. Hypovolemic shock
c. Hypothermia
d. Septicemia

79. Insulin is secreted from _______?


A. Spleen
B. Pancreas
C. Liver
D. Gall bladder

80. For breast self examinations, female patient should be


instructed to use the techniques of:
a-Auscultation & percussion.
b- Inspection & palpation.
c- Palpation & percussion.
d- Palpation & auscultation

81. The postoperative complication that deep breathing


exercises help most to prevent:
a-Phlebitis
b- Atlectasis
c- Hiccups.
d- Nausea.

82. To prevent the formation of thrombi and emboli in


postoperative patient, the nurse should do:
a- Have the patient lie still.
b- Teach foot and leg exercises.
d- Raise the knee gatch
c- Place pillows under the knees

12
83. The child birth weight triples at?
a. 3 months old
b. 6 months old
c. 12 months old
d. 15 months old

84. Paracentesis is best defined as:


a-The removal of fluids from the lungs, bronchi,& trachea.
b- The removal of fluids or air from the pleural cavity.
c- The removal of secretions from the stomach via NG tube.
d- The removal of fluids from the abdominal cavity

85. The nurse is caring for a client admitted with a pelvic fracture. Which assessment
receives priority status?
A. Bowel sound auscultation
B. Pupillary response to light
C. Assessing for hematuria
D. Cranial nerve 8 assessment

86. The nurse understands that the client with pernicious anemia will have which
distinguishing laboratory findings?
a.Schilling’s test, elevated
b. Intrinsic factor, absent.
c. Sedimentation rate, 16 mm/hour
d. RBCs 5.0 million

87. Consolidation of lungs is called?


A. Bronchiectasis
B.Pneumonia
C. Bronchitis
D. COPD

88. A pregnant client is diagnosed with partial placenta previa. In explaining the
diagnosis, the nurse tells the client that the usual treatment for partial placenta previa
is which of the following?
a. Activity limited to bed rest
b. Platelet infusion
c. Immediate cesarean delivery
d. Labor induction with oxytocin

89. Which of the following parameters should be checked before administering digoxin?
A. Apical pulse
B. Blood pressure
C. Radial pulse
D. Respiratory rate

13
90. A client is pregnant for the third time, she has a 3 years old
girl, and she also has
had a spontaneous abortion at 12 weeks gestation. Which of the following is a
correct method of recording her obstetric status?
a) Gravida 3, para 2 b) Gravida 2, para 1
c) Gravida 3, para 1 b) Gravida 2, para 2

91. Another name for the Whipple procedure is a ________.


A. Cholangiopancreatography
B. Pancreatoduodenectomy
C. Cholangiogram
D. Cholecystogram

92. Atropine so4 is given pre-operative to :


a. relax the patient
b. decrease the secretion
c. prevent intra-operative bleeding
d. decrease heart rate

93. The nurse understand that dialysis will be necessary when a child with chronic
kidney disease exhibits
A. Hypotension
B. Hypokalemia
C. Hypervolemia
D. Hypercalcemia

94. Hemolytic anemia is common with ______?


A. physiologic jaundice
B.Blood transfusions
C. Fungal Infections
D. All of above

95. The drug administration into spinal cavity is known as


a) spinal infusion
b)intrathecal
c) intra spinal
d) both B and C

96. Nursing responsibility before giving Digoxin:


a. Check BP
b. Check apical rate
c. Instruct patient to do deep breathing
d. Do ECG
97. The best diagnostic test for suspected leukemia is:
a. CBC
b. Blood chemistry
c. Bone marrow aspiration
d. LFT

14
98. Immediate management for reaction during blood transfusion :
a. Slow down the rate
b. Stop blood transfusion
c. Change blood transfusion with a new pack

99. The most important nursing responsibility following head injury is


a. monitor vital signs and record
b. monitor GCS –Glasgow coma scale
c. observe for restlessness

100. Main complain in the bronchiectasis is the?


A.Rcurrent productive cough
B. Chest pain
C. Red nails
D. Dry coughs

101. A woman with preeclampsia is receiving magnesium sulfate. The nurse assigned to
care for the client determines that the magnesium therapy is effective if:
A. Ankle clonus in noted
B. The blood pressure decreases
C.Seizures do not occur
D. Scotomas are present

102. Which of the following would be the priority nursing diagnosis for the adult client
with acute leukemia?
A. Oral mucous membrane, altered related to chemotherapy
B. Risk for injury related to thrombocytopenia
C. Fatigue related to the disease process
D. Interrupted family processes related to life-threatening illness of a family member

103. drug of choice for type 2 Diabetes mellitus is ?


A. Diet
B. Exercise
C.Metformin
D. Glimepride

104. Most common cause of infantile meningitis is -


A. Pneumococci
B. Meningococci
C. H.influenza
D. Streptococci

105. An injection into the gluteal site must be given in which quadrant of the buttocks
:a. upper – inner quadrant
b. upper-outer quadrant
c. lower – inner quadrant
d. lower – outer quadrant

15
106. A patient with asthma is receiving intravenous aminophylline .the adverse reaction
for which the nurse should observe is :
a. Oliguria
b. Bradycardia
c. hypotension
d. Tachycaredia

107. 112.The nurse has been notified to assist in a disaster. Which plan would be
included in the emergency care of a fracture?
A. Turn the clients to the left side
B. Immobilize the extremity by splinting above and below the fractured site
C. Provide manual traction of the fracture site
D. Reinsert any protruding bones and apply a sterile dressing

108. Another name for the Whipple procedure is a ________.


A. Cholangiopancreatography
B. Pancreatoduodenectomy
C. Cholangiogram
D. Cholecystogram

109. A client is pregnant for the third time, she has a 3 years old girl, and she also has
had a spontaneous abortion at 12 weeks gestation. Which of the following is a correct
method of recording her obstetric status?
a) Gravida 3, para 2
b) Gravida 2, para 1
c) Gravida 3, para 1
d) Gravida 2, para 2

110. metabolic acidosis ??


A.pH 7.45, pCO2 38, HCO3 23
B.pH 7.29, pCO2 22, HCO3 14
C.pH 7.31, pCO2 67, HCO3 32
D.pH 7.39, pCO2 43, HCO3 22

111. Ringing in the ear is called as___


A) Vertigo
B) Dizziness
C) Tinnitus
D) Vibration

112. A nerve which helps in the process of smelling is -


A. Optic nerve
B. Motor nerve
C. Facial nerve
D. Olfactory nerve

16
113. During conversation of Nurse John with a client, he observes that the client shift
from one topic to the next on a regular basis. Which of the following terms describes
this disorder?
a. Flight of ideas
b. Concrete thinking
c. Ideas of reference
d. Loose association

114. Clinical manifest of chronic glomerulonephritis is


a. Bun over 130 oer 85 mmhg
b. Bun level over 6o mg/dl
c. Slightly swollen joints
d. Apprehension

115. To decrease intraocular pressure following cataract surgery, the nurse should
instruct the client to avoid:
1. Lying supine.
2. Coughing.
3. Deep breathing.
4. Ambulation.

116. .The nurse is caring for a client receiving a sedative– hypnotic. Which adverse effect
associated with this drug therapy is the highest priority for the nurse?
1. Urinary incontinence
2. Activity intolerance
3. Risk for falls
4. Poor nutritional intake

117. The most common nursing care for the patient with DVT is
a) Administer prescribed medications
b) Elevate the leg above heart level
c) Check vital signs frequently
d) All of the above

118. A client with cancer is being evaluated for metastasis to the bone. Which laboratory value
would correlate with the suspected metastasis?
A. Serum phosphorus of 3.0 mg/dL
B. Alkaline phosphatase of 70 units/L
C.Serum calcium 16.0 mg/dL
D. Aldolase 3.5 units/dL

119. All of the following are methods of health promotion except


1.) immunization
2.) nutritional changes
3.) behavioural changes
4.) health education

120. The antidote for heparin is


a. warfarin b. protamine sulphate

c. narcan d. vitamin K

17
121. A client with cancer is being evaluated for metastasis to the bone. Which laboratory value
would correlate with the suspected metastasis?
A. Serum phosphorus of 3.0 mg/dL
B. Alkaline phosphatase of 70 units/L
C.Serum calcium 16.0 mg/dL
D. Aldolase 3.5 units/dL

122. Gravida refers to which of the following descriptions?


A. A serious pregnancy
B. Number of times a female has been pregnant
C. Number of children a female has delivered
D. Number of term pregnancies a female has had.

123. The nurse is conducting a physical assessment on a client with anemia. Which of the
following clinical manifestations would be most indicative of the anemia?
A. BP 146/88
B. Respirations 28 shallow
C. Weight gain of 10 pounds in 6 months
D. Pink complexion
124.
Carbohydrates are stored in the body in the form of :
a. Glucose
b. Glucagons
c. glycogen
d. glucose 6 phosphate

125. .A client arrives at a prenatal clinic for the first prenatal assessment. The client tells a nurse
that the first day of her last menstrual period was September 19th, 2013. Using Naegele’s rule,
the nurse determines the estimated date of confinement as:
A. July 26, 2013
B. June 12, 2014
C. June 26, 2014
D. July 12, 2014

126. The normal intraocular pressure is:


a. 2-7 mmHg
b.10-21 mmHg
c. 22-30 mmHg
d. 31-35 mmHg

127. Physiological barriers in communication include all except


a) difficulty in hearing
b) difficult in version
c) neurosis
d) difficulty in expression

128. Most commonest cause of C.V.A.(Cerebro vascular accident )


A. Hypertension
B. Tumor
C. Venous thrombosis
D. Embolism

18
129. When should you wash your hands?
a. when you notice they look or feel dirty
b. when the head nurse tells you to
c. at least twice a day
d. before and after contact with a patient

130. elect the fluid and electrolytes imbalances that result from renal failure.
A.fluid deficit,hyperkalemia,hypercalcemia,hypermagnesemia .
B.fluid deficit,hypokalemia,hypernatremia,hypocalcemia.
C.fluid overload,hyperkalemia,hypocalcemia,hypermagnesemia .
D.fluid overload,hypokalemia,hypernatremia,hypocalcemia.

First investigation for community acquired pneumonia is ?


.
A. Xray chest
B. Blood culture
C. Sputum culture
D. Urine antigen

131. Neonatal neurological complications can be prevented during pregnancy by


supplements of ?
A. Iron
B. Folic
C. B12
D. None of above
132.
Which of the following patients is at greater risk for contracting an infection?
A. A patient with leukopenia
B. A patient receiving broad-spectrum antibiotics
C. A postoperative patient who has undergone orthopedic surgery
D. A newly diagnosed diabetic patient

133. Inhaled steroids are drug of choice for ______?


A. COPD.
B. Asthma.
C. Myocardial infarction.
D. Spenomegally.

134. The Staff Nurse working in a post operative ward planing wound dressing. When
changing a sterile surgical dressing, the nurse must first:
A. wash her hands.
B. apply sterile gloves.
C. remove the old dressing with clean gloves.

D. open sterile packages, and moisten the dressings with sterile saline solution.
135. Which patient is at greatest risk for developing a urinary tract infection (UTI)?
1. A 35 y.o. woman with a fractured wrist
2. A 20 y.o. woman with asthma
3. A 50 y.o. postmenopausal woman
4. A 28 y.o. with angina

19
136.
The white to yellow powder that covers the face of the newborn and results from the
skin secretions is called…
a) Vernix caseosa
b) Milia
c) Nevus
d) Lanugo

137. Which assessment data would indicate to the nurse that the client would be at risk
for a hemorrhagic stroke?
1. A blood glucose level of 480 mg/dl.
2. A right-sided carotid bruit.
3. A blood pressure of 220/120 mmHg.
4. The presence of bronchogenic carcinoma

138. When arranging a patient's room, you should do all of the following EXCEPT
a. check signal cords
b. fix the back and knee rests as directed
c. administer medications
d. check lighting

139.
Which of the following should you observe and record when admitting a patient?
a. color of the stool and amount of urine voided
b. how much the patient has eaten and drunk
c. bruises, marks, rashes, or broken skin
d. requests the patient makes

140. Clients with gestational diabetes are usually managed by which of the following
therapies?
A. Diet
B. NPH insulin (long-acting)
C. Oral hypoglycemic drugs
D. Oral hypoglycemic drugs and insulin
141. When newborns have been on formula for 36-48 hours, they should have a:
A. Screening for PKU
B. Vitamin K injection
C. Test for necrotizing enterocolitis
D. Heel stick for blood glucose leve

142. During a prenatal visit at 38 weeks, a nurse assesses the fetal heart rate. The nurse
determines that the fetal heart rate is normal if which of the following is noted?
A. 80 BPM
B. 100 BPM
C. 150 BPM
D. 180 BPM

143. Which of following is a viral disease ?


A. Impetigo. C. Hydatid cyst
B. Herpes labialis. D. I v cannula infections

20
144. Which sign " Welcome sign of pregnancy "?
A. Engagement
B. Quickening
C. Lightening
D. Internal rotation

145.How should the nursr intervene ??


A- Do nothing until the chemical agent is identified .
B- Irrigate the wounds with water.
C- Wash the wounds with soap and water and apply abarrier cream

146. The nurse is to administer an I.M. injection into a client's left vastus lateralis muscle.
How should the nurse position the client?
A. Lying supine
B. Lying on the stomach
C. Lying on the left side
D. Lying on the right side

147. Which pulse should the nurse palpate during rapid data collection of an unconscious
adult?
A. Radial
B. Brachial
C. Femoral
D. Carotid

148. Diagnosis of Asthma is confirmed by ?


A. Spirometry
B. Xray chest
C. Methacholine challenge
D. Steroid trial

149.
The most serious problem that wrinkles in the bedclothes can cause is
a. restlessness
b. sleeplessness
c.decubitus ulcers
d. bleeding and shock

150. The nurse knows which of the following body systems is responsible for the
production of erythropoietin?
1. Urinary system
2. Cardiovascular system
3. Lymphatic system
4. Endocrine system

151. A company driver is found at the scene of an automobile accident in a state of


emotional distress. He tells the paramedics that he feels dizzy, tingling in his fingertips,
and does not remember what happened to his car. Respiratory rate is rapid at

21
34/minute. Which primary acid-base disturbance is the young man at risk for if medical
attention is not provided?
A. Respiratory Acidosis
B. Respiratory Alkalosis
C. Metabolic Acidosis
D. Metabolic Alkalosis

152.
he nurse is observing a student nurse administer eyedrops, as shown in the figure.
What should the nurse instruct the student to do?
a. Move the dropper to the inner canthus.
b. Have the client raise her eyebrows.
c. Administer the drops in the center of the lower lid.
d. Have the client squeeze both eyes after administering the drops.

153. You touch the inside of the sink while rinsing soap off your hands. You should
a. allow the water to run over your hands for two minutes
b. dry your hands and turn off the faucet with the paper towel
c.repeat the wash from the beginning
d. none of the above

154. To decrease the pain associated with an episiotomy immediately after birth, the
nurse would:..
1) Offer warm blankets
2) Encourage the woman to void
3)Apply an ice pack to the site
4) Offer a warm sitz bath
155.
A nurse is caring for an infant that has recently been diagnosed with a congenital heart
defect. Which of the following clinical signs would most likely be present?
1. Slow pulse rate
2. Weight gain
3. Decreased systolic pressure
4. Irregular WBC lab values

156. scurvy is a deficiency of :


A. Vit a
B. Vit b
C. Vit c
D. Vit d
157.
Which of the following would be most appropriate for a nurse to use when describing
menarche to a 13-year-old?
A. A female’s first menstruation or menstrual “periods”
B. The first year of menstruation or “period”
C. The entire menstrual cycle or from one “period” to another
D. The onset of uterine maturation or peak growth

158. During labor induction with oxytocin, the nurse knows that relaxation of vascular
smooth muscle can cause:
A. Hypertension, premature atrial contractions
B. Hyperglycemia, premature ventricular contractions

22
C. Hyperglycemia, hypertension
D. Hypotension and flushing

159. A client with a history of Polycystic Kidney Disease is admitted to the Renal Unit for
evaluation for dialysis. Which of the following lab values would be MOST significant in
determining renal function?
a. Creatinine 8.7 mg/dl
b. BUN 90 mg/dl
c. Serum K+ 7.0 MEq/l
d. Uric Acid 7.5

A client presents with blistering wounds caused from an unknown chemical agent . D-
Insret 20-gauga I.V.catheter and infuse normal saline solution at 150 ml/hour .

160. Which of the following is a contraindication for use of oxytocin to induce labor?
A. Missed abortion
B. Placenta previa
C. Hyperbilirubinemia
D. Pregnancy past due date

161. which of the following organs is a primary site for the metabolism of drugs ?
a- Heart
b- Liver
c- Pancreas
d- intestine

162. Weight gain is associated with CHF and congenital heart deficits.

yo likes salty foods. Nurse instruct to lower Na intake to:


a. Promote urea nitrogen excretion
b. Improve glomerular filtration rate
c. Increase K absorption
d. Reduce edema

162. As a safety measure, when you give mouth care to an unconscious patient, you
should position the patient
a. on her or his back
b. in semi-Fowler's position
c.with the head turned to the side
d. in the supine position

163. Which of the following statements about shivering is correct?


A. Shivering is a response controlled by the brainstem.
B. Shivering can occur in the absence of hypothermia.
C. Shivering is effectively treated with small doses of naloxone.
D. Shivering is an uncomfortable, though harmless, effect of anesthesia.
164. The largest gauge needle is :
a- 19 gauge
b- 20 gauge

23
c- 21 gauge

165.
The nurse is caring for a child with anemia. Which of the following would indicate to the
nurse that the client’s condition is deteriorating?
1. Circumoral pallor
2. Fatigue with exertion
3. Cardiac murmur
4. Irritability
.
166. Im injection into deltoid muscle should be limited to :
a- 2.5 ml of solution
b- 1 ml of solution
c- 0.5 ml of solution
d- 2 ml of solution

167. The inatant source of Energy in human body is ?


A. Fatty acids
B. Glucose
C. Amino acids
D. Fructose

168. You are told to put a patient in Fowler's position. Before changing the position of the
patient's bed, you should
a. open the window
b. explain the procedure to the patient
c. check with the patient's family
d. remake the bed

168.Adam'S Apple is the _______?


A. Cricoid cartilage
B. Thyroid cartilage
C. Hyoid bone
D. None of above

169.One important way to reduce the incidence of decubitus ulcers is to


a. keep the patient in bed
b. force fluids every 2 hours
c.change position every 2 hours

170.Blood in the urine is called ?


1 uremia
2.haematuria
3.pyuria
4.dysuria

171.hen making a bed, you can save steps and time if you
a.assemble all needed linen before starting to make the bed
b. tuck in bottom linen and top linen at the foot of bed before going to the head of bed
c. use only fitted sheets
d. ask for help from the head nurse

24
172What is the function of temporal lob of brain......
A. Hearing
B. Vision
c.problem solving
d.non above
173.Which of the following can lead to infertility in adult males?
A. German measles
B. Orchitis
C. Chicken pox
D. Rubella

174.A patient is admitted to the hospital with a calcium level of 6.0 mg/dL. Which of
the following symptoms would you NOT expect to see in this patient?
A. Numbness in hands and feet.
B. Muscle cramping.
C . Hypoactive bowel sounds.
D. Positive Chvostek's sign…

A patient on the cardiac telemetry unit unexpectedly goes into ventricular fibrillation. The advanced
cardiac life support team prepares to defibrillate. Which of the following choices indicates the
correct placement of the conductive gel pads?
A. The left clavicle and right lower sternum.
B. Right of midline below the bottom rib and the left shoulder.
C. The upper and lower halves of the sternum.
D . The right side of the sternum just below the clavicle and left of the precordium.

A nurse is administering IV furosemide to a patient admitted with congestive heart failure. After the
infusion, which of the following symptoms is NOT expected?
A. Increased urinary output.
B. Decreased edema.
C. Decreased pain.
D. Decreased blood pressure.

C. Furosemide, a loop diuretic, does not alter pain. Furosemide acts on the kidneys to increase
urinary output. Fluid may move from the periphery, decreasing edema. Fluid load is reduced,
lowering blood pressure.

A patient has been hospitalized with pneumonia and is about to be discharged. A nurse provides
discharge instructions to a patient and his family. Which misunderstanding by the family indicates
the need for more detailed information?
A.The patient may resume normal home activities as tolerated but should avoid physical exertion
and get adequate rest.
B.The patient should resume a normal diet with emphasis on nutritious, healthy foods.

25
C. The patient may discontinue the prescribed course of oral antibiotics once the symptoms have
completely resolved.
D.The patient should continue use of the incentive spirometer to keep airways open and free of
secretions.

Which of the following urinary symptoms does the pregnant woman most frequently experience
during the first trimester?
A. Dysuria
B. Frequency
C. Incontinence
D. Burning

Which of the following medical conditions is commonly found in clients with bulimia nervosa?
A. Allergies
B. Cancer
C. Diabetes mellitus
D. Hepatitis A

C. Diabetes mellitus. Bulimia nervosa can lead to many complications, including diabetes, heart
disease, and hypertension. The eating disorder isn’t typically associated with allergies, cancer, or
hepatitis A

All are the non-modifiable risk factors for atherosclerosis except


1. high cholesterol intake
2. age
3. heredity
4. gender

Approximate increase in weight during pregnancy is :


A. 10.5 kg
B. 12.5 kg
C. 11.5 kg
D. 13.5 kg

Q. The first drug of choice for a patient admitted with status epilepticus is
1. phenytoin
2. carbamezepine
3. diazepam ...

At birth normal head circumference of a new born is ;


A. 28 to 30 cms
B. 30 to 32 cms
C. 33 to 35 cms
D. 36 to 38 cms

While assessing a newborn with cleft lip, the nurse would be alert that which of the following will
most likely be compromised?
A. Sucking ability
B. Respiratory status
C. Locomotion
D. GI function

26
The nurse is caring for a client with acute pulmonary edema. To immediately promote oxygenation
and relieve dyspnea, which of the following interventions is appropriate?
A. Administer oxygen.
B. Have the client take deep breaths and cough.
C. Place the client in high Fowler's position.
D. Perform chest physiotherapy.

When a patient is vomiting post-operatively , the most important nursing


to prevent :
a. dehydration
b. aspiration
c. rupture of suture line
d. metabolic acidosis

Atropin is given to patient undergoing oesophagoscopy to;


a. Increase sedation
b. Reduce pain
c. Decrease secretion
d. Increase alertness

When the skin , whole epidermis , dermis and the underlying structures are affected in a burn , it is
called :
a. first degree burn
b. second degree burn
c. third degree burn
d. none of the above

A patient arrives at the emergency department complaining of mid-sternal chest pain. Which of the
following nursing action should take priority?
A. A complete history with emphasis on preceding events.
B. An electrocardiogram.
C. Careful assessment of vital signs.
D. Chest exam with auscultation.

The nurse should visit which of the following clients first?


a. The client with diabetes with a blood glucose of 95mg/dL
b. The client with hypertension being maintained on Lisinopril
c. The client with chest pain and a history of angina
d. The client with Raynaud’s disease

Which of the following changes in resp functioning during pregnancy is considered normal?
1. increased tidal volume
2. increases expiratory volume
3. decreased inspiratory capacity
4. decreased oxygen consumption

If a S/C injection is to be given, in to which level of tissue must the solution be released :
a. Epidermis
b. Dermis
c. Subcutaneous
d. Any of the above

27
When you obtain a clean-catch urine specimen, you should
a. wash the patient's hands
b.use clean techniques
c. use sterile techniques
d. perform the procedure in the bathroom

A client in labor is transported to the delivery room and is prepared for a cesarean delivery. The
client is transferred to the delivery room table, and the nurse places the client in the:
A. Trendelenburg’s position with the legs in stirrups
B. Semi-Fowler position with a pillow under the knees
C. Prone position with the legs separated and elevated
D. Supine position with a wedge under the right hip

D. Supine position with a wedge under the right hip. Vena cava and descending aorta compression
by the pregnant uterus impedes blood return from the lower trunk and extremities. This leads to
decreasing cardiac return, cardiac output, and blood flow to the uterus and the fetus. The best
position to prevent this would be side-lying with the uterus displaced off of abdominal vessels.
Positioning for abdominal surgery necessitates a supine position; however, a wedge placed under
the right hip provides displacement of the uterus

You administer ranitidine (Zantac) as ordered prior to a planned cesarean birth to


A. Neutralize urine acidity.
B. Promote uterine contractions.
C. Delay uterine contractions.
D. Decrease gastric secretions.

A child is admitted to the hospital with an uncontrolled seizure disorder. The admitting physician
writes orders for actions to be taken in the event of a seizure. Which of the following actions would
NOT be included?
A. Notify the physician.
B . Restrain the patient's limbs.
C. Position the patient on his/her side with the head flexed forward.
D. Administer rectal diazepam.

Hierarchy of needs was given by -


A. Skinner
B. Freud
C. Erickson
D. Maslow

When performing a newborn assessment, the nurse should measure the vital signs in the following
sequence:
1. Pulse, respirations, temperature
2. Temperature, pulse, respirations
3. Respirations, temperature, pulse
4. Respirations, pulse, temperature

How many chromosomes are there in a human mature ovum?


A. 46

28
B. 42
C. 21
D. 23

A patient arrives in the emergency department and reports splashing concentrated household
cleaner in his eye. Which of the following nursing actions is a priority?
A . Irrigate the eye repeatedly with normal saline solution.
B. Place fluorescein drops in the eye.
C. Patch the eye.
D. Test visual acuity.

Sickle cell disease commonly presents with?


A. Hepatitis
B. Renal calculi
C. Biliary disease
D. Hemolytic jaundice

A newly diagnosed 8-year-old child with type I diabetes mellitus and his mother are receiving
diabetes education prior to discharge. The physician has prescribed Glucagon for emergency use.
The mother asks the purpose of this medication. Which of the following statements by the nurse is
correct?
A. Glucagon enhances the effect of insulin in case the blood sugar remains high one hour after
injection.
B . Glucagon treats hypoglycemia resulting from insulin overdose.
C. Glucagon treats lipoatrophy from insulin injections.
D. Glucagon prolongs the effect of insulin, allowing fewer injections.

In case of development of urticaria one-half hour after the blood transfusion has begun, the first
action by the nurse should be
1. to slow the rate of infusion
2. to administer anti-histaminic agent
3. to note vital signs
4. to stop the infusion

A client being treated with sodium warfarin has a Protime of 120 seconds. Which intervention
would be most important to include in the nursing care plan?
A. Assess for signs of abnormal bleeding
B. Anticipate an increase in the Coumadin dosage
C. Instruct the client regarding the drug therapy
D. Increase the frequency of neurological assessments

A 5-year-old is admitted to the hospital with suspected meningitis. Which nursing intervention
would be included in the child's plan of care?
A. Dim the lights and quiet the room as needed.
B. Play music the child enjoys.
C. Provide a high-calorie diet.
D. Measure head circumference.

Causes of suicide
a) worthlessness
b) delusion

29
c) helplessness
d) all of the above

A client arrives from surgery to the postanesthesia care unit. Which of the following respiratory
assessments should the nurse complete first?
1. Oxygen saturation.
2. Respiratory rate.
3. Breath sounds.
4. Airway flow.

A nurse is assessing a clinic patient with a diagnosis of hepatitis A. Which of the following is the
most likely route of transmission?
A. Sexual contact with an infected partner.
B. Contaminated food.
C. Blood transfusion.
D. Illegal drug use.

Which of the following organisms most commonly causes community-acquired pneumonia in


adults?
1. Haemiphilus influenzae
2. Klebsiella pneumoniae
3. Steptococcus pneumoniae
4. Staphylococcus aureus

Cystitis to the inflammation of ;


A. Gall bladder
B. Urinary bladder
C. Cyst
D. Liver

When documenting information in a client's medical record, the nurse should:


A. erase any errors.
B. use a #2 pencil.
C. leave one line blank before each new entry.
D. end each entry with the nurse's signature and title.

Cyanosis is caused in cash of-


A. Lack of water
B. Lack of blood
C. Lack of glucose
D. Lack of oxygen

What is the hormone responsible for production of milk for breast feeding?
Prolactin hormone which secreted from anterior pituitary gland..

After a tonsillectomy,a child is brought to the pediatric unit.The nurse places the child in which
appropriate position?
1.Supine
2.trendelenburg
3.prone
4.high-fowler's

30
Which the longest smooth muscle in the human body
A. In femur
B. In humerus
C. In stomach
D. In pelvic

A client newly diagnosed with diabetes mellitus is experiencing difficulty with self-administration of
insulin. Despite further teaching, the client shows little improvement. What action by the nurse is
most appropriate?
A. Inform the physician of the lack of progress and request that discharge be delayed.
B. Consult with family members and begin family insulin administration education.
C. Notify the physician of the client's lack of progress and request a diabetes education consult.
D. Explain to the physician that a family diabetes education class might be beneficial to the client.

A hospitalized patient has received transfusions of 2 units of blood over the past few hours. A
nurse enters the room to find the patient sitting up in bed, dyspneic and uncomfortable. On
assessment, crackles are heard in the bases of both lungs, probably indicating that the patient is
experiencing a complication of transfusion. Which of the following complications is most likely the
cause of the patient's symptoms?
A. Febrile non-hemolytic reaction.
B. Allergic transfusion reaction.
C. Acute hemolytic reaction.
D . Fluid overload.

Which situation could lead to metabolic acidosis in a client?


1. Frequent vomiting.
2. Nasogastric suctioning.
3. Hyperventilation.
4. ASA overdose.
The most common cause of an increase in a hemoglobin
(Hgb) value is:
1. Dehydration.
2. Infection.
3. Malnutrition.
4. Antibiotic use.
Immunity of BCG vaccination lasts:
A) 10 Years
B) 12 Years
C) 15 Years
D) 5 Years

Ceftrixone belong to which generation of cepirosphorin..


A.1st generation
B.2nd Gen
C.3rd Gen
D.4rd Gen

Early signs and symptoms of local anesthetic toxicity include ALL BUT ONE of the following.
Indicate the exception:
A. Tinnitus
B. Perioral numbness
C. Dizziness

31
D. Hypertension

Aortic valve and pulmonary valve also called ..


A.A.V valve
B.semilunar valve
C.bicuspid valve
D.tricuspid valve

Which of the following drugs requires regular hearing exams with prolonged use?
A. Streptomycin
B. Ciprofloxacin (Cipro)
C. Hydromorphone (Dilaudid)
D. Isoniazid (INH)

Which of the following definitions best describes gastritis?


1. Erosion of the gastric mucosa
2. Inflammation of a diverticulum
3. Inflammation of the gastric mucosa
4. Reflux of stomach acid into the esophagus

Pernicious anemia is caused by a deficiency of :


A. Folic acid
B. Iron
C. Vitamin D
D. Vitamin B12

Most dangerous form of fat in human body ?


A. Visceral Fat
B. Fat in thighs
C. Fat over shoulders
D. Fat in neck

The first action that the nurse should take if she finds the client has an O2 saturation of 68% is:
❍ A. Elevate the head
❍ B. Recheck the O2 saturation in 30 minutes
❍ C. Apply oxygen by mask
❍ D. Assess the heart rate

A nurse is preparing to administer furosemide (Lasix) 40 mg by Intravenous (IV) injection (IV push)
to a client.The nurse administers the medication over a period of:
1. 10 seconds
2. 30 seconds
3.1 minute
4. 5 minutes

In case of heparin over dose, which agent should be used ?


1. Diltiazem
2. Protamine
3. Imferon
4. Warfarin

32
The permanent cessation of menstruation is:
A. Amenorrhea
B. Menopause
C. Oligomenorrhea
D. Hypomenorrhea

Q.When haemoglobin combines with oxygen, it is known as


1. oxyhemoglobin
2. deoxyhemoglobin
3. reduced hemoglobin
4. oxidized haemoglobin

After the initial phase of the burn injury, the client’s plan of care will focus primarily on:
a. Helping the client maintain a positive self concept.
b. Promoting hygiene.
c. Preventing infection.
d. Educating the client regarding care of the skin grafts.

Q.Deficiency of which of the following vitamin during pregnance may cause neural tube defects ?
1 Folic acid
2 Riboflavin
3 Niacin
4 Thiamine

The nurse should question the order of acetaminophen


(Tylenol) for which client?
1. A client with cirrhosis of the liver
2. A client with chronic obstructive pulmonary disease
3. A client with breast cancer
4. A client who is taking warfarin (Coumadin)

A priority goal for the hospitalized client who 2-days earlier had a total laryngectomy with creation
of a new tracheostomy would be to:
a. Decrease secretions
b. Instruct the client in caring for the tracheostomy
c. Relieve anxiety related to the tracheostomy
d. Maintain a patent airway
A patient is in the second stage of labor. During this stage, how frequently should the
nurse in charge assess her uterine contractions?
a.Every 5 minutes
b.Every 15 minutes
c.Every 30 minutes
d.Every 60 minutes

Which is the MOST appropriate action for the nurse to take before administering
digoxin?...
A. Monitor potassium level...
B. Assess blood pressure....
C. Evaluate urinary output....
D. Avoid giving with thiazide diuretic

33
Which of the following indicates the state of health of a community and the success or
failure of health work?
A. Vital events
B. Morbidity and mortality rates
C. Literacy rate
D. Community diagnosis

A patient received 6 units of regular insulin 3 hours ago. The nurse would be MOST
concerned if which of the following was observed?
A. kussmaul respirations and diaphoresis
B. anorexia and lethargy
C.diaphoresis and trembling
D. headache and polyuria

Blurred vision or halos are signs of:..


A. Subtherapeutic digoxin levels...
B. Digoxin toxicity....
C. Nothing related to digoxin....
D. Corneal side effects of digoxin
Which neurotransmitter has been implicated in the development of Alzheimer’s disease?
a. Acetylcholine
b. Dopamine
c. Epinephrine
d. Serotonin

The nurse examines the breast of a primipara 12 hr after delivery. Which of the following is
expected at this time?
1. Soft, no change from delivery.
2.Harder than before delivery.
3. Tender to touch.
4. Smaller than before delivery with a small amount of bloody discharge.

Which of the following is a contraindication for digoxin administration?...


A. Blood pressure of 140/90....
B. Heart rate above 80....
C. Heart rate below 60....
D. Respiratory rate above 20

Abnormal accumulation of free fluid in peritoneal cavity is called..........?


A. Edema.
B.Empyema .
C. Ascites.
D. Pleural effusion.

The agent contraindicated with warfarin treatment is


1. aspirin
2. ranitidine
3. codeine
4. isaphgol

34
The action of medication is inotropic when it:...
A. Decreased afterload...
B. Increases heart rate...
C. Increases the force of contraction....
D. Is used to treat CHF

The term placenta praevia is____


A) Early development of placenta
B) Early placental expulsion
C) Implantation of placenta in lower uterine segment
D) None
Most common cause of hemetemesis is ?
A. Esophageal varices
B. Gastritis
C. NSAIDS
D. Delfouy leisons

Women are more likely to complain of which symptoms when experiencing a myocardial
infarction (MI)?
1. Unexplained fatigue, upper abdominal discomfort, and dizziness
2. "Bandlike" chest pain and nausea
3. General malaise and a feeling of heaviness
4. Crushing chest pain, shortness of breath, and diaphoresis

To compensate for decreased fluid volume (hypovolemia), the nurse can anticipate which
response by the body?
a. Bradycardia
b. Tachycardia
c. Increased urine output
d. Vasodilation

A nursing home client is admitted to the hospital with a pressure ulcer involving full
thickness loss extending to the bone. The nurse documents the pressure ulcer as being at
which of the following stages?
a. Stage I
b. Stage II
c. Stage III
d. Stage IV

The constant presence of an etiologic factor and a disease in a PARTICULAR AREA is


called:
A. Sporadic
B. Endemic
C. Epidemic
D. Pandemic

The nurse is collecting data on a client before surgery. Which statement by the client
would alert the nurse to the presence of risk factors for postoperative complications?
A "I haven't been able to eat anything solid for the past 2 days."
B. "I've never had surgery before."
C. "I had an operation 2 years ago, and I don't want to have another one."

35
D."I've cut my smoking down from two packs to one pack per day."

A patient has dyspnea, edema, hepatomegaly, crackles and jugular vein distention. What
is the cause ?
a) heart failure
b) pulmonary embolism
c) cardiac tamponade
d) tension pneumothorax

A nurse is monitoring a 7-year-old child who sustained a head injury in a motor vehicle
accident for signs of increased intracranial pressure(ICP).
The nurse assesses the child for which early sign of increased ICP?
1.decerebrate posturing.
2.papilledema.
3.alteration in pupil size.
4. nausea.

A client has a serum calcium level of 7.2 mg/dl. During the physical examination, the nurse
expects to assess:
a. Trousseau’s sign.
b. Hegar’s sign.
c. Homans’ sign.
d. Goodell’s sign.

Which of the following terms is used to refer to inflammation of the renal pelvis?
a. Pyelonephritis
b. Interstitial nephritis
c. Urethritis
d. Cystitis

A nurse enters the room of a client newly diagnosed with type 2 diabetes mellitus. The
client states “I feel really dizzy and shaky." Which of the following actions should the nurse
perform?
A. Notify the provider.
B. Give 8 ounces of fruit juice.
C. Administer 10 units of lispro.
D. Obtain a blood glucose level.

When administering an antiarrhythmic agent, which of the following assessment


parameters isthe most important for the nurse to evaluate?..
A. ECG...
B. Pulse rate....
C. Respiratory rate
D. Blood pressure

Which of the following urinary symptoms does the pregnant woman most frequently
experienceduring the first trimester:...
a. frequency....
b. dysuria....
c. incontinence...
d. burning

36
Which of the following assessments should be a priority immediately after nasal surgery?
1. Assessing the client’s pain.
2. Inspecting for periorbital ecchymosis.
3. Assessing respiratory status.
4. Measuring intake and output.

Where do most Myocardial Infarctions occur?


1.Right ventricular wall
2. Left ventricular wall
3.Right atrial wall
4.Left atrial wall

A female client is brought to the emergency department with second- and third-degree
burns on the left arm, left anterior leg, and anterior trunk. Using the Rule of Nines, what is
the total body surface area that has been burned?
A. 18%
B. 27%
C. 30%
D. 36%

The risk of bleeding is increased when warfarin is combined with which of the following?
1. Barbiturates
2. Vitamin K
3. Salicylates
4. Vitamin C

A 39 year old client has been diagnosed with end-stage renal disease and is on the
transplant waiting list. The client has been receiving dialysis through a subclavian central
vein catheter while an arteriovenous fistula is maturing. Besides dialysis access, the
surgical floor nurse can utilize this subclavian central vein catheter for which of the
following?
a. Nothing
b. Blood draws only
c. Infusion of normal saline (0.9% NS) and obtaining blood draws
d. Infusion of medications, all intravenous fluids, and obtaining blood draws

Which is the first biomarker to become elevated after the onset of symptoms of Myocardial
Infarctions?
1. CK-MB
2. Myoglobin
3. Troponin I

The client with a diagnosis of heart failure reports frequently awakening during the night
with the need to urinate. The nurse offers which explanation?
a) The blood pressure is lower when the client is recumbent and this causes the kidneys to
work harder; therefore, more urine is produced.
b) Fluid that is held in the lungs during the day becomes part of the circulation at night and
the kidneys produce an increased amount of urine.
c)Edema is collected in dependent extremities during the day; at night when the client lays

37
down, it is reabsorbed into the circulation and excreted by the kidneys.
d) When the client is in the recumbent position, more pressure is put on the bladder with
the result of increased need to urinate

The most toxic antiarrhythmic agent is:..


A. digoxin (Lanoxin)...
B. lidocaine (Xylocaine)...
C. amiodarone (Cordarone)..
D. quinidine (Cardioquin)

The artery which supply blood to head and neck is -


A. Coronary artery
B. Intra costal artery
C. Superior artery
D. Carotid artery
\
-week-old infant has surgery to repair a cleft lip. Which is the priority concern when the child
returns from the recovery room?
A. Feeding method
B. Maintaining airway
C. Preventing scarring of the lip
D. Preventing incisional infection

743. The nurse is aware that a healthy newborn’s respirations are:


A. Regular, abdominal, 40-50 per minute, deep
B. Irregular, abdominal, 30-60 per minute, shallow
C. Irregular, initiated by chest wall, 30-60 per minute, deep
D. Regular, initiated by the chest wall, 40-60 per minute, shallow..

When a patient in the terminal stages of lung cancer begins to exhibit loss of consciousness, a
major nursing priority is to:
A. Protect the patient from injury
B. Insert an airway
C. Elevate the head of the bed
D. Withdraw all pain medications

To assess the kidney function of a patient with an indwelling urinary (Foley) catheter, the nurse
measures his hourly urine output. She should notify the physician if the urine output is:
A. Less than 30 ml/hour
B. 64 ml in 2 hours
C. 90 ml in 3 hours
D. 125 ml in 4 hours

A female client undergoes yearly mammography. This is a type of what level of prevention?
A. primary
B. secondary
C. tertiary
D. nota

All are features of Pre-eclampsia Except ?


A. Seizures

38
B. Hypertension
C. Pedal edema
D. Protienuria

B.C.G vaccine prevent from disease...


A.Typhoid
B.tuberculosis
C.hepatits
D.h.i.v

Which of the following will not present in patient with duodenal ulcer?
A: pain
B: heartburn
C:nausea
D: weight loss

Which of the following agent causes discolouration of teeth if taken with milk ?
1. Tetracycline
2. Penicillin
3. Metronidazole
4. Ciprofloxacin

After cardiac surgery, a client’s blood pressure measures 126/80. The nurse determines that the
mean arterial pressure (MAP) is which of the following?
1. 46 mm Hg
2. 80 mm Hg
3. 95 mm Hg
4. 90 mm Hg

Most common cause of UTI in adults?


A. E.colo
B. Meningococciii
C. Streptococci
D. Staphylococci

Which of the following is the most common cause of dementia among elderly persons?
A. Parkinson’s disease
B. Multiple sclerosis
C. Amyotrophic lateral sclerosis (Lou Gerhig’s disease)
D. Alzheimer’s disease

Cervical softening and uterine souffle are classified as which of the following?
A. Diagnostic signs
B. Presumptive signs
C. Probable signs
D. Positive signs

What is the primary reason for administering morphine to a client with an MI?
1. To sedate the client
2. To decrease the client’s pain
3. To decrease the client’s anxiety
4. To decrease oxygen demand on the client’s heart

The pregnant woman complains of numbness and "Pins and needles" in her fingers and hands. the
condition is called as?
A) Carpal tunnel syndrome

39
B) Vericosites
C) Pica
D) Heart burn
Most common cause of community acquired pneumonia is ?
A. Proteus
B. Listeria
C. Streptococci
D. Staphylococci

Which of the following would lead the nurse to suspect that a child with meningitis has developed
DIC?
a. Hemorrhagic skin rash
b. Edema
c. Cyanosis
d. Dyspnea on exertion

A common abnormal laboratory result associated with the development of peripheral


vascular disease (PVD) is:
1. High serum calcium level.
2.High serum lipid levels.
3. Low serum potassium level.
4. Low serum lipid levels.

A client is found in the room holding his hands to his throat and his lips are turning blue.
Which action by the nurse is most appropriate?
A. Lying the client on the floor and administering abdominal thrust.
B. Ask the client to cough.
C. Placing both fists around the abdomen above the umbilicus and administering
abdominal thrust.
D. Attempting a blind finger sweep.

Fertilisation of sperm & ovum occurs in _____part of females genital tract ?


A. Uterus
B. Ovaries
C. Abdomen
D. Fallopian tubes

Which goal is the most realistic and appropriate for a client diagnosed with Parkinson's
Disease?
a.To cure the disease
B.To stop progression of the disease
C.To begin preparation for terminal care
D.To maintain optimal body function

What is the most common symptom in a client with abdominal aortic aneurysm?
A. Abdominal pain
B. Diaphoresis
C. Headache
D. Upper back pain

40
What is the most common complication of a myocardial infarction?
A. Cardiogenic shock
B. Heart failure
C. Arrhythmias
D. Pericarditis

A 19-year-old comes into the emergency department with acute asthma. His respiratory
rate is 44 breaths/minute, and he appears to be in acute respiratory distress. Which of the
following actions should be taken first?
1. Take a full medication history
2. Give a bronchodilator by nebulizer
3. Apply a cardiac monitor to the client
4. Provide emotional support to the client.

A sudden redness of the skin is know .


A. Flush
b. Cyanosis
c. Jaundice
d. Pallor

What the drug of tuberculosis who make orange-like urine color ?


A. Isoniazid
B. Rifambin
C. Streptomycin
D. Perazenamide

The nurse teaches a client with COPD to assess for s/s of right-sided heart failure. Which
of the following s/s would be included in the teaching plan?
1. Clubbing of nail beds
2. Hypertension
3. Peripheral edema
4. Increased appetite

The hormone responsible for a positive pregnancy test is:


A. Estrogen
B. Progesterone
C. Human Chorionic Gonadotropin
D. Follicle Stimulating hormone

Which of the following would be the priority nursing diagnosis for a client with an ectopic
pregnancy?
A. Risk for infection
B. Pain
C. Knowledge Deficit
D. Anticipatory Grieving

B. For the client with an ectopic pregnancy, lower abdominal pain, usually unilateral, is the
primary symptom. Thus, pain is the priority. Although the potential for infection is always
present, the risk is low in ectopic pregnancy because pathogenic microorganisms have not

41
been introduced from external sources. The client may have a limited knowledge of the
pathology and treatment of the condition and will most likely experience grieving, but this is
not the priority at this time.

The drug used for gout disease is :


A. rifambin
B. Allopurinol
C. Perazenamide
D. Streptomycin

In all of the following may giving anticholenergic drug like artane except:
A. Parkinsonism
B. Urge incontinence
C. Distonia
D. Tradive dyskinesia

Extraperamidal Symptoms (EPS) is a group of symptoms that may appear after using of tipical
antipsychotic drugs .. EPS include:
.1Dystonia .. tenderness of facial muscle .. with lifting hands and turning face aside .. appear mostly
in young man who use drugs once (rapid appear ) treated by anticholenergic drugs including artane
..kemedrin .. and dekinate.

B. Akathesia .. internal restlessnes with psychomotor restlessnes .. patient feels pressure in


mediastenum .. treated by anticholenergic drugs .. and beta blockers..

C. Parkinsonism .. limbs tremor with mask face .. treated by decreasing dosage .. anticholenergic
drugs .. antiparkinsonism drug..

D. Tradive doskonesia .. mastication and sucking movement with hypersalivation .. appear after 6
months of using drugs .. diagnosed by salive on the patient billow at the morning .. treated by
stopping medication .. forbidden using of anticholinergics because these drugs worsen the condition

A male client undergoes total gastrectomy. Several hours after surgery, the nurse notes
that the client’s nasogastric (NG) tube has stopped draining. How should the nurse
respond?
A. Notify the physician
B. Reposition the tube
C. Irrigate the tube
D. Increase the suction level

Joshua is receiving furosemide and Digoxin, which laboratory data would be the most
important to assess in planning the care for the client?
A. Sodium level
B. Magnesium level
C. Potassium level
D. Calcium level

42
The rout of medication administration that lead to 100% given drug bioavailability is :
1. PO
2.IV
3.IM
4.SL

The decreasing in a drug bioavailability that


given PO in comparing with drug that given IV due to :
1. The First Pass effect
2. The chemestry of drugs
3. The long tract passage of gastric system
4. The structure of drugs

pH is the log of ______?


A. H ions
B. Cl ions
C. C ions
D. Fl ions

Following a heart transplant, a client is started on medication to prevent organ rejection.


Which category of medication prevents the formation of antibodies against the new organ?
A. Antivirals
B. Antibiotics
C. Immunosuppressants
D. Analgesics

To facilitate of oral secretion in a child who had cleft repair, the nurse should place the
child in what position?
A. Supine.
B. Side-lying
C. Trendeleburg
D. High-fowler

After repair of a cleft palate, the child should be placed in a side-lying position. This
position helps promote drainage and maintain an open airway. so the answer is B

Q. The lung function tests are determined by


1. tonometer
2. spirometer
3. sphygmomanometer
4. barometer

A 68-year-old client with a history of mild CHF and glaucoma isreceiving IV mannitol
(Osmitrol) to decrease intraocular pressure. The nurse would monitor the client for signs
and symptoms of:...
A.Fluid volume excess...
B. Fluid volume deficit...

43
C. Hyperkalemia...
D. Hypernatremia

The true values of secondary hypothyrodism is :


1. Decreased FSH .. Decreast T3 .. Decrease T4
2. Decrease FSH .. Increase T3 .. Increase T4
3. Increase FSH .. Decrease T3 .. Decrease T4
4. Normal FSH .. Decrease T3 .. Decrease T4
In primary hypothyrodism .. the problem in the thyroid gland itself .. no secretion from the gland as
decreased in T3 and T4 .. lead to an increase of secretion of TSH in order to stimulate thyroid gland
to secrete its hormones ..

In secondary hypothyrodism .. the problem in the pituitary gland that has a deficiency in secretion
of TSH .. lead to a defeciency in secretion of T3 and T4 ..

In tertiary hypothyrodism .. the problem in the hypothalamus .. there is a defeciency in TRH


(thyroid releasing hormones) .. lead to a defeciency in TSH .. lead to a defeciency in T3 and T4
production..

Primary hypothyrodism : increase TRH .. increase TSH .. decrease T3 and T4

Secondary hypothyrodism : Increase TRH .. decrease TSH .. decrease T3 and T4

Tertiary hypothyrodism : Decrease TRH .. decrease TSH .. decrease T3 and T4


You are providing care for a patient who underwent thyroidectomy 2 days ago. Which
laboratory value requires close monitoring?
A.Calcium
B. Sodium
C. Potassium
D. White blood cells

The nurse admitting a preterm newborn to the nursery should assess the newborn for
which of the following?
1. Respiratory distress
2. Shoulder dystocia
3. Clavicle fracture
4. Palsies

Contraindication for streptokinase treatment is :


1. Symptoms of MI that last 5 hours
2. Patient has a streptococus pneumonia infection 3 months ago
3. Patient with lateral wall MI
4. Patient with carotid plaque
streptokinaze treatment taken from streptococcus bacteria .. contraindication if the patient has a
streptococcus infection 6 months ago

44
year old patient is inquiring about the methods or ways to detect cancer earlier. The nurse
least likely identify this method by stating:
A. Annual chest x-ray.
B. Annual Pap smear for sexually active women only.
C. Annual digital rectal examination for persons over age 40.
D. Yearly physical and blood examination

The nurse is making clinical rounds on a group of clients in a newborn nursery. Which
infant is at greatest risk of developing respiratory distress syndrome (RDS)?
a. A neonate born at 36 weeks’ gestation.
b. A neonate born by Cesarean section.
c. A neonate experiencing apneic episodes.
d. A neonate who is 42 weeks’ gestation

Q. Normal respiration of healthy person is:


1. 5-10/min
2. 10-15/min
3. 15-20/min
4. 20-30/min

Which of the following systems is not blocked by spinal anesthesia?


a. The sympathetic nervous system.
b. The sensory system.
c. The parasympathetic nervous system.
d. The motor system.

Which of the following blood components is decreased in anemia?


A. Erythrocytes
B. Granulocytes
C. Leukocytes
D. Platelets

Xerostomia means__
A) Inflamation of the stomach
B) Dry mouth
C) Hair loss
D) Difficulty in swallowing

Contraindication for endarterectomy surgery ?


A. Patient with strong heart failure
B. Patient with CVA
C. Patient with Complete blockage of carotid artery
D. Patient with class II obesity
If the patient has 100% (complete) obstruction .. they said that the patient use collateral circulation
to supply his brain and no need for endarterectomy!!

45
Which change in vital signs would you instruct a nursing assistant to report immediately for
a patient with hyperthyroidism?
A.Increased and rapid heart rate
B. Decrease systolic blood pressure
C. Increased respiratory rate
D. Decreased oral temperature

.Average human liver weighs:..


A. 6. 0 kg...
B. 5.0 kg....
C. 0.5 kg....
D. 1.5 kg

Which of the following assessments should be a priority immediately after nasal surgery?
a. Assessing the client’s pain.
b. Inspecting for periorbital ecchymosis.
c. Assessing respiratory status.
d. Measuring intake and output

The nurse is caring for a 2-day-old neonate in the recovery room 30 minutes after surgical
correction for the cardiac defect, transposition of the great vessels. Which of the following
would alert the nurse to notify the physician?
1. Oxygen saturation of 90%.
2. Pale pink extremities.
3. Warm, dry skin.
4. Femoral pulse of 90 bpm.

The patient complained of pain and difficulty in swallowing. This term is referred as:
A. Odynophagia
B. Dysphagia
C. Pyrosis
D. Dyspepsia
When difficulty of swallowing is accompanied with pain this is now referred as odynophagia.
Dysphagia is difficulty of swallowing alone.
During the period of induction of labor, a client should be observed carefully for signs of:
A. Severe pain
B. Uterine tetany
C. Hypoglycemia
D. Umbilical cord prolapse
B. Uterine tetany. Uterine tetany could result from the use of oxytocin to induce labor. Because
oxytocin promotes powerful uterine contractions, uterine tetany may occur. The oxytocin infusion
must be
stopped to prevent uterine rupture and fetal compromise.

A pregnant client states that she “waddles” when she walks. The nurse’s explanation is
based on which of the following as the cause?

46
A. The large size of the newborn
B. Pressure on the pelvic muscles
C. Relaxation of the pelvic joints
D. Excessive weight gain
C .During pregnancy, hormonal changes cause relaxation of the pelvic joints, resulting in the typical
“waddling” gait. Changes in posture are related to the growing fetus. Pressure on the surrounding
muscles causing discomfort is due to the growing uterus. Weight gain has no effect on gait

The patient was admitted in the Medical Floor due to pyrosis, dyspepsia and difficulty of
swallowing. Based from the symptoms presented, the Nurse might suspect:
A. Esophagitis
B. Hiatal hernia
C. GERD "Gastroesophageal Reflux Disease "
D. Gastric Ulcer
GERD "Gastroesophageal Reflux Diseaseis the backflow of gastric or duodenal contents into the
esophagus caused by incompetent lower esophageal sphincter. Pyrosis or heartburn, dyspepsia and
dysphagia are cardinal symptoms.

A client with cirrhosis is at risk for developing complications. Which condition is the most
serious and potentially life-threatening?
A. Esophageal varices
B. Ascites
C. Peripheral edema
D. Asterixis (liver flap)

When a client has esophageal varices, the vessels become very fragile and massive hemorrhage can
occur. The mortality rate is 30% – 50% after an episode of bleeding. Ascites and edema occur when
liver production of albumin fails. Asterixis is a sign of hepatic encephalopathy .Focus:Prioritization

The most difficult time to control diabetes during maternity period is;-
A. First trimester
B. Second trimester
C. Labour & delivery
D. Last trimester

Digesive juice contains catalyticagents called...


:
A. Vitamins...
B. Hormones...
C.Enzymes....
D. Nitrates

pH 7.6, PaCO2 53, HCO3- 38


A. Metabolic Alkalosis, Partially Compensated
B. Metabolic Alkalosis, Fully Compensated
C. Respiratory Acidosis, Partially Compensated

47
D. Respiratory Alkalosis, Fully Compensated

A female client is admitted with a diagnosis of delusions of GRANDEUR. This diagnosis


reflects a belief that one is:
A. Being Killed
B. Highly famous and important
C. Responsible for evil world
D. Connected to client unrelated to oneself

A 90-year-old client’s condition is one of lethargy, poor capillary perfusion, and urinary
incontinence. These findings should indicate to a nurse that this client is at greatest risk
for:
1. Aspiration.
2. Contractures.
3. Dehydration.
4. Skin breakdown.

Answer 1 is incorrect because these conditions do not cause aspiration, for which risk factors would
come from secretions, food, or fluid.

Answer 2 is incorrect because these conditions do not cause contractures .

Answer 3 is incorrect because these conditions do not cause dehydration .

Answer 4 is correct because all of the conditions are risk factors for skin breakdown—inactivity,
poor circulation, and skin wet from urine

Generalized anxiety disorder is characterized by...


A. Free-floating anxiety....
B. Acute exacerbations...
C. Restlessness...
D. Palpitation and dizziness...
E.All of the above

The client is receiving heparin for thrombophlebitis of the left lower extremity. Which of the
following drugs reverses the effects of heparin?
❍ A. Cyanocobalamine
❍ B. Protamine sulfate
❍ C. Streptokinase
❍ D. Sodium warfarin

A client in her third trimester tells the nurse, “I’m constipated all the time!” Which of the
following should the nurse recommend?
A. Daily enemas
B. Laxatives
C. Increased fiber intake
D. Decreased fluid intake

Untreated hyperthyroidism during pregnancy may result in all of the following except:
A. Premature birth and miscarriage
B. Low birthweight

48
C. Autism
D. Preeclampsia

Claustrophobia means fear of....


A. Open spaces....
B. Closed spaces.....
C. Height....
D. Lizards

Patients with Type 1 diabetes mellitus may require which of the following changes to their
daily routine during periods of infection?
A. No changes.
B. Less insulin.
C. More insulin.
D. Oral diabetic agents.

A client is experiencing hyperventilation, depersonalization, and palpitations. Which


nursing diagnosis takes priority?
1. Social isolation.
2.Ineffective breathing pattern.
3. Risk for suicide.
4. Fatigue.

Peptic ulcer disease particularly gastric ulcer is thought to be cause by which of the
following microorgamisms?
A. E. coli
B. H. pylori
C. S. aureus
D. K. pnuemoniae

While assessing a child with pyloric stenosis, the nurse is likely to note which of the
following?
A. Regurgitation
B. Steatorrhea
C. Projectile vomiting
D. Currant jelly” stools

an appropriate nursing diagnosis of a client with a major depression is


a. alteration in activity
b. alteration in perceptions
c. altetation in affect
d. alteration in social activity

Which of the following nursing assessment is the most important in the patient with
hyperthyroidism and risk for thyrotoxic crisis or thyroid storm?
A. Intake and output
B. Heart sounds
C. Bowel sounds

49
D. Vital signs

Which drug is used to stop bleeding associated with heparin overdose?..


A. urokinase (Abbokinase)....
B. aminocaproic acid (Amicar)...
C. vitamin K (AquaMEPHYTON)...
D. protamine sulfate (Protamine

A patient is admitted to the hospital with a diagnosis of acute hepatitis B. The nurse would
expect a rise in which of the following serum labs?
SELECT ALL THAT APPLY:
A.ALT
B.AST
C.Bilirubin
D. Creatinine
E. Neutrophils
F. Platelets

For lipid-lowering agents to be successful, drug therapy must lower:...


A. HDL...
B. LDL..
C. Total fat...
D. All of the above

Which of the following is a potential side effect of IV furosemide (Lasix)?...


A. Drowsiness...
B. Diarrhea...
C. Cystitis....
D.Hearing loss

The nurse is planning room assignments for the day. Which client should be assigned to a
private room if only one is available?
❍ A. The client with Cushing’s disease
❍ B. The client with diabetes
❍ C. The client with acromegaly
❍ D. The client with myxedema
Patient with Cushing syndrome is more susceptible for infection because over secretion of
corticosteroids of adrenal gland

Q. The function of fallopian tube is:


1.Secretion of oestrogen and progesterone
2.Passage way for foetus
3.Occurrence of fertilization
4.Developing foetus

Which of the following would not be considered an acute effect of diabetes mellitus?
A. Polyuria
B. Weight gain
C. Polydipsia

50
D. Polyphagia

The major goal of therapy for a client with heart failure and pulmonary edema should be to:
1. Increase cardiac output.
2. Improve respiratory status.
3. Decrease peripheral edema.
4. Enhance comfort.

Which is the correct procedure for collecting a sputum specimen for culture and sensitivity
testing?
a. Have the patient place the specimen in a container and enclose the container in a
plastic bag
b. Have the patient expectorate the sputum while the nurse holds the container
c Have the patient expectorate the sputum into a sterile container
d. Offer the patient an antiseptic mouthwash just before he expectorate the sputum

CSF analysis is diagnostic procedure for meningitis. Which of the following results
confirms viral infection?
a. clear appearance
b. decrease glucose level
c. cloudy appearance
d. increase protein
Viral or aseptic meningitis has a clear appearance while bacterial has a cloudy or turbid
appearance.. Bacterial CSF results inc wbc, inc protein, low glucose

As a competent nurse, you are aware that vasodilators are used mainly to treat:...
A. Diabetes....
B. Hypertension....
C. Atrial fibrillation...
D. Hypotension

A female patient with a terminal illness is in denial. Indicators of denial include:


a.Shock dismay
b. Numbness
c. Stoicism
d. Preparatory grief

Which of the following may be seen in dissociative disorders?.....


A. Hallucinations....
B. Delusions....
C.Amnesia...
D. Phobias

Parkinson disease are the type of


A. Metabolic disorder
B. Mortor disorder
C. Bone disorder

Which of the following amounts of blood loss following birth marks the criterion for
describing postpartum hemorrhage?
A. More than 200 ml

51
B. More than 300 ml
C. More than 400 ml
D. More than 500 ml

As a nurse .. when the most appropriate time for glucophage taking ?


A. With or after the meal
B. Half hour before the meal
C. Half hour after the meal
D. 10 minute before the meal

the least severe type of paranoid disorder is


a. paranoid schizophrenia
b.paranoid personality
c. paranoia
d. paranoid state

a client is diagnosed with catatonic schizophrenia which is a highest priority nursing


diagnosis?
a.noncompliance
b. impaired communication
c. ineffective coping
d.selfcare deficit

The definition of inflammation is the:


A. Specific response to cellular injury
B. Nonspecific cellular response to tissue injury
C. Complex progression of tissue changes in response to injury
D. Invasion and multiplication of pathogenic microorganisms in body tissue

Which of following is an anticoagulant ?


A. Streptokinase
B. Alteplase
C. Heparin
D. Reteplase

Which of these gases are important in the human body?


A. Nitrogen and hydrogen
B. Hydrogen and carbon dioxide
C. Oxygen and carbon dioxide
D. Nitrogen and oxygen

The client is complaining of coolness swelling and pain at the IV site. What potential
complication does the client have?
A. Phlebitis
B. Infiltration
C. Thrombosis
D. Infection

What is cause of difficulty in breathing in asthmatic patient

52
a) spasm of bronchi
b) lesser amount of oxygen in air
c) environmental problem
d) all of the above

The supplements that should taking while taking glucophage treatment is:
A. Caltral
B. Tribimen
C. Folic Acid
D. Ferrous Iron
While taking glucophage .. should taking tribimen at the same time..
Tribimen is a combination of vitamins 1 + 6 + 12
The nurse is caring for a client experiencing metabolic alkalosis. Which of these conditions
may cause this?
A. Overly vigorous mechanical ventilation
B. Hyperkalemia
C. Prolonged nasogastric suctioning
D. Pain

Which of these postoperative complications is MORE likely to occur in older adults clients?
A. Deep vein thrombosis
B. Dehydration
C. Pneumonia
D. Unpredictable responses to medications and an esthetics

The sleep induced drug who not lead to addiction is:


A. Lorazepam
B. Diazepam
C. Nocturno
D. Bondormin

larazepam and diazebam not sleep induced .. its an anti anxiety and lead to addiction .. bondormin
is sleep induced and lead to addiction .. nocturno is sleep induced but not lead to addiction!

A client is being treated for injuries sustained in an automobile accident. The client has a
central venous pressure (CVP) line in place. The nurse recognizes that CVP measurement
reflects which of the following?
a. Cardiac output
b. Pressure in the left ventricle
c. Pressure in the right atrium
d. Pressure in the pulmonary artery

You are caring for a client with peptic ulcer disease. Which assessment finding is the most
serious?
A. Projectile vomiting
B. Burning sensation 2 hours after eating
C. Coffee-grounded emesis
D.Board-like abdomen with shoulder pain

53
ANSWER D – A board-like abdomen with shoulder pain is a symptom of a perforation, which is
most lethal complication of peptic ulcer disease. A burning sensation is a typical complaint, which
can be controlled with medications. Projectile vomiting can signal an obstruction .Coffee-ground
emesis is typical of slower bleeding and will require diagnostic testing. Focus: Prioritization

SubCutenous heparin should be administered in the:...


A. Flank...
B. Abdominal fat...
C. Leg...
D. Gluteal area

The elbow is _____ to the wrist:..


a. distal..
b. lateral..
c. ventral..
d. proximal

Clients with diabetes mellitus require frequent vision assessment. The nurse should
instruct the client about which of the following eye problems most likely to be associated
with diabetes mellitus?
a. Cataracts
b. Retinopathy
c. Astigmatism
d. Glaucoma

Anti dote of pethedine.


A. Fentanyl..
B. Narcan.
C. Trilene.
D. Isofluran.

The best method of evaluating the amount of peripheral edema is:


[a .Weighing the client daily
b.Measuring the extremity
c.Measuring the intake and output
d.Checking for pitting

A nurse is performing continuous bladder irrigation at 1 L/h. Which assessment is the


priority?
A. The amount of fluid being returned.
B. The size of the indwelling catheter.
C. The client’s knowledge level of the procedure.
D. The percentage of formalin ordered for irrigation.

Patent who recieve captopril .. start to cough 3 days ago .. can change treatment from
captopril to :
A. Verapamil
B. Nefedipine
C. Losartan
D. Propanolol

54
The true answer is C

In case of hypotension .. the renin angiotensin system (RAS) will work as follow:

.1Decrease hypotension lead to

.2Decrease renal perfussion lead to

.3Kidneys to feel like as shock lead to

.4Kidney secrete renin who lead to

.5Two function .. firstly stimulate secretion of aldesteron from adrenal gland and stimulate
secretion of angiotensin 1 .. lead to:

.6Aldesteron increase reabsorbtion of sodium from kidneys to blood .. and angiotensin 1 Converted
to angiotensin 2 by angiotensin converting enzyme (ACE) in the lung, thus:

.7Increasing sodium will lead to water also going to blood (where is the concentration of salt
increase) and release of angiotensin 2 construct blood vessels who lead to

.8Increase blood volume by aldesteron .. and vasoconstriction by angiotensin 2 lead to

.9Increase blood pressure again

ACEI (all medication who end by PRIL) .. inhibit the function of converting angiotensin 1 to
angiotensin 2 .. so .. no angiotensin 2 .. no vasoconstriction .. no increase in blood pressure..

ACEI prevent from number 6 to occur .. so no completion to number 9

One of the drug related to ACEI is CAPTOPRIL .. the side effect of ACEI come from
CAPTOPRIL:

C .. Cough
A .. Angioedema / Agranulocytosis
P .. Potassium Elevation
T .. Teratogenic / Tachycardia
O .. Orthostatoc Hypotension
P .. Pancreatitis
R .. Renal failure
I .. Increase Bradykinaze
L .. Liver Failure

If the patient has a cough during using ACEI .. we change the ACEI to medication who act on the
same system (renin angiotensin system .. RAS) ..

Answe A .. virapamil .. calcium channel blocker act on the AV node .. not correct

Answer B .. nefidipine is a calcium channel blocker .. act on blood vessels .. not correct

Answer D .. propanolol .. beta blocker .. act on the sympathetic system .. not correct

55
Answe C .. losartan .. all the medication ended by (sartan) ia ARB ( Angiotensin receptor blockers)
.. the ACE (ENZYME) available normaly but no receptor available due that the drug ocuupied the
receptor site of ACEI .. so no production of Angiotensin 2 .. no vasoconstriction .. no increase
blood pressure

Q. The nurse understands that electroconvulsive therapy is primary used in psychiatric


care for the treatment of:
A. Anxiety disorders.
B. Depression
C. Mania
D. Schizophrenia
D .. is a treatment of choice for catatonic schizophrenia
The drug of choice for patint complaining od hypertension is:
A. Hydrochlorothiazide
B. ACEI
C. Beta Blockers
D. Calcium Channel Blockers

The drug of choice for:


)1hypertensive patient .. is Hydrochlorothiazide (soft diuretics )

)2hypertensive patient with diabetes mellitus .. is ACEI

)3hypertensive patient with MI .. is beta blockers and ACEI

)4hypertensive patient with heart failure .. is ACEI and Diuretics

)5hypertensive patient with benign prostatic hyperplasia .. is alpha blockers

The nurse is caring for a client with hyponatremia. When reviewing the client's health
history the nurse can identify the condition is the result of:
A. Eating foods high in calcium
B. Namely suctioning
C. Prolonged vomiting or diarrhea
D. Dehydration

The client has experienced pulmonary embolism. The nurse assesses for which of the
following symptoms most commonly reported?
a. Dyspnea noted when deep breaths are taken
b. Hot, flushed feeling
c. Chest pain that occurs suddenly
d. Sudden chills and fevers

Shenaya will be having her exam in pharmacology tomorrow. She should be aware that
antitussive is indicated to:
A. encourage removal of secretions through coughing.
B. relieve rhinitis.
C. control a productive cough.
D. relieve a dry cough.

56
Q. The nurse correctly teaches a client taking the benzodiazepine to avoid excessive
intake of:
A. Cheese
B. Coffee
C. Cheese
D. Shellfish

A male patient is admitted to the health care facility for treatment of chronic obstructive
pulmonary disease. Which nursing diagnosis is most important for this patient?
A. Activity intolerance related to fatigue
B. Anxiety related to actual threat to health status
C. Risk for infection related to retained secretions
D. Impaired gas exchange related to airflow obstruction

A patient admitted to the hospital with myocardial infarction develops severe pulmonary
edema. Which of the following symptoms should the nurse expect the patient to exhibit?
A. Slow, deep respirations.
B. Stridor.
C. Bradycardia.
D. Air hunger.
Patients with pulmonary edema experience air hunger, anxiety, and agitation .Respiration is fast and
shallow and heart rate increases. Stridor is noisy breathing caused by laryngeal swelling or spasm
and is not associated with pulmonary edema.
A client admitted with angina compains of severe chest pain and suddenly becomes
unresponsive. After establishing unresponsiveness, which of the following actions should
the nurse take first?
A. Activate the resuscitation team
B. Open the client’s airway
C. Check for breathing
D. Check for signs of circulation
The nurse is preparing to administer a unit of PRBCs to an anemic client. After obtaining
the blood from the blood bank, the nurse must begin administering it within which of the
following time periods?
a. 15 minutes
b.30 minutes
c. 45 minutes
d. 60 minutes

)1(The nurse has up to 30 minutes to begin administering the blood product. (2) CORRECT: After
obtaining the blood product from the blood bank, the nurse must begin administering the product
within 30 minutes )3( .This time period is too long. (4) This time period is too long.
While assessing a child with pyloric stenosis, the nurse is likely to note which of the
following?
A. Regurgitation
B. Steatorrhea
C. Projectile vomiting
D. Currant jelly” stools

Which of the following factors would the nurse suspect as predisposing a client to
placenta previa?
A. Multiple gestation

57
B. Uterine anomalies
C. Abdominal trauma
D. Renal or vascular disease

A .Multiple gestation is one of the predisposing factors that may cause placenta previa. Uterine
anomalies abdominal trauma, and renal or vascular disease may predispose a client to abruptio
placentae.

The charasteristic of deudinal ulcer in comparing with peptic ulcer that the:
A. The pain in dudinal ulcer appear 0.5 to 1 hour after meal while 2-3 hour after meal in
peptic ulcer
B. Eating in dudinal ulcer decreasing pain while the eating in peptic ulcer increasing pain
C. Weight of patient decrease in dudinal ulcer while increase in peptic ulcer
D. Perforation risk decrease in dudinal ulcer while increase in peptic ulcer

Which of the following would be a priority nursing diagnosis for the client with heart failure
and pulmonary edema?
A. Risk for infection related to stasis of alveolar secretions
B. Impaired skin integrity related to pressure
C. Activity intolerance related to pump failure
D. Constipation related to immobility

Aspirin is administered to the client experiencing an MI because of its:


A. Antipyrectic action
B. Antithrombotic action
C. Antiplatelet action
D. Analgesic action

B .Antithrombotic action. Aspirin does have antipyretic, antiplatelet, and analgesic actions, but the
primary reason ASA is administered to the client experiencing an MI is its antithrombotic action.

Samer .. is a chronic asthmatic patient .. as a nurse .. what you expect to find in taking
ABG sample ?
A. Mixed Acidosis
B. Respiratory Alkalosis
C. Mixed Alkalosis
D. Respiratory Acidosis

After spleenectomy .. the expected hematological finding is:


A. Thrombocytosis
B. Leukocytosis
C. Erythrocytosis
D. Pancytopenia

The spleen contain approximately one unit of blood

In spleenectomy .. the body lost the spleen and lost this unit of blood .. it feel bleeding like situation
.. and increase number of platelet to stop this false bleeding as compansatory mechanism ..
(thrombocytosis)

58
Which cranial nerve is responsible for eye movement?....
1-Occulomotor...
2-Vagus...
3-Trigeminal...
4-Olfactory

A nurse prepares to administer a vitamin K injection to a newborn infant. The mother asks
the nurse why her newborn infant needs the injection. The best response by the nurse
would be:
A. “You infant needs vitamin K to develop immunity.”
B. “The vitamin K will protect your infant from being jaundiced.”
C. “Newborn infants are deficient in vitamin K, and this injection prevents your infant from
abnormal bleeding.”
D. “Newborn infants have sterile bowels, and vitamin K promotes the growth of bacteria in
the bowel.”

C .. vitamin k vital in thrombotic activity to prevent bleeding and synthezied by liver .. the infant
has immature liver and there is no enogh production of vitamin K .. so we should give this vitamin
to prevent bleeding

Osteoporosis is:
1..Bone infection
2..Bone cancer
3..Bone disease that can lead to increased risk of fractures
4..Bony outgrowths

A client with thyroid cancer undergoes a thyroidectomy. After surgery, the client develops
peripheral numbness, tingling, and muscle twitching. Which type of medication should the
nurse be prepared to administer?
a. Thyroid supplement
b. Antispasmodic
c. Barbiturate
d. Calcium replacement
Which of the following classes of medications maximizes cardiac performance in clients
with heat failure by increasing ventricular contractility?
A. Beta-adrenergic blockers
B. Calcium channel blockers
C. Diuretics
D. Inotropic agents

D ..

Positive inotropes increase cardiac contractility

Negative inotropes decrease cardiac contractility

Positive chronotropes increase heart rate

Negative chronotropes decrease cardiac contractility

Positive dromotropes increase the electrical movement from atriums to ventricles

59
Negative dromotropes decrease the electrical movement from atriums to ventricles

For example::::

Beta blocker decrease heart rate and decrease muscle contractility .. so its negative chronotropes
and negative inotropes

Digoxin .. decrease heart rate and increase muscle contractility .. so its negative chronotropes and
positive inotropes

What position should the nurse place the head of the bed in to obtain the most accurate
reading of jugular vein distention?
A. High-fowler’s
B. Raised 10 degrees
C. Raised 30 degrees
D. Supine position

The visceral pleura:.


.a. is the membrane lining surface of the lungs
b. is the membrane lining the wall of the thoracic cavity..
c. is the fluid around the lungs..
d. is the thinnest portion of the peritoneum

What is the term used to describe an enlargement of the heart muscle?


A. Cardiomegaly
B. Cardiomyopathy
C. Myocarditis
D. Pericarditis

Rasha .. is an asthmatic patient who diagnosed since one month .. as a nurse .. what you
expect to find in taking ABG sample?
A. Mixed Acidosid
B. Respiratory Alkalosis
C. Mixed Alkalosis
D. Respiratory Acidosis

In acute asthma .. respiratory alkalosis

In chronic asthma .. respiratory acidosis

In acute asthma .. compensation presence .. more ventilation .. mor exit of CO2 .. lead to respiratory
alkalosis

In chronic asthma .. no compensation .. the CO2 accumulate in Respiratory system .. lead to


respiratory acidosis

What is the function of serous membranes:..


a. to prevent fluid loss from an organ..

60
b. to reduce friction between internal organs..
c. to circulate blood around the organ..
d. to conserve heat within the organ

Histology is the study of:..


a. cells and membranes..
b. skin..
c. organs and organ systems..
d. tissues

The nurse is teaching a client who is scheduled for a paracentesis. Which of the following
statements by the client to the nurse indicates that teaching has been successful?
1. “I will be in surgery for less than an hour.”
2. “I must not void prior to the procedure.”
3. “The physician will remove 2 to 3 liters of fluid.”
4. “I will lie on my back and breathe slowly.”

Which of the following sign’s may indicate pregnancy?


A. Chadwick’s sign
B. Turner’s sign
C. Virchow’s sign
D. Kock’s sign

Chedwick sign .. early sign appear after 5 to 9 week of pregnancy .. increase blood follow to the
labias .. vagina .. and servix .. resulting in blue discolor of them

Which of the following tests is used most often to diagnose angina?


A. Chest x-ray
B. Echocardiogram
C. Cardiac catherization
D. 12-lead electrocardiogram (ECG)

Chemotherapy is one of the therapeutic modalities for cancer. This treatment is


contraindicated to which of the following conditions?
A. Recent surgery
B. Pregnancy
C. Bone marrow depression
D. All of the above

Warfarin is antagonize ( antidote ) by ?


.
A. Heparin
B. Vit K
C. Protamine
D. Warfarin -Excess

A client is admitted to the hospital and has a diagnosis of early stage chronic renal failure.
Which of the following would the nurse expect to note on assessment of the client?
A. Polyuria

61
B. Polydipsia
C. Oliguria
D. Anuria

A .Polyuria occurs early in chronic renal failure and if untreated can cause severe dehydration.
Polyuria progresses to anuria, and the client loses all normal functions of the kidney. Oliguria and
anuria are not early signs, and polydipsia is unrelated to chronic renal failure.

The sudden onset of which of the following indicates a potentially serious complication for
the client receiving an I.V. infusion?
1. Noisy respirations.
2. Pupillary constriction.
3. Halitosis.
4. Moist skin

The nurse administers an intradermal injection to a client. Proper technique has been used
if the injection site demonstrates which of the following?
1. Minimal leaking.
2. No swelling.
3. Tissue pallor.
4. Evidence of a bleb.
d )Evidence of a bleb or wheal. Reason: A properly administered intradermal injection shows
evidence of a bleb or wheal at the injection site. There should be no leaking of medication from the
bleb; it needs to be absorbed into the tissue. Lack of swelling at the injection site means that the
injection was given too deeply. The presence of tissue pallor does not indicate that the injection was
given correctly.
The nurse must verify the client’s identity before administration of medication. Which of the
following is the safest way to identify the client?
A. Ask the client his name
B. Check the client’s identification band
C. State the client’s name aloud and have the client repeat it
D. Check the room number
Occurrence of which symptom best differentiates between mild and severe concussion?
1. Loss of consciousness
2. Tremor
3. Unstable gait
4. Impaired speech

When assessing a client with a pleural effusion, which would be an expected finding?
a. Expiratory wheeze
b. Productive cough
c. Decreased breath sounds
d. Stridor

In Maslow’s hierarchy of physiologic needs, the human need of greatest priority is:
A. Love
B. Elimination
C. Nutrition
D. Oxygen

62
Which of the following medications should the nurse anticipate administering in the event
of a heparin overdose?
1. Warfarin sodium (Coumadin).
2. Protamine sulfate.
3. Acetylsalicylic acid (ASA).
4. Atropine sulfate.

Thoracentesis is useful in treating which of the following pulmonary disorders except:


A. Hemothorax
B. Tuberculosis
C. Hydrothorax
D. Empyema

Which of the following suspiciously indicate lymphoma ?


A. fever .. sweating .. weight loss .. fatigue
B. Fever .. sweating .. weight gain .. fatigue
C. Hypothermia .. sweating .. weight loss .. fatigue
D. Hypothermia .. sweating .. weight gain .. fatigue

Neoadjuvant oncology treatment goal is:


A. To prevent tumor reoccurance
B. To relief on patient pain
C. To decrease size of tumor
D. To protect other organs from spreading of tumor

Specific antidote of Organophosphorus poisoning is ?


A. Atropine
B. Pralidoxime
C. Acetylstiene
D. Vit K

Oncological patient who has nausea as a side effect of chemotherapy .. what is the most
approbriate antiemetic drug will be used to relieve nausea :
A. Pramin
B. Ranitidine
C. Zofran
D. Omeprazole

Amoxil (amoxicillin) suspension 180 mg PO bid is ordered for a patient who cannot
swallow pills. It is supplied as 125 mg/5 mL. How many milliliters should you administer?
A. 1.4 mL
B. 3.5 mL
C. 5 mL
D. 7.2 mL

The pathological process causing esophageal varices is:


a. ascites and edema.
b. systemic hypertension.

63
c. portal hypertension.
d. dilated veins and varicesitis.

Your client is receiving a continuous gastric tube feeding. The rate of the tube feeding is
75 mL per hour. You aspirate 45 mL of gastric contents. What should you do?
A. Return the measured residual and discontinued the feeding
B. Discard the residual and discontinue the tube feeding
C. Continue the feeding and discard all of the residual
D. Return the measured residual and continue the feeding

A normal, healthy infant is brought to the clinic for the first immunization against polio. The
nurse should administer this vaccine by what route?
1. Oral route.
2. I.M. route.
3. Subcutaneous route.
4. Intradermal route.

A client with chronic renal failure has asked to be evaluated for a home continuous
ambulatory peritoneal dialysis (CAPD) program. The nurse should explain that the major
advantage of this approach is that it:
A. Is relatively low in cost
B.Allows the client to be more independent
C. Is faster and more efficient than standard peritoneal dialysis
D. Has fewer potential complications than standard peritoneal dialysis

B .The major benefit of CAPD is that it frees the client from daily dependence on dialysis centers,
home health care personnel, and machines for life-sustaining treatment. The independence is a
valuable outcome for some people. CAPD is costly and must be done daily. Side effects and
complications are similar to those of standard peritoneal dialysis

The nurse is caring for the mother of a newborn. The nurse recognizes that the mother
needs more teaching regarding cord care because she
a. keeps the cord exposed to the air.
b. washes her hands before sponge bathing her baby.
c. washes the cord and surrounding area well with water at each diaper change.
d. checks it daily for bleeding and drainage.

C.... Exposure to air helps dry the cord. Good hand washing is the prime mechanism for preventing
infection. Washing the surrounding area is fine but wetting the cord keeps it moist and predisposes
it to infection. It is important to check for complications of bleeding and drainage that might occur.

The pituitary hormone that stimulates the secretion of milk from the mammary glands is:
1-Prolactin
2-Oxytocin
3-Estrogen
4-Progesterone

64
Which of the following statement is true regarding the visual changes associated with
cataracts?
A. Both eyes typically cataracts at the same time
B. The loss of vision is experienced as a painless, gradual blurring
C. The patient is suddenly blind
D. The patient is typically experiences a painful, sudden blurring of vision

The physician orders an IV with heparin at 700 units/hour. You have a bag with 100
units/mL of solution. How many milliliters per hour will you set on the infusion controller?
A. 7 mL/hour
B. 8 mL/hour
C. 9 mL/hour
D. 10 mL/hour

At what stage of labor and delivery does a primigravida differ mainly from a multigravida?
A. Stage 1
B. Stage 2
C. Stage 3
D. Stage 4

Answer :(A) Stage 1. In stage 1 during a normal vaginal delivery of a vertex presentation, the
multigravida may have about 8 hours labor while the primigravida may have up to 12 hours labor.

If nurse administers an injection to a patient who refuses that injection, she has committed:
A. Assault and battery
B. Negligence
C. Malpractice
D. None of the above

A .Assault is the unjustifiable attempt or threat to touch or injure another person. Battery is the
unlawful touching of another person or the carrying out of threatened physical harm. Thus, any act
that a nurse performs on the patient against his will is considered assault and battery.

Hemodilution during pregnancy is mayo due to ?


A. Vaginal Blood loss
B. Poor eating
C. Fluid retention
D. Bone marrow suppression

A nurse is assessing the patency of an arteriovenous fistula in the left arm of a client who
is receiving hemodialysis for the treatment of chronic renal failure. Which finding indicates
that the fistula is patent?
A. Absence of bruit on auscultation of the fistula.
B. Palpation of a thrill over the fistula
C. Presence of a radial pulse in the left wrist
D. Capillary refill time less than 3 seconds in the nail beds of the fingers on the left hand.
B .The nurse assesses the patency of the fistula by palpating for the presence of a thrill or
auscultating for a bruit. The presence of a thrill and bruit indicate patency of the fistula. Although
the presence of a radial pulse in the left wrist and capillary refill time less than 3 seconds in the nail
beds of the fingers on the left hand are normal findings, they do not assess fistula patency.

65
To assess the kidney function of a patient with an indwelling urinary (Foley) catheter, the
nurse measures his hourly urine output. She should notify the physician if the urine output
is:
A. Less than 30 ml/hour
B. 64 ml in 2 hours
C. 90 ml in 3 hours
D. 125 ml in 4 hours
A .A urine output of less than 30ml/hour indicates hypovolemia or oliguria, which is related to
kidney function and inadequate fluid intake.

A client is admitted with a diagnosis of a fractured right hip. The health care provider writes
an order for Buck's traction. Which action, if taken by the nurse, is most important?
1. Turn the client every 2 hrs to the unaffected side
2. Maintain the client in a supine position
3. Encourage the client to use a bedside commode
4. Place a footboard on the bed
)1(correct—immobility is a leading cause of problems with Buck's traction; important to turn client
to unaffected side

)2(head of the bed should be elevated 15-20° because the supine position can increase problems
with immobility

)3(client is on strict bedrest

)4(would interfere with the traction


Best drug for ischemic stroke ?
.
A. Aspirin
B. Dipyradimol
C. Warfarin
D. Heparin

In atrial fibrillation, which one of the following waves is absent in ECG?


A) P wave
B) QRS complex
C) T wave

which of the following is a probable sign of pregnancy?


a.amenorrhea
b.breast changes
c.hegar's sign
d.fetal heart beat

Amenorrhea is presumptive- breast changes is Presumptive signs - fetal heart beat is positive sign

A nurse is assigned to care for four clients. When planning client rounds, which client
would the nurse check first?
A. A client on a ventilator
B. A client in skeletal traction

66
C. A postoperative client preparing for discharge
D. A client admitted on the previous shift who has a diagnosis of gastroenteritis

The nurse is assessing a postoperative adult patient. Which of the following should the
nurse document as subjective data?
a. Vital signs
b. Laboratory test result
c. Patient’s description of pain
d. Electrocardiographic (ECG) waveforms

In planning the care for a severely burned client, the nurse should select which of the
following as the priority nursing diagnosis?
1. Pain related to burn injury and treatments
2. Impaired physical mobility related to contractures
3.Risk for deficient fluid volume related to a fluid shift, evaporation, and plasma loss
4. Imbalanced nutrition: less than body requirements related to the body’s need for an
increased calorie intake

The action of medication is inotropic when it:


A. Decreased afterload
B. Increases heart rate
C. Increases the force of contraction
D. Is used to treat CHF

The nurse is preparing to make rounds. Which client should be seen first?:
A. A 1-year-old with hand-and-foot syndrome
B. A 69-year-old with congestive heart failure
C. A 40-year-old with resolving pancreatitis
D. A 56-year-old with Cushing’s disease

Which finding indicates that fluid resuscitation has been successful for a client with a burn
injury?
A. Hematocrit = 60%
B. Heart rate = 130 beats/min
C. Increased peripheral edema
D. Urine output = 50 mL/hr

D .. correct .. 50 ml/h .. there is mean 1200 ml/day .. normal situation .. because urine output should
be minimummmm 40 ml/h .. minimum not average value .. below 40 .. there is a problem .. below
500 ml/day there is oligurea .. below 50 ml/day .. there is an unurea

Answer A incorrect .. hematocrit (HCT) is the percentage of RBC in all blood volume (cells and
fluids(plasma)) .. if RBC increase .. the HCT increase .. if RBC decrease .. the HCT decrease .. if
fluid increase .. the RBC more diluted so the HCT decrease .. if fluids decrease .. the RBC more
concentrated .. so HCT increase .. from these notes .. the HCT increase due to decreasing of fluids

Answer B incorrect .. in case of fluid loss .. blood volume decrease .. blood not adequate reach all
the body tissue .. there is increasing in heart rate as compensationnnnnn

67
Answer C incorrect .. in case of burn .. the K inside the cells exit to the blood .. so lead to
hyperkalmia .. the body exit the K through the kidney .. in the other hand .. according to NA/K
pump .. the Na should exit from the blood .. who exit to the peripheral .. the water or fluid
withdrawn the Na .. so the fluid shifting to the peripheral leading edema .. because this reason .. the
ringer lactate is the most fluid of choice for burn to return the salt in the blood .. to withdrawn the
water

Which of following Anti pyretic drug acts centrally ( hypothallamus )?


.
A. Aspirin
B. Paracetamol
C. Mefenamic acid
D. Heparin

Propranolol is contraindication in :
1. bronchial asthma
2. hypertension
3. hyperthyroidism
4. atrial fibrillation

The most common type of intravenous (IV) solution used during surgery is
a. Lactated Ringer's
b. 0.45% Sodium chloride
c. 5% Dextrose in water
d. 10% Dextrose in saline

What type of milk is present in the breasts 7 to 10 days postpartum ?


A. Colostrum
B. Hind milk
C. Mature milk
D. Transitional milk

the strongest nd longest bone of the body ?


A-occipital
B- femur
C-back bone
D-ulnar

The Ventricular repolarization in ECG iz best seen in ?


A- "P" wave
B- "Q" wave
C- "R" wave
D- "T" wave

P wave .. reflect atrial depolarization

QRS wave .. reflect ventricular depolarization

T wave .. reflect ventricular repolarization

68
The nurse prepare IM injection that is irritating to the subcutaneous tissue. Which of the
following is the best action in order to prevent tracking of the medication
A. Use a small gauge needle
B. Apply ice on the injection site
C. Administer at a 45° angle
D. Use the Z-track technique

Drug of choice for Asthma is ?


.
A. Ipratropium bromide
B. Propanolol
C. Salbutamol
D. Carvidilol

salbutemol (ventolin) .. is beta 2 receptor agonist .. beta 2 receptors found in the airway .. beta 2
agonist is sympatomemic who lead to bronchodilation to relieve the bronchoconstriction who made
by the asthma as disease

Answer A incorrect .. ipratropium promide (aerovent) given also in asthma .. but is not the drug of
choice .. is anticholenergic drug .. given to make atropinization (dryness) .. to drying the mucos
produced by asthmatic disease ..

Answe B and D incorrect .. propanolol .. is a non selective beta blocker .. block beta 1 and beta 2
contraindicated in asthma .. beta blocker blocking b1 lead to decrease heart rate .. and block beta 2
lead to bronchoconstriction .. asthma is a disease who make allergy lead to bronchoconstriction .. so
beta blocker excarbate the asthmatic condition .. carvedilol like propanolol who non selective beta
blocker and selective alpha 1 blocker .. like propanolol mechanism .. who excarbate the disease

The nurse is reviewing laboratory results on a client with acute renal failure. Which one of
the following should be reported IMMEDIATELY?
a. Blood urea nitrogen 50 mg/dl
b. Hemoglobin of 10.3 mg/dl
c. Venous blood pH 7.30
d. Serum potassium 6 mEq/L

ANSWER: D: Serum potassium 6 mEq/L. Although all of these findings are abnormal, the elevated
potassium level is a life threatening finding and must be reported immediately.

the sunshine vitamin is ?


A-vita A
B-vita c
C-vita D
D-vita K

Average weight of new born baby....?


2500-4000

Patient on nasogastric (NG) suction is at risk for_____.Why?


a.Hyponatremia

69
b.Hypernatremia
c.Hypokalemia
d.other

Fluid Electrolyte imbalance is complication of NG suction & hyponatremia is likely to occur being
most common disorder r/t fluid loss

Pernicious aneamia is characterized by deficiency of ?


.
A. Iron deficiency
B. Folate deficiency
C. B12 deficiency
D. Sideroblastic anaemia

Most common type of aneamia during pregnancy is ?


.
A. Iron deficiency
B. Folate deficiency
C. B12 deficiency
D. Sideroblastic anaemia

the largest organ of the body is ?


A-skin
B-liver
C-bone
D-intestine

Normal amount of amniotic fluid....?!


800_1200

Which of the following will the nurse include in the care plan for a client hospitalized with
viral hepatitis?
A. Increase fluid intake to 3000 ml per day
B. Adequate bed rest
C. Bland diet
D. Administer antibiotics as ordered

Which type of incision is done in classical


caesarean section?
A. Vertical in lover uterine segment.
B. Transverse in lover uterine segment.
C. Longitudinal in upper uterine segment.
D. Transverse in upper uterine segment

The nurse suspects that a client with polyuria is experiencing water diuresis. Which
laboratory value suggests water diuresis?
1. High urine specific gravity
2. High urine osmolarity
3. Normal to low urine specific gravity
4. Elevated urine pH

70
Specific gravity is the ratio of the density of a substance to the density of water .. for example .. if
density of substance 2 .. density of water 1 .. the ratio 2:1=2..

If the substance increase .. the density increase .. for example increase to 3 .. so the ratio 3:1=3 .. so
the specific gravity increase

If the water increase .. the density of water increase .. for example 2:2=1 .. so the specific gravity
decrease

Roleeeeeee:

The normal specific gravity 1.010 to 1.025

If the substance increase .. specific gravity increase .. more than 1.025

If the substance decrease .. specific gravity decrease .. less than 1.010

If the water increase .. specific gravity decrease .. less than 1.010

If the water decrease .. specific gravity increase .. more than 1.025

The urine osmolality .. reflect what the concentration of solute in a solution ..

If solutes increase .. the osmolarity increase .. for example .. cup of tea with two teaspoon of sugar
more concentrated and more osmolarity than the same cup of tea with only one teaspoon of sugar..

If fluids increase .. the osmalirity decreased .. for example .. bottle of tea with one teaspoon sugar
less concentrated and less osmolarity than cup of tea with one teaspoon of sugar ..

Roleeeeee:

The normal value of osmolarity 500 mmole to 800 mmole

If the substances or solutes increase .. the osmolarity increase .. more than 800 mmole

If the substances or solutes decrease .. the osmolarity decrease .. less than 500 mmole

If the fluids increase .. the osmolarity decrease .. less than 500 mmole

If the fluids decrease .. the osmolarity increase .. more than 800 mmole

In the question .. water diuresis .. so .. having urine more watery .. increase water .. lead to decrease
specific gravity .. and decrease osmolarity .. no relation with Ph .. because ph reflected by log (H+)
..reflected by presence of hydrogen ion .. no relation

Answer is C

Whts is HELLP Syndrome ???


H hemolysis , el elevated liver enzyme , lp low platelet count

71
H:Hemolysis
EL:Elevated liver enzyme
LP: low platelet count
HELLP Syndrome occur in sever pre-eclampsia condition

'' Dryness of mouth '' is the side effect seen with?


A. Atropine
B. Aetylcholine
C. Morphine
D. None

The blood cancer is known as


a) Haemolysis
b) Leukaemia
c) Haemophilia
d) Thrombosis

Which of the following diagnostic test would confirm a diagnosis of tuberculosis?


A. Bronchoscopy
B. Chest x-ray
C. Sputum culture
D. Tuberculin skin test

What event occurring in the second trimester helps the expectant mother to accept the
pregnancy?
A. Lightening
B. Ballotment
C. Pseudocyesis
D. Quickening

Superior vena cava opens in ____ part of heart ?


A.Right atrium
B. Right ventricle
C. Left atrium
D. Left ventricle

The normal amount of CSF in a man is about


A: 50 cc
B: 100 cc
C : 150 cc
D: 200 cc

What is the meaning of "Osteo " in medical terminology ?


a.bone
b.joint
c.cartilage

72
d.muscle

The nurse assesses the respiratory status of a client who is experiencing an exacerbation
of COPD secondary to an upper respiratory tract infection. Which of the following findings
would be expected?
1. Normal breath sounds
2. Prolonged inspiration
3. Normal chest movement
4. Coarse crackles and rhonchi

Antiembolism stockings are used primarily to:


A. Promote venous circulation
B. Provide external warmth
C. Prevent dependent edema
D. Hold foot dressings
(A )Promote venous circulation. Antiembolism stockings are elastic stockings designed to maintain
compression of small veins and capillaries in the legs.

Clotting time for humen?


A 2-8min
B 1-3min
C 15-17min
D 1min

Mr. Jose is admitted to the hospitalwith a diagnosis of pneumonia and COPD. The
physician orders an oxygen therapy for him. The most comfortable method of delivering
oxygen to Mr. Jose is by:
A. Croupette
B. Nasal Cannula
C. Nasal catheter
D. Partial rebreathing mask

(B )Nasal Cannula. The nasal cannula is the most comfortable method of delivering oxygen because
it allows the patient to talk, eat and drink

Pink puffer also known as.....????


Pink Puffer = Emphysema

Blue Bloater = Chronic Bronchitis

The nurse should turn the client on bed rest every 2 hours to prevent the development of
pressure ulcers. In addition, the nurse should:
1. Have the client walk at least twice a day.
2. Insert an indwelling urinary catheter.

73
3. Monitor serum albumin.
4. Monitor the white blood cell count.

The nurse should monitor the client’s serum albumin. A decreased serum albumin indicates
malnutrition and is considered a risk factor in the development of pressure ulcers. Other risk factors
include immobility ,incontinence, and decreased sensation. Having the client walk and inserting an
indwelling catheter require a physician’s order. The white blood cell count is monitored if an
infection is present.

In.........; "iron" lost is " twice " as normal.


A.menstruation
b. Dehydration
c. Dysentory
d. Atelactasis

Which of the following is characteristic of cardiogenic shock?


1.Hypovolemia.
2. Increased cardiac output.
3. Decreased myocardial contractility.
4. Infarction.

Which laboratory result will provide the most important information regarding the
respiratory status of a child with an acute asthma exacerbation?
1. Complete blood count (CBC).
2. Arterial blood gas (ABG).
3. Blood urea nitrogen (BUN).
4. Partial thromboplastin time (PTT).

B..
In acute asthma should be respiratory alkalosis (Ph above 7.45 .. CO2 below 35 .. HCO3 normal or
below 22) .. these result because have hyperventilation as compensation so excit more CO2..

In chronic asthma should be respiratory acidosis ( Ph below 7.35 .. CO2 above 45 .. HCO3 above
26) .. these results because there is no compensation and have CO2 retention..

Where are find the maximum sweat glands?


A glans penis
B palm&soles
C axilla& groin
D none of these

The term gavage indicates:


A. Administration of a liquid feeding into the stomach
B. Visual examination of the stomach
C. Irrigation of the stomach with a solution
D. A surgical opening through the abdomen to the stomach

gavage: administration of a liquid feeding into the stomach

lavage: irrigation of the stomach with a solution

74
Blood pressure measurement is an important part of the patient’s data base. It is
considered to be:
A. The basis of the nursing diagnosis
B. ************************ive data
C. An indicator of the patient’s well being
D. Subjective data
A patient about to undergo abdominal inspection is best placed in which of the following
positions?
A. Prone
B. Trendelenburg
C. Supine
D. Side-lying.

A female client is brought to the emergency department with second- and third-degree
burns on the left arm, left anterior leg, and anterior trunk. Using the Rule of Nines, what is
the total body surface area that has been burned?
A. 18%
B. 27%
C. 30%
D. 36%

Which factor increase heart beat?


A high temperature
B high pH
C high metabolism
D all

All........drugs; work as "vaso_dilators" !.


a. Beta_blockers
b. Nitrates
c. Diuretics
D. Calcium channel blockers

Which of the following findings in a 2-year-old child assists in identifying the cause of a
grand mal seizure?
A. Fever
B. Crackles in the lungs
C. Abdominal tenderness
D. Cardiac dysrhythmia

Which nursing diagnosis takes highest priority for a client with Parkinson’s crisis?
A. Imbalanced nutrition: Less than body requirements
B. Ineffective airway clearance

75
C. Impaired urinary elimination
D. Risk for injury

Answer B. In Parkinson’s crisis, dopamine-related symptoms are severely exacerbated, virtually


immobilizing the client. A client confined to bed during such a crisis is at risk for aspiration and
pneumonia. Also ,excessive drooling increases the risk of airway obstruction. Because of these
concerns, the nursing diagnosis of Ineffective airway clearance takes highest priority. Although the
other options also are appropriate ,they aren’t immediately life-threatening.

The time required for coagulation to take place is known as the.......time.


a. Bleeding
b. Prothrombin
c. Thrombin
d. Clotting

The physician has prescribed nitroglycerin to a client with angina. The client also has
closed angle glaucoma. The nurse contacts the physician to discuss the potential for:
1. Decreased intraocular pressure.
2. Increased intraocular pressure.
3. Hypotension.
4. Hypertension.

A client with pneumonia develops dyspnea with a respiratory rate of 32 breaths/minute and
difficulty expelling his secretions. The nurse auscultates his lung fields and hears bronchial
sounds in the left lower lobe. The nurse determines that the client requires which of the
following treatments first?
1. Antibiotics
2. Bed rest
3. Oxygen
4. Nutritional intake

Maximum heat loss from body?


A lungs
B skin
C brain
D kidney

A 7-month-old has a low-grade fever, nasal congestion, and a mild cough. What should
the nursing care management of this child include?
1. Maintaining strict bedrest.
2. Avoiding contact with family members.
3. Instilling saline nose drops and bulb suctioning.
4. Keeping the head of the bed flat.

Nausea and vomiting is an expected side effect of chemotherapeutic drug use. Which of
the following drug should be administered to a client on chemotherapy to prevent nausea

76
and vomiting?
A. Metochlopramide (Metozol)
B. Succimer (Chemet)
C. Anastrazole (Arimidex)
D. Busulfan (Myleran)

A .Metochlopramide (Metozol) – antiemetic. Succimer (Chemet) – chelating agent for lead


poisoning. Anastrazole (Arimidex) – hormone regulator .Busulfan (Myleran) – alkylating agent

A client has a history of chronic obstructive pulmonary disease (COPD). As the nurse
enters the client’s room, his oxygen is running at 6 L/min, his color is flushed and his
respirations are 8/min. What should the nurse do FIRST?
A. Obtain a 12-lead EKG
B. Place client in high Fowler’s position
C. Lower the oxygen rate
D. Take baseline vital signs

C .. in COPD patiend the oxygen should be 1 to 2 liter per minute .. unless in life threatening
sotuation we can set him on the full concentration

An expected physiologic response to a low potassium level is:


a. Cardiac dysrhythmias.
b. Hyperglycemia.
c. Hypertension.
d. Increased energy.

Which of the following treatments would the nurse expect for a client with a spontaneous
pneumothorax?
A. Antibiotics
B. Bronchodilators
C. Chest tube placement
D. Hyperbaric chamber

Which finding would the nurse consider abnormal when performing a physical assessment
on a 6-month-old?
1. Posterior fontanel is open.
2. Anterior fontanel is open.
3. Beginning signs of tooth eruption.
4. Able to track and follow ************************s.

Anterior frontanil should closed at 9 to 18 months..

Posterior frontanil should closed at 2 to 3 months..

which of the following spread through Respiratory route?


a) Measles
b) Mumps
c) influenza
d) all of these

77
Who needs more iron?
A Men
B Women of childbearing age
C Postmenopausal women
D All of the above

Hormone responsible for Milk Ejection


a. prolactin
b.LH
c.FSH

All are good factors for wound healing EXCEPT ?


A. Vit C
B. Foreign body
C. Blood flow
D. Margin approximation

Morphine administration produces all of the following effects except:


(a) Diarrhoea
(b) Miosis
(c) Bradycardia
(d) Respiratory depression

Morphin is a narcotic .. one of the major side effect of narcotic is constipation .. in addition ..
lopramide(emidium) is a weak narcotic given to patients with diarrhea to use its side effect of
constipation to treat diarrhea

Main predisposing factor for lung cancer is ?


A. Environmental
B. Salted water
C. Cigarette smoking
D. Recurrent RTIs

Schizophrenia is characterized by all of following EXCEPT ?


A. Good insight
B. Hallucinations
C. Delusions
D. Abnormal behavior
Blood is formed in the human adult by the
a Heart
b Spleen
c Kidney
d Bone marrow

The Universal donor group of blood is


aO
bA
cB
c AB

78
The physician has ordered an MRI for a client with an orthopedic ailment. An MRI should
not be done if the client has:
A. The need for oxygen therapy
B. A history of claustrophobia
C. A permanent pacemaker
D. Sensory deafness

And patients with claustrophobia


We should make him calm or sedate him
For ortho patients any orthopedic implement (metallic) is considered contraindications.

Which of the following is a major risk factor for having a low-birth-weight baby?
1. Heredity.
2. Age.
3. Drug use during pregnancy.
4. Poor nutrition.

Uses Of Ranitidine?
A) Fever
B) Flu
C) Ulcer
D) Constipation

While performing cardiopulmonary resuscitation (CPR) on a 5-year-old child, the nurse


palpates for a pulse. Which of the following sites is best for checking the pulse during CPR
in a 5-year-old child?
a. Femoral artery.
b. Carotid artery.
c. Radial artery.
d. Brachial artery.

What two organs in the body serve as a compensatory function to maintain acid base
balance?
A. Kidneys and Lungs
B. Lungs and Spleen
C. Heart and Liver
D. Gallbladder and Appendix

A 1-month-old full-term infant is receiving breast milk only. The infant should receive
supplementation of:
a) Iron
b) Vitamin D
c) Protein
d) Vitamin B12

79
Rahul is admitted to the hospital with a possible diagnosis of appendicitis. On physical
examination, the nurse should be looking for tenderness on palpation at McBurney’s point,
which is located in the
A. left lower quadrant
B. left upper quadrant
C. right lower quadrant
D. right upper quadrant

If the LMP is Jan. 30, the expected date of delivery (EDD) is


A. Oct. 7
B. Oct. 24
C. Nov. 7
D. Nov. 8

A client is diagnosed with a spontaneous pneumothorax necessitating the insertion of a


chest tube. What is the best explanation for the nurse to provide this client?
A. “The tube will drain fluid from your chest.”
B. “The tube will remove excess air from your chest.”
C. “The tube controls the amount of air that enters your chest.”
D. “The tube will seal the hole in your lung.”

Which condition is most likely to have a nursing diagnosis of fluid volume deficit?
A. Appendicitis
B. Pancreatitis
C. Cholecystitis
D. Gastric ulcer

Trousseau's sign is associated with which electrolyte imbalance?


A. Hyponatremia
B. Hypocalcemia
C. Hypernatremia
D. Hypercalcemia

Presence of blood in urine is called.....


A-Blooduria
B-Haematuria
C-Chyluria
D-Mellituria

Obesity is diagnosed in most children by:


a) Hemoglobin A 1C
b) Serum leptin levels
c) Inspection
d) Body mass index (BMI)

80
Which drug name is not a generic name?
A. Atropine
B. Valium
C. Ipratropium
D. Hyoscine
If you are in a state of analgesia, you are
A Unable to stop pain
B Unable to feel pain
C Unable to locate pain
D Medically in shock because of pain

The bone that protects the brain is called the:


Sternum
Cranium
Clavicle
Pelvis

given carefully in diabetic patient:


A. glucomin
B. Pronolol
C. Ratadine
D. Avandia

Which of the following condition has an increased risk of for developing hyperkalemia?
A. Crohn's disease
B. Cushing's disease
C. Chronic heart failure
D. End-stage renal disease

Milk ijection by -??


A- estrogen
B- prolactin
C- oxitocin
D- FSH

‫بروالكتين لتصنيع الحليب ولكن اخراج الحليب هوا االوكسيتوسين‬

Which of the following nursing intervention is appropriate when an IV infusion infiltrates?


A. Elevate the site
B. Discontinue the infusion
C. Attempt to flush the tube
D. Apply warm, mosit compress

Only vitamin formed by skin


a. A
b. D
c. E
d. K

81
Kernig,s sign is suggestive og
A= tuberculosis
B= Meningitis
C= Encephalitis
D=Rubella

Pernicious anemia is caused by a deficiency of ;


A. Iron
B. Folic acid
C. Thiamine
D. Vitamin B12

pernicious anemia caused by impaired absorption of vit B12 because of lack of internisic factor in
gastiric secretion

A patient with a history of heart failure has sustained a head injury. He is receiving
mannitol because he develops cerebral edema. The patient should be monitored for what
possible problem?
A. Pruritus
B. Bradycardia
C. Disorientation
D.Circulatory overload

Manitol is a osmotic diuretics lead the fluids to shift from third space (brain) in a case of cerebral
edema to the blood .. leading to increase overload of solution in blood (circulatory overload) .. lead
to increase blood pressure (the pressure exerted from the blood on the blood walls)..

To prevent this overload .. the patient should recieve two diuretics at the same time: (manitol &
lazix)

manitol lead to fluid shifting from the third space to the blood
lazix lead to fluid execreted from the blood to outside the body through the kidneys

A client has been taking furosemide (Lasix) for 2 days. The nurse should assess the client
for:
1. An elevated blood urea nitrogen (BUN) level.
2. An elevated potassium level.
3. A decreased potassium level.
4. An elevated sodium level.

Lazixs or fusemide is a loop diuretics lead to all the electrolyte Hypo .. except glucose and uric acid
.. make them hyper ..

Hydrocholorothiazide lead to all electrolyte hypo except glucose and uric acid and calcium .. make
them hyper..

Manitol .. osmotic diuretics .. all the diuretics make hypotension because execrete the fluids from
the blood except manitol make hypertention because it increase exertion of fluids from third space
to blood .. important diuretics in cerepral edema..

82
Lazix should used at the same time of using mannitol .. manitol excrete fluids from third space to
blood .. lazix excrete fluids from blood to outside the body through the kidneys..

Aldactone .. potassium sparing diuretics lead to gynecomastia and hypercalcemia..

In acute ascitis or anasarca (generalized edema) given furesmide

In chronic acsitis or anasarca (generalized edema) given aldactone

What is the meaning of " stetho " in the medical term ?


a. Chest
b. Stomach
C. Heart
d. Lungs

The " white " part of your " Eye " is called the ........?
a.iris
b.sclera
c. Lens
d. Pupil

Colour of " Blood " carried by "cappillaries " of the body is........?
a. Blue
b. Violet
c. Bright red
d. Dark red

The client is receiving heparin for thrombophlebitis of the left lower extremity. Which of the
following drugs reverses the effects of heparin?
❍ A. Cyanocobalamine
❍ B. Protamine sulfate
❍ C. Streptokinase
❍ D. Sodium warfarin

What is the meaning of "B.B.F " during prescription of drugs?


a. Before dinner
b. Before breakfast
c. Bed time
D. Four times a day

Which " cough " is more " dangerous ". ?


a. Dry
b. Productive

Those " drugs " which are used in the treatment of " cancer diseases" are called the.........?
a. Anti_Microbial Agent
b. Chemotheraphy
C. Antibiotic Agent

83
d. Phsiotheraphy

Pregnant diagnosed with UTI. The safest antibiotic is:


a) Ciprofloxacin
b) Ampicillin
c) Tetracycline

A nurse is administering IV furosemide to a patient admitted with congestive heart failure.


After the infusion, which of the following symptoms is NOT expected?
A. Increased urinary output.
B. Decreased edema.
C. Decreased pain.
D. Decreased blood pressure.

The first milk of female after parturation is called?..


(a)colostrum
(b)seminal fluid
(c)viscous liquid
(d)none

Clostrum milk .. 1-3 days


Transitional milk .. 3_10 days
Mature milk .. after 10 days

The nurse cares for a client receiving a balanced complete food by tube feeding. The
nurse knows that the MOST common complication of a tube feeding is which of the
following?
a. Edema
b. Diarrhea
c. Hypokalemia
d. Vomiting

Loss of " memory "(Dementia ) is also known as the .............disease.


a. Grave's
b. Alzheimer's
c. Epilepsy
d. Parkinson's

The nurse is caring for a client diagnosed with an anterior myocardial infarction 2 days
ago. Upon assessment, the nurse identifies a new systolic murmur at the apex. The nurse
should first:
a. Assess for changes in vital signs.
b. Draw an arterial blood gas.
c. Evaluate heart sounds with the client leaning forward.
d. Obtain a 12 Lead electrocardiogram.

The nurse is assessing a client who is in the early stages of cirrhosis of the liver. Which
focused assessment is appropriate?
1. Peripheral edema.
2. Ascites.
3. Anorexia.

84
4. Jaundice.

A client is brought to the emergency room bleeding profusely from a stab wound in the left
chest area. The nurse’s assessment reveals a blood pressure of 80/50, pulse of 110, and
respirations of 28. The nurse should expect which of the following potential problems?
a. Hypovolemic shock
b. Cardiogenic shock
c. Neurogenic shock
d. Septic shock

Which of the following nursing interventions is most important in preventing postoperative


complications?
1. Progressive diet planning.
2. Pain management.
3. Bowel and elimination monitoring.
4.Early ambulation.

A patient who has been diagnosed with vasospastic disorder (Raynaud's disease)
complains of cold and stiffness in the fingers. Which of the following descriptions is most
likely to fit the patient?
1. An adolescent male.
2. An elderly woman.
3.A young woman.
4. An elderly man

Barrel shaped chest is the feature of ________ ?


.
A. Interstitial lung disease
B. Bronchiectasis
C. COPD
D. Bronchiolitis

PPH can be prevented by administration of _______ drug ?


.
A. Prolactin
B. Oxytocin
C. Prostaglandins
D. None of above

A nurse is providing discharge information to a patient with peripheral vascular disease.


Which of the following information should be included in instructions?
1. Walk barefoot whenever possible.
2. Use a heating pad to keep feet warm.
3.Avoid crossing the legs.
4. Use antibacterial ointment to treat skin lesions at risk of infection.

As a knowledgeable nurse, you know that the following are part of the five rights except:
A. Right dose
B. Right route
C. Right drug
D. Right room

85
The nurse in charge is caring for a patient who is in the first stage of labor. What is the
shortest but mostdifficult part of this stage?.....
a. Active phase..
b. Complete phase..
c. Latent phase..
d.Transitional phase

" Lumbar Puncture (L.P ) " is usually performed at ..........or lower to avoid damage to the
spinal cord.
a. L1.......L2
B. L3.......L4
C. L2........L4
D. L4........L5

What is the meaning of " P O " during prescription of drugs ?


A.by vein
b.by muscle
c.by mouth
d.by rectum

The nurse is performing an assessment in a client with a suspected diagnosis of cataract.


The chief clinical manifestation that the nurse would expect to note in the early stages of
cataract formation is:
1. Eye pain
2. Floating spots
3. Blurred vision
4. Diplopia

Only the__________ nerve extends from cranial to abdomen.


A: accessory
B: vagus
C: trigeminal
D: none of the above

What two organs in the body serve as a compensatory function to maintain acid base
balance?
A. Kidneys and Lungs
B. Lungs and Spleen
C. Heart and Liver
D. Gallbladder and Appendix

When assessing the abdomen, which sequence does the nurse follow?
1. Palpation, percussion, auscultation, and inspection
2. Inspection, palpation, percussion, and auscultation
3. Percussion, inspection, auscultation, and palpation
4. Inspection, auscultation, percussion, and palpation

The ..........is a "non_infectious " disease .

86
a. Aids
b. Tuberculosis
c. Syphilis
d. Diabetes

Everyday , the human " stomach " produces about ........liters of " Hydrochloric Acid ".
a. 4
b. 2
c. 8
d. 9

A client has developed thrombophlebitis of the left leg. Which of the following nursing
interventions should be given the HIGHEST priority?
1) Elevate leg on two pillows
2) Apply support stockings
3) Apply warm compresses
4) Maintain complete bed rest

Best blood test for MI is....... ‫؟!؟‬


Troponin .. specific to MI
CPKMP .. for recurrent MI
Myoglobin .. the first will increased in MI

When a patient is asymptomatic, this means that the patient ________.


A)shows no symptoms or signs
B) has a hospital-acquired infection
C) is a hypochondriac
D) shows severe symptoms or signs

A client comes to the clinic complaining of unexplained black and blues and bloody
appearing urine. Which type of medication is it most important to find out if the client is
taking?
a. Antibiotic
b. Antipruritic
c. Antianemic
d. Anticoagulant

The pharynx have all the following parts except


a) Nasal pharynx
b) Oro pharynx
c) Larango pharynx
d) Ulvula pharynx

Which gland is play both role exocrine and endocrine....?


A. Pituitory
b. Thyroid
c. Pancrease
d. Adrenal

which solution are used to fill the balloon of a Foley catheter


a. Ns

87
b. Distel water
c. Both

The preffered blood vessel to make CABG surgery:


A. LIME (Left Internal Mammary Artery)
B. RIMA (Right Internal Mammary Artery)
C. Right Sephenous Vein
D. left Suphenous Vein
Peripheral Blood Vessels for CABG : LIMA (Left Internal Mammary Artery)
usual Using Blood Vessels in CABG: Lesser Suphenous Vein nor Right or Left

Very high level of B-hCG is indicator of ?


. A. Normal pregnancy
B. Twin pregnancy
C. Singleton pregnancy
D. Infertility
In singleton (one baby) pregnancy .. the placenta secrete B-HCG to stimulate corpus
leutum (fertelized ovum) to produce estrogen and progesteron to promote pregnancy .. this
occur until the placenta can produce estrogen and progesteron alone ..
in twins .. more B-HCG secreted in order to stimulate two corbus leutum
note B-HCG who the cause of nausea and vomiting in pregnancy leading to hyperemesis
gravidarum .. more BHCG .. more hyperemesis .. so un twins .. more nausea and vomiting

Most common cause of death in acute renal failure is ?


A. Hyperkalemia.
B. Hypokalemia.
C. Hypernatremia.
D. Hyponatremia.

Most common cause of post partum hemorrhage is ?


A. Lacerations
B. Infections
C. Uterine atony
D. Endometriosis

4T cause of PPH
TONE 70%
TRAUMA 20%
TISSUE 10%
THROMBIN 1%

the indication of apendicitis is:


A. Negative Obturator sign
B. Positive Psoas Sign
C. Negative Rebound sign
D. Positive Bruzinsky sign

To assess appendicities by signs .. we have 3 test:

88
.1Answer C .. incorrect .. Rebound test .. positive in appendicitis .. press deeply on mecbury point
(area of right lower quadrent .. ) and release quickly .. in positive test .. pain will occur in rovsing
point( area of left lower quadrent.. )

.2Answer A .. incorrect .. Obturator test .. positive in appendicitis .. bending both legs toward
abdomen .. in positive finding .. the pain exarbated in mecbury sign

.3Answer B .. correct .. Psoas test .. positive in appendicitis .. bending both legs toward back .. in
positive fondings .. the pain will exarbated in mecbury point..

Answer D .. incorrect .. bruzinsky sign .. is a sign used to assess manengeal stimulation in


menengitis .. bending both legs .. and the patient responsively bending his head toward chest to
decrease manengial pain!!

Which of the following local anesthetics has the shortest half-life?


1. Lidocaine
2. Prilocaine
3. Bupivacaine
4. Articaine

While performing cardiopulmonary resuscitation (CPR) on a 5-year-old child, the nurse


palpates for a pulse. Which of the following sites is best for checking the pulse during CPR
in a 5-year-old child?
a. Femoral artery.
b. Carotid artery.
c. Radial artery.
d. Brachial artery.

D
For adult .. carotid
For pediatric .. brachial

An expected physiologic response to a low potassium level is:


1. Cardiac dysrhythmias.
2. Hyperglycemia.
3. Hypertension.
4. Increased energy.

A client is admitted for an MRI. The nurse should question the client regarding:
A. Pregnancy
B. A titanium hip replacement
C. Allergies to antibiotics
D. Inability to move his feet

Answer A is correct. Clients who are pregnant should not have an MRI because radioactive isotopes
are used. However, clients with a titanium hip replacement can have an MRI, so answer B is

89
incorrect. No antibiotics are used with this test and the client should remain still only when
instructed, so answers C and D are not specific to this tes

What is the term that means "new growth?"


A. Anaplasia
B. Metaplasia
C. Neoplasia
D. Hyperplasia

A client is receiving Total Parenteral Nutrition (TPN). Which is a common complication of


TPN for which the nurse must monitor?
A. Phlebitis
B. Hypoglycemia
C. Electrolyte Imbalance
D. Fluid Volume Deficiency

The answer is D.The hypertonic fluid draws water from the tissues and can lead to fluid volume
deficit. Hyperglycemia may be a problem, not hypoglycemia. The physician writes TPN orders
based on the client’s electrolyte balance TPN is always administered by central line so phlebitis
would be a minimal risk.

A client telephones the clinic to ask about a home pregnancy test she used this morning.
The nurse understands that the presence of which hormone strongly suggests a woman is
pregnant?
A. Estrogen
B. HCG
C. Alpha-fetoprotein
D. Progesterone

After a nasogastric (NG) tube has been inserted, the nurse can most accurately determine
that the tube is in the proper place if which of the following can be demonstrated?
1. The client is no longer gagging or coughing.
2. The pH of the aspirated fluid is measured.
3. Thirty milliliters of normal saline can be injected without difficulty.
4. A whooshing sound is auscultated when 10 mL of air is inserted.

Four routes to determine NG place:

.1Push 10 ml air and auscultate stomach


.2Draw gastric fluids
.3Put the tube in water
.4Xray (more expenssive and less comon)

The emergency department nurse receives a client with extensive injuries to the head and
upper back. The nurse will perform what action to allow the best visualization of the
airway?
a. Head-tilt chin-lift in the supine position on a backboard

90
b. Head-tilt chin-lift in the Trendelenburg position
c. Jaw-thrust maneuver in semi-Fowler's position
d. Jaw-thrust maneuver in the supine position on a backboard

A nurse is caring for a child with acute glomerulonephritis. Frequent


monitoring/assessment of which of the following is a priority?
a. Blood pressure
b. Hematuria
c. Intake and output
d. Peripheral edema

The correct answer is 1. Acute glomerulonephritis (AGN) in children is most commonly


an immune complex disease induced by prior group A beta-hemolytic streptococcal
infection ofthe skin orthroat. A latent period of 2-3 weeks occurs between the
streptococcal infection (eg .pharyngitis) and the symptoms of AGN. Clinical
manifestations of AGN include periorbital and facial'generalized edema, hypertension,
and oliguria, which are due primarily to fluid retention (decreased kidney filtration). The
urine is tea-colored and cloudy due to the presence of protein and blood. Although most
clients recover spontaneously within days, severe hypertension is an anticipated
complication that must be identified early. Monitoring and control of blood pressure are
most important as these prevent further progression of kidney injury and development of
hypertensive encephalopathy or pulmonary edema.
How long after the administration of tuberculosis intradermal skin test, result should be
evauted?
a- Immediately
b- within 24 hours
c- In 48 to 72 hours
d- After 5 days

Serious adverse effects of oral contraceptives include:


A. Increase in skin oil followed by acne.
B. Headache and dizziness.
C. Early or mid-cycle bleeding.
D. Thromboembolic complications.

A female nurse in the intensive care unit is caring for a client who is intubated and has
subclavian central venous access. Which nursing intervention is most important to prevent
the spread of infection to this client?
a. Frequent hand hygiene
b. No artificial nails
c. Use of chlorhexidine bath wipes
d. Wearing personal protective equipment

Drug of choice for Pregnancy induced Diabetes mellitus ?


.
A. Insulin.
B. Glimepride.
C. Gliburide.
D. Metformin.

Which of following drug is contraindicated in atrial fibrillation ?

91
.
A. Propanolol.
B. Atenolol.
C. Salbutamol.
D. Verampimil.

Increased steroids level in body is called _____ ?


A. Cushings syndrome.
B. Hypothyroidism.
C. Hyperthyroidism.
D. None of above

Treatment of choice for type II diabetes mellitus is ______ ?


.
A. Metformin.
B. Glimepride.
C. Gliburide.
D. Insulin.

Which of following is oral anticoagulant ____ ?


A. Heparin sulfate.
B. Enoxaparin.
C. Warfarin.
D. Protamine sulfate.

Acute myeloid leukemia is characterized by increased number of _____ cells ?


. A. RBCs.
B. Neutrophils.
C. Platelets.
D. Lymphocytes.

The Heimlich maneuver (abdominal thrust), for acute airway obstruction, attempts to:
A. Force air out of the lungs
B. Increase systemic circulation
C. Induce emptying of the stomach
D. Put pressure on the apex of the heart

When caring for an infant who has undergone surgical repair of a myelomeningocele,
which of the following should the nurse report to the surgeon?
1. Seizures and vomiting.
2. Frontal bossing and sunset eyes.
3. Increased head circumference and bulging fontanel.
4. Irritability and shrill cry.

The client has clear drainage from the nose and ears after a head injury. How can the
nurse determine if the drainage is CSF?
A. Measure the ph of the fluid
B. Measure the specific gravity of the fluid
C. Test for glucose

92
D. Test for chlorides

An adult is receiving Total Parenteral Nutrition (TPN). Which of the following assessment
is essential?
A. evaluation of the peripheral IV site
B. confirmation that the tube is in the stomach
C. assess the bowel sound
D. fluid and electrolyte monitoring

A client has a herniated disk in the region of the third and fourth lumbar vertebrae. Which
nursing assessment finding most supports this diagnosis?
a. hypoactive bowel sounds
b. severe lower back pain
c. sensory deficits in one arm
d. weakness and atrophy of the arm muscles

Which of the following vein is commonly used for CABG


a)Femoral
b)greater saphenus
c)popliteal
d)brachial

abruptio placentae is a type of ..


a.postpartum hemorrhage
b.antepartum hemorrhage
c.both
d.non

An ABG analysis report shows: pH - 7.20; PCO2 - 35mmHg; HCO3- -20mEq/L.These


findings are suggestive of
A. Metabolic acidosis
B. Metabolic alkalosis
C. Respiratory alkalosis
D. Respiratory acidosis

Diabetes mellitus is characterized by the following except:


a) hypoglycemia.
b) polyuria.
c) polyphagia.
d) polydipsia
e) glycosuria

Which of the following blood study results would the nurse expect as most likely when
caring for the child with iron deficiency anemia?
A. Increased hemoglobin
B. Normal hematocrit
C. Decreased mean corpuscular volume (MCV)
D. Normal total iron-binding capacity (TIBC)

Most specific enzyme for MI?


A.CPK-M,

93
B.CPK-MB,
C.CPK-BB,
D.LDH,

Ck-mb ‫للقلب‬
Ck-bb‫للمخ‬
Ck-mm‫للعظام‬
‫هسا لو جينا وطلبنا للمريض‬ck ‫ لحالو واجا عالي مش مؤشر لوجود‬mi ‫ ليش النو لو اي عضله انضربت بالجسم بيرتفع ال‬ck
‫اما اذا بدنا نشخص انو‬mi ‫ الزم‬ck- mb‫ ساعات من الجلطه‬4 ‫والتربنين الي بيرتفع بعد‬

Deferoxamine is administered in overdose of:


A. Iron
B. Calcium gluconate
C. Digoxin
D. Beta blockers

What is the first choice of MI


a)Ecospirin
b)Streptokinase
c)Morphine
c)Heparin

The first intervention of MI


MONA
M: morphine
O: oxygen
N: nitroglycride
A:baby aspirin

ECT is applied to patient ?


A.Anti-social behavior
B.psychotic depression
C.Hysteria
D.Obsessive - compulsive behavior

he most important nursing responsibility following head injury is?


A . Monitoring vital sign & record
B.monitor GCS
C.Obser for restlessness

Exceeding which of the following serum cholesterol levels significantly increases the risk of
coronary artery disease?
A. 100 mg/dl
B. 150 mg/dl
C. 175 mg/dl
D. 200 mg/dl

The best drug for treatment of Diabetes ketoacidosis is _____?


.
A. Metformin.

94
B. Glimepride.
C. Gliburide.
D. Insulin.
E. Ringolactate.

a disease requiring restriction of fluids and salts are ?


(a)acute nephritis with edema .
(b)anemia.
( c)pellagra.
( d)peptic ulcer.

If the pregnant woman has Dystocia,the nurse should observe 1.hemorrhage


2.fetal death
3.cord prolaps
4.hypertension

A patient's chest pain, which only occurs with activity, is said to have:
a. Unstable angina
b. Stable angina
c. Angina pectoris
d. Myocardial infarction (MI)

Which is true about airways in asthma?


1. Hyporesponsive
2. Hyperresponsive
3. Nonreactive
4. None of the above

Anterior frontanella is closed by


a.18 month
b.15month
c.20 month
d.16 month

The rapid response team has been called to manage an unwitnessed cardiac arrest. The
estimated maximum time a person can be without cardiopulmonary function and still not
experience permanent brain damage is:
1. 1 to 2 minutes.
2. 4 to 6 minutes.
3. 8 to 10 minutes.
4. 12 to 15 minutes.

After delivery mother position is


a.Dorsal
b.Lithotomy
c.Leftlateral
d.None

A nurse is helping a suspected choking victim. The nurse should perform the Heimlich
maneuver when the victim:
1. Starts to become cyanotic.

95
2. Cannot speak due to airway obstruction.
3. Can make only minimal vocal noises.
4. Is coughing vigorously

Which of the following electrolytes can be used in the management of digitalis-induced


arrhythmias?
a. Calcium
b. Magnesium
c. Phosphorous
d. Sodium

Bell’s palsy affects which cranial nerve?


A. Optic nerve ( II Cranial nerve)
B. Occulomotor( III Cranial Nerve)
C. Trochlear ( IV Cranial nerve)
D. Facial ( Vll Cranial nerve)

Jaundice is defined as "Yellowish discoloration " of................?


A.................Viscera.
B.................Sclera.
C.................Conjunctiva.
D.................Cornea.

Which of the following is not a bone in the


human body?
(a)Sternum
(b) Humerus
(c)Pericardium
(d) Tibia

Which is the following largest and most muscular chamber of heart


a)right atrium
b)right ventricle
c)left ventricle
d)left atrium

The creamy substance on the newborn is known as


1.Amniotic sac
2.Normal skin
3.Vernix caseosa
4.Launge

A confirmational test for gestational diabetes ?


A. Fasting blood sugar
B. Urine sugar
C.glucose tolerance test
D. Fasting lipid profile with RBS

When dose ovulation occur


a. Between 12 to 16 days

96
b.Between 10 to 12 days
c.Between 18 to 21 days
A female client undergoes yearly mammography. This is a type of what level of
prevention?
A. primary
B. secondary
C. tertiary
D. nota

Which of the following factors would the nurse suspect as predisposing a client to placenta
previa?
A. Multiple gestation
B. Uterine anomalies
C. Abdominal trauma
D. Renal or vascular disease

.A.Multiple gestation is one of the predisposing factors that may cause placenta previa.
Uterine anomalies abdominal trauma, and renal or vascular disease may predispose a
client to abruptio placentae.

By the end of which of the following would the nurse most commonly expect a child’s birth
weight to triple?
A. 4 months
B. 7 months
C. 9 months
D. 12 months

A 58 year old woman has newly diagnosed with hypothyroidism. The nurse is aware that
the signs and symptoms of hypothyroidism include:
a. Diarrhea
b. Vomiting
c. Tachycardia
d. Weight gain
When administering an I.M. injection to an infant, the nurse in charge should use which
site?
a. Deltoid
b. Dorsogluteal
c. Ventrogluteal
d. Vastus lateralis

Which vaccine is contraindicated in pregnancy?


A. Rubella
B. Diphtheria
C. Tetanus
D. Hepatitis B

The client with cancer is receiving chemotherapy and develops thrombocytopenia. The
nurse identifies which intervention as the highest priority in the nursing plan of care?
1. Ambulation three times a day

97
2. Monitoring temperature
3. Monitoring the platelet count
4. Monitoring for pathological factors

What is the disease in which the individual has an extra chromosome (number 21)?
A Cushing's Syndrome
B Marfan Syndrome
C Down Syndrome

The client is receiving peritoneal dialysis. If the dialysate returns cloudy, the nurse should:
1. Document the finding
2. Send a specimen to the lab
3. Strain the urine
4. Obtain a complete blood count

A patient in the cardiac unit is concerned about the risk factors associated with
atherosclerosis. Which of the following are hereditary risk factors for developing
atherosclerosis?
A. Family history of heart disease.
B. Overweight.
C. Smoking.
D. Age.

A vaginal exam reveals a footling breech presentation. The nurse should take which of the
following actions at this time?
A. Anticipate the need for a Caesarean section.
B. Apply an internal fetal monitor.
C. Place the client in Genu Pectoral position.
D. Perform an ultrasound.

Applying a fetal heart monitor is the correct action at this time. There is no need to prepare
for a Caesarean section or to place the client in Genu Pectoral position (knee-chest), so
answers A and C are incorrect. Answer D is incorrect because there is no need for an
ultrasound based on the finding.

How many chromosomes are their altogether in a normal human cell?


A. 44
B. 46
C. 22
D. 26

The obstric forceps consists of


1.Two pieces of blades
2.1 piece
3.3piece
4.4piece

Which division of the nervous system initiates a response known as fight or flight?
A. The sympathetic nervous system
B. The parasympathetic nervous system
C. The somatic nervous system

98
D. None of the above

What is the most appropriate intervention lor a patient who develops heart block following
an anterior myocardial infarction and is hemodynamically stable?
1. IV lidocaine
2. IV epinephrine
3. Pacemaker placement
4. Electrical cardioversion

A patient with malnutrition will most likely exhibit an increased risk of which of the
following?
a. Cancer
b. Psychosis
c. Infection
d. Hypertension

jaundice is disease of???


1) pancreas
2) liver
3) stomuch
4) intestine

Difficulty in swallowing is clinically termed as :-


A. Dyspepsia
B. Dysphagia
C. Dyspnoea
D. Dysurea

increase in size or mass of tissue is considered as....


a) growth
b)development
c)maturation
d)progress

A patient come under RTA case. What is the first action nurse..
A. Taking history
B. Fluid supply
C. Stop bleeding
D. Pain killer administration

Peptic ulcer disease particularly gastric ulcer is thought to be cause by which of the
following microorgamisms?
A. E. coli
B. H. pylori
C. S. aureus
D. K. pnuemoniae

Which of the following diagnostic tools is most commonly used to determine the location of
myocardial damage?
a. Cardiac catheterization
b. Cardiac enzymes

99
c. Echocardiogram
d. Electrocardiogram

Where is insulin manufactured?


A. In the cells of living tissue.
B. In the lining of the small intestine.
C. In the gall bladder.
D. In the pancreas

GLOMERULOSCELOROSIS IS A FEATURE OF:


A) DIABETES MELLITUS
B) HYPERTENSION
C) PRE-RENAL CAUSES OF RENAL IMPAIRMENT
D) POST RENAL CAUSES OF RENAL IMPAIRMENT

WHICH OF THE FOLLOWING IS NOT INCLUDED IN NEPHROTIC SYNDROME TRIAD?


A) HYPOALBUMINEMIA
B) PROTEINURIA
C) EDEMA
D) HAEMATURIA

WHICH VEIN IS INVOLVE IN CORONARY BYPASS:


A) AZYGOUS VEIN
B) INFERIOR VENA CAEVA
C) GREAT SEPHENOUS VEIN

When assessing the abdomen, which sequence does the nurse follow?
1. Palpation, percussion, auscultation, and inspection
2. Inspection, palpation, percussion, and auscultation
3. Percussion, inspection, auscultation, and palpation
4. Inspection, auscultation, percussion, and palpation

.4Inspection, auscultation, percussion, and palpation


Because percussion and palpation will affect abd sounds when auscultation so auscult before them
... And inspection always 1st

Injections of which of the following decreases the blood glucose level-


1. Pro-insulin
2. Insulin
3. Glucagon
4. Aldosterone

Malaria is a disease which Affects the


1_spleen
2_lungs
3_kidney
4_heart

- 1spleen
Can cause spleenomegaly
Alaso hepatomegaly & cerebral malaria affects the brain

100
Which of the following is the most important risk factor for development of Chronic
Obstructive Pulmonary Disease?
A. Cigarette smoking
B. Occupational exposure
C. Air pollution
D. Genetic abnormalities

What is the most common cause of diarrhea in a child under the age of 6?
a. Salmonella
b. Rotavirus
c. Bacillus cereus
d. Clostridium difficile

The nurse has developed a plan of care for a client diagnosed with anorexia nervosa.
Which client problem should the nurse select as the priority in the plan of care?
A. Malnutrition
B. Inability to cope
C. Concern about body appearance
D. Lack of knowledge about nutrition

Captopril may be administered to a client with HF because it acts as a:


1. Vasopressor
2. Volume expander
3. Vasodilator
4. Potassium-sparing diuretic

Which of the following should the nurse do when a primipara who is lactating tells the
nurse that she has sore nipples?
A. Tell her to breast feed more frequently
B. Administer a narcotic before breast feeding
C. Encourage her to wear a nursing brassiere
D. Use soap and water to clean the nipples

A .Feeding more frequently, about every 2 hours, will decrease the infant’s frantic, vigorous
sucking from hunger and will decrease breast engorgement, soften the breast, and promote ease of
correct latching-on for feeding. Narcotics administered prior to breast feeding are passed through
the breast milk to the infant, causing excessive sleepiness .Nipple soreness is not severe enough to
warrant narcotic analgesia. All postpartum clients, especially lactating mothers, should wear a
supportive brassiere with wide cotton straps. This does not, however ,prevent or reduce nipple
soreness. Soaps are drying to the skin of the nipples and should not be used on the breasts of
lactating mothers. Dry nipple skin predisposes to cracks and fissures, which can become sore and
painful.

Which one of the following blood value determinations is most likely be useful to evaluate
the adequacy of the fluid replacement?
a. Creatinine levels
b. Blood urea nitrogen
c. Hematocrit level
d. C02 tension

101
The primary goal of therapy for a client with pulmonary edema and heart failure?
A Enhance comfort
B Improve respiratory status
C Peripheral edema decreased
D Increase cardiac output

Increasing cardiac output is the main goal of therapy for the client with heart failure or pulmo-nary
edema. Pulmonary edema is an acute medical emergency requiring immediate intervention. Respi-
ratory status and comfort will be improved when cardiac output increases to an acceptable level.
Peripheral edema is not typically associated with pulmonary edema

During the first 4 hours after a male circumcision, assessing for which of the following is
the priority?
A. Infection
B. Hemorrhage
C. Discomfort
D. Dehydration

Toxicity from which of the following medications may cause a client to see a green-yellow
halo around lights?
1. Digoxin
2. Furosemide
3. Metoprolol
4. Enalapril

A male elderly client is admitted to an acute care facility with influenza. The nurse monitors
the client closely for complications. What is the most common complication of influenza?
A. Septicemia
B. Pneumonia
C. Meningitis
D. Pulmonary edema
Which of the following is known as Cancer????
A.Benign tumor
B.malignant tumor
C.Both
D.None

What organ is inflamed when one has encephalitis?


A.) the heart
B.) the liver
C.) the brain
D.) the large intestine

The average daily urinary output for an adult should be?


a. 1000 to1200 mL
b. 1200 to 1400 mL
c. 1500 to 1600 mL
d. 1700 to 1800 mL

102
The primary cause of anemia in a client with chronic renal failure is:...
a) Decreased water intake....
b) Poor iron absorption....
c) Insufficient erythropoietin...
d) Acute infection

A nurse is caring for a pt of chronic diarrhea the monitor the client knowing that client may
show respiratory rate:
A.RR 30
B. RR 20
C. RR 14
D. RR 18

The nurse is writing a care plan for the 5-year-old child diagnosed with gastroenteritis.
Which client problem is priority?
1. Imbalanced nutrition.
2. Fluid volume deficit.
3. Knowledge deficit.
4. Risk for infection.

A common abnormal laboratory result associated with the development of peripheral


vascular disease (PVD) is:
a. High serum calcium level.
b. High serum lipid levels.
c. Low serum potassium level.
d. Low serum lipid levels.

What is the complication if a highly hypertonic solution has been administration into the
peripheral veins??
A.thrombus
B.phlebitis
C.extravasation
D.infiltration

A man is admitted to the telemetry unit for evaluation of complaints of chest pain. Eight
hours after admission, the client goes into ventricular fibrillation. The physician defibrillates
the client. The nurse understands that the purpose of defibrillation is to do which of the
following?
1. Increase cardiac contractility and cardiac output.
2. Cause asystole so the normal pacemaker can recapture.
3. Reduce cardiac ischemia and acidosis.
4. Provide energy for depleted myocardial cells.

A 69-year-old female client admitted with pneumonia is receiving gentamicin. For this
client, which of the following laboratory values would be MOST important for the nurse to
monitor?
a. BUN and creatinine
b. Hemoglobin and hematocrit
c. Sodium and potassium
d. Platelet count and clotting time

103
how many bone of newborn
280
300
220
240

What is the involuntary passing of urine during sleep?


A.incontinence
B.oliguria
C.nocturnal enuresis
D.nocturia

Nocturnal enuresis or nighttime urinary incontinence, commonly called bedwetting or sleepwetting,


is involuntary urination while asleep after the age at which bladder control usually occurs.

A nurse is preparing to insert a nasogastric tube into a client. The nurse places the client in
which position for insertion?
1. Right side
2. Low Fowler’s
3. High Fowler’s
4. Supine with the head flat

D.K occurs as a complication of:


a. ITP
b. Myelodysplasia
c. Obstetric cases
d. Aplastic anemia
e. DM

what are the name of smallest bone of our body ?


Stapes or stirrup is the smallest bone in the human body. It is one of the ear ossicles

Insulin is:
1-Protein
2-Carbohydrates
3-Fat
4-Mineral

Which class of drug is given to prevent heart failure in the first 24 hours after a myocardial
infarction (MI)?
1. Calcium channel blocker
2. ACE inhibitor
3. Beta blocker
4. Digitalis derivative

104
The answer is B. Ace inhibitors prevent conversion of angiotensin I to angiotensin II. A is
incorrect—Calcium channel blockers cause coronary and peripheral vasoconstriction. C is
incorrect—Beta blockers reduce heart rate and contractility. D is incorrect—Digitalis slows the
heart and increases the force of contraction.

which of the following complication is most common in patient receiving Total parentaral
nutrition (TPN)....??
1.Hypoglycemia
2.Electolyte Imbalance
3.Phlebitis
4.Fluid Volume Deficiency

When administering an I.M. injection to an infant, the nurse in charge should use which
site?
A. Deltoid
B. Dorsogluteal
C. Ventrogluteal
D. Vastus lateralis

A 10-year-old child with appendicitis is being prepared for surgery. Which of the following
would be the nurse's first action?
A. Administer an enema.
B. Insert a nasogastric tube.
C. Obtain vital signs.
D. Administer antibiotics.

After stroke, most common complication is


a. Neurological
b. Cardiac
c. Gastrointestinal
d. Respiratory……

What is the name of the transparent sac that encloses the fetus in uterus?
A amniocentesis sac
B amniotic sac
C congenital membrane
D trisomic membrane

A client has just returned to a nursing unit after an above-knee amputation of the right leg.
A nurse places the client in which position?
1. Prone
2. Reverse Trendelenburg’s
3. Supine, with the amputated limb flat on the bed
4. Supine, with the amputated limb supported with pillows

....... is used to find unknown genotype of an individual .


1)black cross
2)reciprocak cross
3)test cross
4)monohydric cross

105
Drug of choice in ischemic stroke is _____?
. A. Nimodipine.
B. Captopril.
C. Aspirin.
D. Dipyramidol.

who is formation of insulin in body?


a.pancreas's
b.beta cell's
c.liver
d.kidney

A 70 year old diabetic patient fell on ground and became unconscious.


What investigation will you order:
a. Urine ketone….
b. CT scan head
c. MRI
d. UCE
e. ECG

A client with thyroid cancer undergoes a thyroidectomy. After surgery, the client develops
peripheral numbness, tingling, and muscle twitching. Which type of medication should the
nurse be prepared to administer?
1. Thyroid supplement
2. Antispasmodic
3. Barbiturate
4. Calcium replacement

A patient with gastritis which types of blood they have in the stool
A.Clay color stool
B. Black stool
C. Fress blood in stool
D. Cofy brown

Mrs. Alaniz, a 36 years old client, returns to the unit following extensive colon surgery.
Mrs. Alaniz is receiving total parenteral nutrition (TPN). The function of total parenteral
nutrition is to:
a. Providing supplemental nutrition
b. Controlling nausea and vomiting
c. Achieving optimal nutritional status
d. Providing treatment for dysphagia

A client is being prepared for a thoracentesis. A nurse assists the client to which position
for the procedure?
1. Lying in bed on the affected side
2. Lying in bed on the unaffected side
3. Sims’ position with the head of the bed flat
4. Prone with the head turned to the side and supported by a pillow

106
Rationale :To facilitate removal of fluid from the chest, the client is positioned sitting at the edge of
the bed leaning over the bedside table, with the feet supported on a stool or lying in bed on the
unaffected side with the head of the bed elevated 30 to 45 degrees. The prone and Sims ’positions
are inappropriate positions for this procedure.

Test-Taking Strategy: Use the process of elimination. Eliminate option 1 first because, if the client
was lying on the affected side, it would be difficult to perform the procedure. Option 3 can be
eliminated next because the Sims’ position is used primarily for rectal enemas or irrigations. Next,
visualize the prone position. In the prone position ,the client is lying on the abdomen, which is not
an appropriate position for this procedure. Review the procedure for a thoracentesis if you had
difficulty with this question.

All of the following are cluster A disorder of personality except


A paranoid
B. Schizoid
C. Schizotypal
D. Borderline

Q.Which valve prevents the backwards flow of blood in to the left atrium?
A) Aortic valve
B) Pulmonary valve
C) Mitral valve
D) Tricuspid valve

patient in an accident lost 2500 ml of blood:


1. a transient response of fluid therapy occurs
2. he is president with tachycardia only
3. need blood transfusion

The nurse is providing initial first aid for a client with thermal burn injury in a community
setting. Which action is appropriate?
1. Apply betadine ointment over the area affected
2. Cover the burn with an occlusive dressing
3. Flush the burned area with cool water
4. Remove any adhered clothing that is on the burn are

A nurse is caring for a newborn immediately following a cesarean delivery. The nurse’s
highest priority is to
monitor the newborn for
1 Respiratory distress
2 Renal impairment
3 ABO incompatability
4 Kernicterus

Which of the following assessment finding is expected in a client with bacterial


pneumonia?
1. Increased fremitus.
2. Bilateral expiratory wheezing.
3. Resonance on percussion.

107
4. Vesicular breath sounds.

A nurse is preparing to remove a nasogastric tube from a client. The nurse should instruct
the client to do which of the following just before the nurse removes the tube?
1. Exhale.
2. Inhale and exhale quickly.
3. Take and hold a deep breath.
4. Perform a Valsalva maneuver.

CSF blockage or over production occurs in which


A.Cancer
B. Hydrocephalus
C. Shazophrenia
D. non of the abve

Which of the following is an isotonic solution?


(a)Lactated Ringers
(b) Half –Normal Saline
(c) One-third Normal Saline
(d) Mannitol

A common drug given to induce labor is ____.


a. Methergine
b. oxytocin
c. dantrolene
d. Demerol

Which type of shock is associated with low blood levels?


A Septic shock
B Cardiogenic shock
C Hypovolemic shock

nitial step while detecting pulmonary embolism?


A. Start IV line
B. Check vitals
C. Administer morphine
D. Administer oxygen

The physician has ordered an MRI for a client with an orthopedic ailment. An MRI should
not be done if the client has:
A. The need for oxygen therapy
B. A history of claustrophobia
C. A permanent pacemaker
D. Sensory deafness

The nurse is caring for a male client with a diagnosis of chronic gastritis. The nurse
monitors the client knowing that this client is at risk for which vitamin deficiency?
A. Vitamin A
B. Vitamin B12

108
C. Vitamin C
D. Vitamin E

What should a nurse do if she finds a client diagnosed with a kidney stone pacing the
hallway and complaining of pain?
a) Encourage the client to continue walking.
b) Encourage the client to rest to reduce the pain.
c) Instruct the client to sit in a chair to reduce pain.
d) Instruct the client to return back to bed and then position the client in a Semi-Fowler's
position.

The nurse receives emergency laboratory results for a client with chest pain and
immediately informs the physician. An increased myoglobin level suggests which of the
following?
1. Cancer.
2. Hypertension.
3. Liver disease.
4. Myocardial damage.
which of the following organ used in the purification of blood in human body?
a) liver
b) kidney
c) spleen
d)) lungs

we are performing CPR for an infant. Nurse understands that the chest compression rate
is at least?
A. 80 times/minute
B. 100 times/minute
C. 120 times/minute

he name selected by the original manufacturer based on the chemical structure of the drug
is the:
A Chemical name
B Drug name
C Generic name
D Trade name

There are a number of risk factors associated with coronary artery disease. Which of the
following is a modifiable risk factor?
A) Gender
B) Age
C) Obesity
D) Heredity

The nurse is assessing the reflexes of a newborn. The nurse assesses which of the
following reflexes by placing a finger in the newborn’s mouth?
1. Moro reflex

109
2. Sucking reflex
3. Rooting reflex
4. Babinski reflex

A child comes to the emergency room with right lower quadrant pain and vomiting. Which
of these test results, if elevated would be indicative of acute appendicitis?
A) Hematocrit
B) White blood cells
C) ESR
D) Blood Glucose

Which disorder places the client at risk for tissue necrosis and breakdown of bone
structure with decalcification?
a. Osteoarthritis
b. Osteomyelitis
c. Osteoporosis
d. Osteogenesis
The prescription orders 0.125 mg digoxin orally. Tablets containing 0.25 mg is available.
How many tablets will you administer?
A. 1/2
B. 5
C. 2
D. 1/5

The nurse receives emergency laboratory results for a client with chest pain and
immediately informs the physician. An increased myoglobin level suggests which of the
following?
1. Cancer.
2. Hypertension.
3. Liver disease.
4. Myocardial damage.

Which of the following is an isotonic solution?


(a)Lactated Ringers
(b) Half –Normal Saline
(c) One-third Normal Saline
(d) Mannitol

When administering an I.M. injection to an infant, the nurse in charge should use which
site?
a. Deltoid
b. Dorsogluteal
c. Ventrogluteal
d. Vastus lateralis

During first stage of labour; nurse says that cervix is fully dilated; it means cervix is dilated
:-
A. 0 cm
B. 8 cm
C. 10 cm

110
D. 12 cm

The occurrence of chronic illness is greatest in :-


A. Older adult
B. Adolescents
C. Young children
D. Middle-aged adults

A child is hospitalized because of persistent vomiting. The nurse monitors the child closely
for:
1. Diarrhea
2. Metabolic acidosis
3. Metabolic alkalosis
4. Hyperactive bowel sounds

The nurse is caring for a patient with the following blood gas results: pH: 7.52, pCO2:
30mmHg, HCO3: 24mEq/L. The nurse would interpret these values as which acid base
disorder?
1) Metabolic Acidosis
2) Metabolic Alkalosis
3) Respiratory Acidosis
4) Respiratory Alkalosis

A 24-year-old female client is scheduled for surgery in the morning. Which of the following
is the primary responsibility of the nurse?
a. Taking the vital signs
b. Obtaining the permit
c. Explaining the procedure
d. Checking the lab work

A nurse is reviewing the laboratory results for a child scheduled for tonsillectomy. The
nurse determines that which laboratory value is most significant to review?
1. Creatinine level
2. Prothrombin time
3. Sedimentation rate
4. Blood urea nitrogen level

Untreated hyperthyroidism during pregnancy may result in all of the following except:
A. Premature birth and miscarriage
B. Low birthweight
C. Autism
D. Preeclampsia

The Number Of Ribs In Human Body Is?


A) 12
B) 18
C) 20
D) 24

111
Lithotomy position is given to a patient in all of
the following conditions except :
A) rectal examination
B) enema
C) Delivery
D) catheterization

Paracetamol and Aspirin used as-


1. Antipyretic
2. Antiseptic
3. Analgesic
4. 1 & 3

Which of the following intravenous solutions would be appropriate for a patient with severe
hyponatremia secondary to syndrome of inappropriate antidiuretic hormone (SIADH)?
A. hypotonic solution
B. hypertonic solution
C. isotonic solution
D. normotonic solution

Hormones that promote the re-absortion of water


by nephron is:
A- androgen
B- vasopressin
C- parathormone
D- oxytocin

Hypnosis is not effective in


a) anxiety disorder
b) Hysteria
c) phobia
d) eating disorder

The term new born refer to


a) from birth to one week
b) from birth to one month of child
c) from birth to one year
d) non of these

From birth to 28days

Which hormone cause rapid mammary gland (Breast) enlargement in females at puberty?
A. Estrogen
B. Oxytocin
C. Prolactin
D. TSH

112
A. Estrogen
(Prolactin is responsible for milk production)

A client with suspected active lung tuberculosis is being scheduled for Diagnostic tests.
The nurse anticipates that which diagnostic test will most likely be prescribed to confirm
the diagnosis?
1. Chest x-ray
2. Skin testing
3. Sputum culture
4. White blood cell count

The nurse is preparing to insert a nasogastric tube into a client. The nurse should place
the client in which position for insertion?
1.Right side
2.Low fowler's
3.High fowler's
4.Supine with the head flat
during pregnancy HIV transmission occurs mostly during
a) first Trimester
b)second trimester
c)third trimester
d)during labour

Absence of speech and activity is


A.Autism
B. Mutism
C. Aphasia

Autism‫ مرض التوحد‬..


Mutism‫ انسان قادر ع الكالم لكنه يلتزم الصمت‬..
Aphasia‫ عدم القدره على الكالم‬..

A nurse is monitoring a 3-year-old child for signs and symptoms of increased intracranial
pressure (ICP) after a craniotomy. The nurse plans to monitor for which early sign or
symptom of increased ICP?
1. Excessive vomiting
2. Bulging anterior fontanel
3. Increasing head circumference
4. Complaints of a frontal headache

Which of the following is the most common symptom of myocardial infarction?


a. Chest pain
b. Dyspnea
c. Edema
d. Palpitations

The condition in which the heart cannot pump enough blood to meet the metabolic
requirement of body tissues is called:
a) Embolus

113
b) Arteriosclerosis
c) Heart failure
d) Heart block
e) Ischemia

Which type of shock is associated with low blood levels?


A Septic shock
B Cardiogenic shock
C Hypovolemic shock
D. Anaphylactic shock

The extension of cancer to distant tissues and organs is called:


A) apoptosis.
B) tropism.
C) metastasis.
D) malignancy.

A condition in which a person experiences a loss of appetite is called:


A Ileus
B colic
C emesis
D anorexia

A common drug given to induce labor is ____.


a. Methergine
b. oxytocin
c. dantrolene
d. Demerol

Islets of langerhns secret the following hormones


a. Prolactin
b. Testosterone
c. Insulin
d. Thyroxin
e. Glucagon

Kernig's sign is a striking feature of ....


1. Meningitis
2. Nehritis
3. Neuritis
4. Dendritis

The nurse is preparing a discharge plan to a female client with peptic ulcer for the dietary
modification she will need to follow at home. Which of the following statements indicates
that the client understands the instruction of the nurse?
A) "I should not drink alcohol and caffeine."
B) "I should eat a bland, soft diet."
C) "It is important to eat six small meals a day."
D) "I should drink several glasses of milk a day."

Presence of sugar in the urine called .

114
A. Renal threshold
B. Glycosuria
c.Oliguria
D. Enuresis

nurse analyzes the laboratory results of a child with hemophilia. The nurse understands
that which of the following would most likely be abnormal in this child?
1. Platelet count
2. Hematocrit level
3. Hemoglobin level
4. Partial thromboplastin time

The nurse is caring for a male client postoperatively following creation of a colostomy.
Which nursing diagnosis should the nurse include in the plan of care?
A. Sexual dysfunction
B. Body image, disturbed
C. Fear related to poor prognosis
D. Nutrition: more than body requirements, imbalanced

All of the following are signs of anaemia in a patient EXCEPT.


A. Yellowness of the skin
B. Tachycardia
C. Dyspnoea on exertion
D. Lethargy

Which of the following conditions is caused by elevated intra ocular pressure?


(a)Glaucoma
(b) Cataract
(c) Strabismus
(d) Myopia.

The differentiate sign to diagnose Myocardial Infraction from Angina pectoris is :


a. The patient’s pain will relieve with rest.
b. The patient’s Pain will relieve by taking Isordil Sublingual.
c. The Patient’s pain increased by breathing
d. The Patient’s pain will not relieve with rest or Isordil.

2. Post operative Patient complain of infected wound after 24 hours from the surgery day, you can
found in the wound site:
a. redness
b. swelling
c. bad odor

115
d. all of the answer are rights
3. how you will intervene to relieve swelling lower extremity with cast for 4 days?
a. change the cast.
b. Call the doctor
c. Elevate the extremity
d. Give message

4. patient with blood infusion, you found it with fever after the infusion begin with 19 minutes.
The right action is:
a. stop infusion immediately.
b. Call the doctor who incharge.
c. Continue blood infusion with slow rate.
d. a+b

5. The inferior vena cava bring the deoxygenated blood to heart via:
a. right ventricle
b. lift ventricle
c. right atrium
d. lift atrium
6. The artery which supply the myocardial muscle with oxygen and nutrient, called:
a. carotid
b. anterior vena cava
c. pulmonary artery
d. coronary arteries

7. One of The immediate action to a patient with Myocardial infraction is to give patient:
a. Paracetamol
b. Morphine
c. Oxygen
d. B+C

8. Patient with burn in his back and abdomen, then the percent of the burn is :
a. 18 %
b. 36 %
c. 27 %
d. 45 %

116
9. What is the first priority to do in a patient with burn?
a. check pulse
b. check breathing
c. give sedative
d. apply cold compresses

10. you can say the patient is Full oriented, if he is oriented to :


a. place
b. time
c. person
d. all of them

11. The best cannula used when you want to give blood intravenous to an adult patient is :
a. Yellow cannula
b. Blue cannula
c. Pink cannula
d. Green cannula

12. The patient told you that he still feels with a back pain after the administration of sedative
drug I.M by an hour, you think that he is lying. What is the right action you must take it?
a. Ignore his complain.
b. Inform the Doctor
c. Give him extra dose from drug.
d. Tell him that he is not feel pain and he have delusion.

13. doctor order is to give a drug to the patient Q.I.D., the best times to give this drug is at like
schedule
a. 6am-12pm-6am-12pm
b. 3am-9am-3pm-9pm
c. 6am-2pm-10pm
d. 6 am -6pm
14. The medication order is to give pethedine 100 stat. that mean you will give it :
a. now for one time
b. two times as needed
c. three times daily

117
d. once daily

15. if you find that the patients output is more than his intake from the fluids since three days, you
will suspect that he is complain or will complain from:
a. edema
b. dehydration
c. its normal result
d. give him lasix

16. if you read in the medication chart beside the drug name this abbreviation ( N/A), you will
know that this drug is :
a. stopped
b. increase the dose
c. decrease the dose
d. the drug is Not Available

17. you will use the Oropharengeal tube when the patient is :
a. cannot talk
b. unconscious patient
c. patient with N.G. tube
d. Patient with sever throat pain

18. R.T.A. Patient in E.R. , he start complain from Increase Pulse rate, Low blood pressure, and
decrease level of consciousness, you must think that he develop:
a. Urinary Tract Infection
b. Shock
c. Coma
d. Respiratory distress.

19. “Code Blue “, means :


a. Patient with arrest
b. Patient with hemorrhage
c. Patient in O.R
d. Patient with blood infusion

118
20. you find a victim arrested in the street according to car accident, you found a clear fluid
dropping from his ear, and you must give him rescues breathing, what you will do?
a. head tilt, chin lift
b. chest thrust
c. jaw thrust
d. put him in left side position

21. you heard the nurse talk to the doctor and he said “ the patient has macro Haematuria”. What
he means by haematuria?
a. Difficult of breathing
b. Difficulty of urination
c. The urine seems bloody
d. The patient cannot urinate.

22. You found the cardiac monitor show straight line in a sleeping patient in CCU, what you will
do?
a. call for CPR
b. give cardiac massage
c. check cardiac monitor leads
d. give cardiac shock
23. before you send the patient to the operation room, you must check :
a. is the patient fasting?
b. Did he sign the consent form?
c. Did he shaved the site of operation?
d. All of them
24. the yellow hard containers in the hospital used to waste which of the following?
a. used gauze, cotton.
b. Sharp waste
c. Used papers and fluids.
d. All of them.
25. if u know that the patient is complain from quadriplegia, you will know that he have weakness
in :
a. his arms
b. his legs
c. his legs and arms.
d. One arm and one leg.

119
26. Carotid artery is located in :
a. the nick
b. the head
c. the legs
d. the abdomen
27. The route of drug administration that provides the dependable absorption is-:
a. Oral
b. Intermuscular
c. Subcuntaneous
d. Intravenous

28. medication is being administered , the most accurate way to verify a patient’s identification is
to-:
a. call the patient by the name in the drug card or cardex
b. ask the patient to state his or her name
c. ask another nurse to identify the patient
d. check the patient’s identification bracelet.

29. when the nurse is administering medications , the patient informs the nurse that the tablet
usually received is a different color . the nurse should-:
a. insist that the patient take the tablet she poured
b. have the patient take the tablet & then recheck the order
c. leave the medication at the bedside & recheck the order
d. recheck the order before giving the drug
e. verify the medication with the physician.

30. all of the following are examples of mild allergy symptoms that may occur in response to
antibiotic therapy except
a. urticaria ( redness)
b. rash
c. wheezing
d. (pruritus ( itching

31. for which client is the oral rout of administration appropriate?


a. client D who has vomited twice in the last hour.
b. client E who is complaining of a sever headache
c. client F whose assessment reveals an absence of bowel sounds.
d. Client G who is only responsive to painful stimuli.

120
32. Which of the following organs is a primary site for the metabolism of drugs?
a. Heart
b. Liver
c. Pancreas
d. Intestine

33. Mr .Jonnes has the following order : ASA ( Aspirin ) 5 gr p.o bid pc . which of the following is
the best interpretation & correct scheduling of this order?
a. Give him ASA 5 gr at 9 : 00 am after breakfast) & 6:00 pm).
b. Give him ASA 5 gr at 10 :00 , 2 :00 & , 6:00 round the clock.
c. Give him ASA 5 gr per intramuscular injection at 9 : 00 at 3:00 pm.
d. Question the order as it is too high a dose then schedule it for 7 : 30 am & 5:00 pm.

34. Mrs. Higgins has refused to take her ASA ordered by the physician . the first action of the nurse
should be
a. Notify the physician of the refusal
b. Chart the refusal on the medication administration record.
c. Find out why she doesn’t want to take the ASA
d. Tell her she must take the ASA because the physician has ordered it

35. a medication order should never be implemented if- :


a. the nurse doesn’t know the physician.
b. the nurse doesn’t know the patient’s history.
c. the nurse Questions any part of the order
d. the nurse did not know witness the writing of the order.

36. Orderd : penicillin 400.000 u im now


Available : penicillin 800.000 u per 2 cc
How many ml should you give?
a. 2 .ml
b.1 ml
c. 0.2 ml
d. 0.4ml

37. Orderd : Deltasone 7.5 mg now.


Available : Deltasone 2.5 mg scored tablets.
How many should you give?
a. 3 tablets
b. 1 tablets
c. tablets ½ 2
d. tablets ½ 1

121
38. 20 minutes after receiving her noon dose of oral medication , Mrs. Rooney vomits . to assure
accuracy in medication administration the first thing nurse Mulligan should do is to-:
a. report the situation to the head nurse
b. contact the physician
c. administer another dose of medication
d. examine the vomits for signs of medication

39. which of the following routs for drug administration is the most common , least expensive ,
safest , and best tolerated by patient?
a. intramuscular
b. topical
c. oral
d. intravenous

40. you arrives in Mr. Rich’s room with the ASA [ aspirin ] he requested for the headache . you
find him in the bathroom . you are very busy & don’t have time to wait . you should:
a. tell him you will return and take the medication with you
b. ask him to com out of the bathroom immediately
c. ask his roommate to give him the ASA
d. leave the medication on the over-the-bed table since ASA is a nonprescription drug

41. you are preparing to administer Mrs. Carter’s eye drops . the correct position for her to
assume would be:
a. head titled forward , placing the drops in the conjunctival sac.
b. head titled backward , placing the drops in the lower conjunctival sac.
c. head titled forward , placing the drops directly on the eyeball.
d. head titled backward , placing the drops directly on the eyeball.

42. to instill drops in the adult patient , the ear canal is opened by pulling the ear-:
a. up and back
b. down and back
c. up and forward
d. back and forward

43. The nurse correctly administer an IM injection a ……………… degree angle


a. 15
b. 30
c. 45
d. 90

122
44. when giving medication , the label should be checked 3 times . which of the following is
not one of these times?
a. when the nurse reaches for the container
b. immediately prior to pouring medication
c. when the nurse located the drug on the shell
d. when replacing the container to the drawer or shelf.

45. which of the following is the reason for using the Z tract technique for injections ?
a. for medication of over 5cc in quantity
b. for medication that is highly irritating to subcutaneous tissue
c. for medication that stains the tissue
d. for medication that cannot be given orally

46. which of the following orders is complete-:


a. ampicillin 250mg IM q12hrs x 10 days
b. Maalox 30 cc po
c. humulin R insulin SQ in AM & noon
d. (OPV ( oral polio vaccine ) 0.5 cc

47. Mrs. Kelly tells the nurse that her arm is sore from an injection she received early in the day .
she states “ the nurse gave me a shot , and I heard her say that the needle was a 25 gauge . isn’t
that too big for a local injection ?” . your best response would be:
“ a. a 25 gauge needle is a small needle , but it can cause som discomfort . let me see your arm”.
b. really, Mrs. Kelly, no once gets hurts with a 25 gauge needle . you are over –reacting to the
shot“
c. a 25 gauge needle is very small . you are just nervous about the injection”.
d. a 25 gauge needle is very small. So you are wrong about the pain from the injection”.

48. Mrs. Davis has a written order from her physician for Demerol 100 mg stat . which of the
following best explain this order?
a. give it needed
b. give once when needed.
c. give once immediately
d. give once when specified

49. while checking Mr .Appendectomy’s vital signs , he requests his “ medication for the pain in
my incision “ the vital signs were:
BP 100/60 , T – 98.4 F , P – 66 , RR – 10. you decide he may not have the Morphine sulfate
injection at this time due to witch vital signs reading?
a. blood pressure
b. pulse
c. respiration
d. temperature

123
50. after withdrawing the needle from the tissue when administering an ID injection you -:
a. apply firm pressure to the site with your thumb.
b. do not rewipe the site
c. gently wipe the site
d. massage the site vigorously

51. Mr. Harvey , age 54 , is admitted with a diagnosis of congestive heart failure secondary to
chronic obstructive pulmonary disease[ COPD] . the physician orders sublingual nitroglycerin , 0.4
mg p.r.n , for chest pain , whin administering the nitroglycerin , the nurse should -:
a. tell Mr. Harvey to hold the tablet under his tongue and let it dissolve.
b. tell Mr. Harvey to swallow the tablet with water.
c. apply nitroglycerin ointment to Mr. Harvey’s chest.
d. mix the nitroglycerin with applesauce for Mr. Harvey to eat

52. asking the patient if he is allergic to any medication is a part of the ……………
phase of the nursing process?
a .assessment
b. planning
c. implementation
d. evaluation

53. teaching the patient about the medication is a part of the …………… phase of the nursing
process?
a. assessment
b .planning
c. implementation
d. evaluation

54. a nurse is unable to read the label on a bottle of liquid medication because the label is stained
from spillage . the nurse knows that the correct procedure is to -:
a. ask the charge nurse to verify the medication in the bottle and apply a new label.
b. smell and test the medication and apply a new label if certain of the contents .
c. empty the contents down the drain and notify the charge and the pharmacist.
d. send the bottle back to the pharmacy to be relabeled .

55. Mr. Walker will receive regular insulin . the nurse monitors for the onist of action in
approximately
a. 5 to 10 minutes.
b. 150 to 20 minutes
c. 30 to 60 minutes
d. 1 to 2 hours .

124
56. is it necessary for the nurse to wear gloves during administration of an intermittent feeding
through a G-tube?
a. yes , because it is a sterile procedure.
b. yes , because universal precautions must be maintained.
c. no , because it is not a sterile procedure.
d. no , because there is no danger of contact with body fluid if the procedure is performed
correctly.

57. to prepare the skin for injection the nurse would use:
a. friction with back and forth motions at the site with alcohol swab.
b. friction and alcohol swab moving from outer edge to center of site.
c. friction and alcohol swab with circular motions and palpate site gently with finger pad before
injection.
d. friction and circular motions with alcohol swab from center of site outward.

58. IM injection into the deltoid muscle should be limited to-:


a. 2.5 ml of solution
b. 2 ml of solution
c. 1 ml of solution
d. 0.5 ml of solution

59. a primary concern when giving heparin SC is to prevent:


a. pain and bruising.
b. pain and bleeding
c. bleeding and bruising
d. injecting a vein.

60. the type of needle selected for the ( dorsogluteal ) intramscular ( IM ) injections would be
based on
1 the size of the patient [ e.g., obese versus debilitated]
2 the viscosity of the antibiotic.
3 the position of the patient.
4 the volume of medication.
a. 1,2,3,4
b. 1,2,3
c. 2,3,4
d. 1,2

61. the largest gauge needle of the following is-:


a. 19 gauge
b. 20 gauge
c. .21 gauge
d. 22 gauge

125
62. Optimum normal urine out put hourly to an adult:
a .10cc
b. 25cc
c. 50cc
d. 100cc
63. scurvy is a deficiency of:‫اسقربوط‬
a. vit A
b. vit B
c. vit C
d. vit D
64. Specific gravity of urine to have a normal fluid balance
a. 1.005
b. 1.015
c .1.03
d. 1.020
65. 12 hours after delivery the fundus is at the level of
a. one fingerbreadth below the umbilicus.
b. one fingerbreadth above the umbilicus.
c. at the level of umbilicus.
d. below symphysis pubis

66. In dystocia the ,mother should be watched for-:


a. post partum haemorrhage.
b. hypertension.
c. cord proplase.
d. fetal death.

67. in the first trimester of pregnancy the vaccination to avoid-:


a. polio
b. rubella.
c .measles.
d. small pox.

68. in fetal circulation , the oxygenated blood placenta travels via-:


a. umbilical artery
b. umbilical vein.
c. doctus arteriosus.
d. ductus venosus.
69. first stage of labor ends with-:
a. pain occurs.
b. dilation of cervix 10 cm
c .delivery of the baby
d. delivery of the placenta

126
70. an electrolyte deficiency that affect the heart muscular activity-:
a. K
b. CAL
c .NA
d. MG
71. The most vital immediate observation to a new born baby is-:
a. Cardiac rate
b. Respiration rate.
c .Color
d. Tone
72. Signs of dehydration-:
a. Loss of skin turgor.
b. Low body temperature
c. High body temperature
d. Sweating
73. Common cause of death in burns-:
a. Hemorrhage
b. Neurogenic shock
c. Sepsis
d. Hypovolemic shock

74. The difference in close & open fracture is that , in open fracture you have to watch for-:
a. Infection
b. Hemorrhage
c .Inflammation
d. Pain

75. Epinephrine is not given via-:


a. IV
b .Orally
c. Parentally
d. Intracardiac
76. Ultra sound used -:
a .To detect fetal position
b. To detect diameter of fetal head
c. To detect placenta abnormalities
d. All of the above.
77. To prevent oral mucosa infection q4h , instruct patient to use-:
a. Warm saline gargle
b. Glycerin oil
c. Antibiotic
d. Warm gargle with mineral oil

127
78. One of the vital care after cholecystectomy-:
a. Bed rest
b. Low fat diet
c. Low Cholesterol diet
d. Low protein diet

79. X-ray of upper GI tract , the nurse should instruct the patient to take-:
a. Laxative previous day
b. NPO 8-12 hours
c. High fat diet
d. Normal diet

80. inorder to administer 5 ml syrup to patient , we use a-:


a. ½ tsp
b. 1 tsp
c. ½ table spoon
d. 1 table spoon

81. A very serious complication of hysterectomy-:


a. vaginal pain
b. vaginal discharge
c. abdominal distention
d. no urine out put

82. Deep breathing and coughing exercise done at-:


a. angle of 90 degree
b. angel 45 degree
c. lie laterally
d. semi-fowler’s position

83. Patient with spinal cord injury must be-:


a. not to move at all
b. log rolled
c. turn the back with support in the head

84. After spinal anesthesia , watch fore-:


a. hypotention / headache
b. fowler’s position to the optimum
c. numbness of extremities
d. analgesic

128
85. The cure for chronic renal failure is-:
a. dialysis
b. kidney transplant
c. diuretics
d. force fluids

86. Acetyl salycilate [ aspirin ] immediate side effect-:


a .tinnitis
b. nausea & vomiting
c. abdominal pain
d. vomiting & diarrhea

87. digoxin can be with held to a patient with an apical pulse rate of below-:
a. 45
b. 60
c. 70
d. 80

88. the best time to collect urine for culture-:


a. early morning awaking sample.
b. 30 minutes after voiding
c. 30 minutes after fluid
d. double voided

89. The postpartal patient should be watched closely during the first hours after delivery for-:
a. Uterine contraction
b .Vaginal bleeding
c. Hypotention
d. All of the above

90. The major goals for the burn patient in the first 48 hours after burn , is that the patient
a. Has fluid and electrolyte balance maintained
b. Develops no contractures
c. Dose not develop hyperthermia
d .Develops minimal scarring

91. The normal fasting blood glucose rang mg per 100 ml . of venous blood is :
a. 60-80
b. 80-120
c. 100-150
d. 100-200

129
92. before irrigating a client’s NGT the nurse must first-:
a. assess breath sounds
b. instill 15 ml. Of normal saline
c. ausculate for bowel sounds
d. check the tube for placement

93. the best method of preventing the spread of infection is-:


a. isolating all patients suspected of having an infection
b. wearing rubber gloves when performing all nursing procedures
c. washing the hands thoroughly before & after each contact with a patient
d. sterilizing the hands with strong germicide at least once a day

94. when caring for a client who has an open reduction and internal fixation of hip , the nurse
encourages active leg and foot exercise of the unaffected leg every 2 hours to help to-:
a. reduce leg discomfort.
b. maintain muscle strength.
c. prevent formation of clots.
d. limit venous inflammation

95. when administering an antibiotic or a vaccine , the nurse must be alert for the possibility of -:
a. overdoses and CNS depression
b. hypersensitivity and possible anaphylaxis
c. sings of increasing infection
d. orthostatic hypotension

96. immediately after a child is admitted with acute bacterial meningitis , the nurse should plan to
-:
a. assess the child’s vital signs every 3 hours
b. administer oral antibiotic medication as ordered
c. check the child’s level of consciousness every hour
d. restrict parental visiting until isolation is discontinued

97. when assessing the unconscious victim for pulselessness , which of the following is the best
artery to chick-:
a. radial
b. femoral
c. brachial
d. carotid

98. common signs and symptoms of jaundice include-:


a. ascites
b. dermatitis
c. icteric sclera

130
d. dark-colored stools

99. patient with head injuries are not given sedative because these drugs may-:
a. produce coma
b. depress the patient’s respiration
c. mask the patient’s symptoms
d. lead to cerebral hemorrhage
100. in treating puncture wounds , the first priority is to-:
a. stop the bleeding
b. cleans the wound
c. give prophylactic treatment aginst tetanus
d. remove the object from the wound

101. ones the bleeding site has been determined , the first emergency measure to institute during
hemorrhage would be to-:
a. apply a firm – pressure dressing
b. apply direct , firm – pressure over the bleeding area or the artery involved
c. apply a tourniquet just proximal to the wound
d. elevate the extremity

102. nursing measures that can be used to lower core body temperature include-:
a. immersing the patient in cold water
b. placing the patient on a hypothermic blanket
c. administering chilled saline enemas
d. all of the above measures

103. the female hormone that induces changes in endometrium to prepare uterus for
implantation of a fertilized ovum and maintenance of a pregnancy is called-:
a. aldosterone
b. testoterone
c. progesterone
d. estrogen
104. for a hearing – impaired client to hear a conversation , a nurse should-:
a. use a louder tone of voice than normal
b. use visual aids such as the hands and eyes when speaking
c. approach a client quietly from behind before speaking
d. select a public area to have a conversation

105. when dealing with a client with aphasia , the nurse should remember to-:
a. wait for him to communicate
b. speak loudly to ensure that the massage is received
c. speak from the client’s side to avoid overload
d. encourage writing of massages

131
106. Immunization should be started when child is-:
a. 6 months
b. 2 months
c. 1 months
d. 1 week

107. the most reliable method used for sterilizing hospital equipment to be free of spores and
bacteria is a. soaking in strong chemical
b. washing and drying it thoroughly after use
c. applying steam under pressure in an autoclave
d. boiling the equipment

108. nursing care of a child admitted with acute glomerulonephritis , should be directed toward-:
a. forcing fluids
b .promoting diuresis
c. enforcing strict bed rest
d. eliminating sodium from diet

109.the nurse empties a portable wound suction device when it is only half full because-:
a. it is easier and faster to empty the unit when it is only half full
b. this facilitates a more accurate measurement of drainage output.
c. their negative pressure in the unit lessens as fluid accumulates in it , interfering with further
drainage
d. as fluid collects in the unit it exerts positive pressure , forcing drainage back up the tubing and
into the wound.

110. a patient develops a small decubitus ulcer on the sacral area . the nurse should plan to deal
with this problem by-:
a .keeping the area dry
b. applying moist dressing
c. providing a low caloric diet
d. keep the patient on the right side

111. when administering heparin , the substance the nurse would keep available as the antidote
is-:
a. magnesium sulphate
b. protamine sulphate
c. calcium gloconate

112. the main effect of excess calcium on the myocardium is-:


a. spastic contraction
b. cardiac flaccidity
c .tetany
d. bradycardia

132
113. which of the following vitamin increase the retention of calcium and phosphorous ions in the
blood-:
a. Vit A
b .Vit B12
c. Vit. D
d. Vit C

114. If a SC injection is in to which level of tissue must the solution be released-:


a. Epidermis
b .Dermis
c. Subcutaneous
d. Any of the above

115. A critical nursing measure to employ with a patient on any sulphonamide drug is to -: ( ‫يعمل‬
‫)كرستل‬
a .Monitor blood pressure every 30 minutes
b. Force fluids
c. Watch for tinnitus.
d. Monitor electrolytes

116. The primary cause of decubitus ulcer is-:


a. Excessive perspiration
b. Pressure on bony area
c. Poor nutrition and inadequate fluid intake
d. Inability to control voiding

117. Contra indications to the use of anticoagulant include-:


a .Blood dyscrasias
b. Liver or kidney disease
c. Peptic ulcer
d. All of the above conditions

118. Progesterone is primarily used for the treatment of-:


a. Hypertension of pregnancy
b. Abnormal uterine bleeding
c. Unovulation
d. Hypermenorrhagia

119. The average daily amount if water eliminated from the kidneys as urine is-:
a. 500 cc
b. 1500 cc
c. 2000 cc
d. 1200 cc
the normal average is between 1200 – 1500 cc

133
120. surgical patients should be taught to perform leg exercises for the main purpose of-:
a. preventing muscle atrophy
b. preventing joint degeneration
c. improving circulation
d. preventing boredom

121. When a patient is vomiting post-operatively , the most important nursing objective is to
prevent-:
a. dehydration
b. aspiration
c .rupture of suture line
d. metabolic acidosis

122. which of the following is an appropriate diet for a patient with congestive heart failure -:
a. low-calorie , high-residue diet with no caffeine
b. low-calorie , low- residue diet with low Na
c. high-calorie , low-fat , low-protein diet
d. high –protein , no fat , no carbohydrate diet

123. To obtain a truly estimation of the patient’s average blood pressure , your nursing
assessment should include-:
a. only one blood pressure reading
b. serial reading , which should be taken at the same time each day for 7 days
c. serial reading , which should be taken every 2 hours over an 8 hour period for 2 days
d .blood pressure reading on both arms should be taken sitting and standing once a day for 2 days

124. dietary control of patients with ulcers should include-:


a. taking antacids every hour
b. increase roughage in the diet
c. eliminate highly seasoned food
d. increasing amount of carbohydrate

125.During physical examination , the part of the kidney may be felt on deep palpation is-:
a. lower pole of the right kidney.
b. lower pole of the left kidney.
c. entire right kidney.
d. right and left kidney

126. The major nursing goals post-operative care of the cataract patient is to-:
A. prevent hemorrhage and stress on the sutures
B. prevent hemorrhage and eye stress.
C .prevent increased intro-ocular pressure and to promote better visual acuity
D. promote decreased intro-ocular pressure and to maintain a visible lens .

134
127. which of the following lab. test must be done on a patient with major burns , prior to
administration of antibiotics-:
A. complete blood account
B. wound culture
C. type and cross match
D. sensitivity studies
128. for how many days after a facial wound are sutures usually left in place?
A. 3 days
B. 10 days
C. 14 days
D. 7 days

129. To control bleeding from the radial artery , pressure to be applied directly above the-:
A. Thumb on the flexor surface of the arm.
B. Thumb on the extensor surface of the arm.
C. Little finger on the flexor surface of the arm.
D. Little finger on the extensor surface of the arm.

130. All patients taking tranquilizers must be warned that they may feel-:
A. Anxious
B. Nauseated
C. Clumsy
D. Drowsy

131. Witch of the following indicates placental separation after delivery-:


A. A globular – shaped uterus
B. A sudden rise of the funds
C. A sudden gush of blood
D. All of the above signs

132. If a patient whose membranes ruptures during labor and prior to descent of the head , the
nurse must assess the patient fore-:
A. Hemorrhage
B. Pain
C. Cord prolapsed
D. Uterine inertia

133. A must important nursing measure in the prevention of thrombophebitis for the post-
partum patient is-:
A .Elastic stocking
B. Early ambulation
C. Anticoagulant
D. Isometric exercise

135
134. As a child increases age , cardiac and respiratory rate should-:
A. Increase
B. Decrease
C. Remain unchanged
D. Stabilize at the adult level

135. An excellent diet suggest for an elderly patient is-:


A. High-calorie , low-fat , high carbohydrate diet
B. low-calorie , low-fat , lot of fruits and vegetables
C. High-calorie , high-fat , high protein diet
D. low-calorie , high-fat , high protein diet
136. in opened and close fracture , the nurse should be aware of which of the following when
open fracture is presented?
A. mal-alignment
B. infection
C .bleeding
D. pain

137. nursing consideration for a child with osteomyelitis should include-:


A. relieving of pain
B. maintaining antibiotic therapy
C. maintaining adequate nutrition
D. all of the above

138.how long should the affected extremity be kept elevated after the application of cast?
A. until the cast is dry
B. when the leg is no longer painful
C. for 24 hours after casting
D. for 72 hours after casting

139. when the skin , whole epidermis , dermis and the underlying structures are affected in a burn
, it is called
A. first degree burn
B. second degree burn
C. third degree burn

140. the most common and serious complication of burns that often lead to death is -:
A. hypovolemic shock
B. hypothermia
C. sepsis.
D .Infection

136
141. before giving antibiotic to a burn patient which of the following should be done first-:
A. wound culture
B. blood tests
C .wound dressing
D. sensitivity testing.

142. when the patient is diaphoretic , there is tachycardia and decrease blood pressure , he is
experiencing-:
A. neurogenic shock.
B. Hypovolemic shock
C .Hypothermia
D. Septicemia

143. What type of isolation precaution is indicated for a burn patient?


A. Standard precaution
B. Reverse isolation ‫حماية المريض‬
C. Proper isolation‫حماية األخرين‬
D. Disposal of sharps
144. Which is the best aid to prevent breast cancer?
A. Teaching women how to perform self breast exam
B. Public knowledge about chemotherapy
C. To eat fruits and vegetable only
D. Encourage women to perform self breast examination monthly

145. In cute stage of osteomyelities , the child should be-:


A. Confined to bed.
B. Up with crutches.
C. Up in a wheelchair
D. Up and about

146. Which one should the nurse monitor first in an infant?


A. Respiratory rate
B. Cardiac rate
C. Pulse rate
D .Motor functioning

147. When assessing neonates hydration , the nurse should check for the skin’s-:
A. Elasticity
B. Tone
C. Moisture
D. Color

137
148. Cerebral palsy can be detected in which of the following stage of child’s life?
a. During the early months
b. When the child is walking
c. When the child is playing
149. What is the normal weight of the new born according to statistics?
A. 3700 gms
B. 3200 gms.
C. 2500 gms
D. 4500 gms
150. when the head of the baby is already out in a cervix , the nurse should do which of the
following first?
A. ask the mother to push more
B. check if the baby is breathing
C. wait for the baby to come out
151. FHR is heard clearly on fetal-:
A. head
B. back
C. abdomen
152. soon after the delivery of head of the baby the nurse should -:
A. suction mouth and oral cavity
B. wait for the delivery of the baby
C. ask her to push
153. the fetus will get immunity from-:
A. lymph system
B .Placenta
C. Both of them
D. Non of them
154. diet and nutritional status of pregnancy should be encouraged-:
A. first trimester
B. second trimester
C. third trimester
155.when the bag of water ruptures , the nurse should watch for which of the following-:
A. cord prolapse
B. bleeding
C. fetal death
D .fetal distress
E. respiratory distress
156. during labor a mother experienced dystocia , which of the following should the nurse watch
out for?
A. fetal death
B. hemorrhage
C. cord prolapse D. Hypertention

138
157. which are the signs of placenta separation?
A. rising of the fundus
B. sudden gush of blood
C. globular shape of the uterus
D. all of the above

158. during hyperthermia which of the following should be done as measure to lower the temp. of
the baby?
A. immerse baby in normal heat water
B. give cold enema
C. external cooling measures
D. all of the above

159. premature labor occurs during-:


A. between 20-36 weeks
B. between 36-42 weeks
C. before 20 weeks
D. first 12 weeks

160. which of the following hormones prepares the uterus for the arrival of a fertilized ovum for
implantation
A. progesterone
B. estrogen
C. FHS
D. non of the above

161. What is the most common complication after hysterectomy?


A .Abdominal distention
B. Hemorrhage
C. Vaginal pain
D. Vaginal discharge

162. When does ovulation occur?


A. Between 10-12 days
B. Between 12-16 days
C. Between 18-21 days

163. Diphtheria vaccine is being given during infancy and repeated-:


A. Booster dose at age 6
B. Every after 5 years
C. Every after 10 years.
D. No booster is recommended

139
164. There is no vaccination for-:
A. Small pox
B .Measles
C. DPT
D. Polio

165. The end of the first stage of labor is-:


A. When the cervix starts to dilate
B. Full dilatation of cervix up to 10 cm
C. Birth of baby
D. Separation of placenta

166. One sign of vaginal infection is-:


A. Pain
B. Itchiness
C. Foul smelling discharges

167. One post-partum nursing responsibility for preventing vaginal bleeding is-:
A. Put patient on trendelenburg position.
B .Regulate IV fluid with oxytosis
C. do fundal massage frequently

168. Fetal heart tone is best heard at-:


A. anterior part of the fetus
B. posterior back of the fetus
169. During pregnancy iron supplement starts on-:
A. first trimester
B. second trimester
C. third trimester

170. 72hours post placental delivery , fundic height will be-:


A. 1-2 finger above the umbilical
B. 1-2 finger below the umbilical
C. on the level of umbilical
171. in the hospital , isolation is indicated if the patient-:
A. staphylococcal pneumonia
B. local staphylococcal infection
C. puerperal fever
D. all are indicated

172. at which age is administration of the measles vaccine the most effective -:
A. 2 months
B. 6 months
C. 12 months
D. 15 months

140
173. the postpartal patient should be watched closely during the first hours after delivery for-:
A. uterine contraction
B. vaginal bleeding
C .hypotension
D. all of the above

174. premature labor is the labor that occurs between-:


A. 20-36 week gestation
B. 28-37 week gestation
C. 30-38 week gestation
D. any of the above

175. which of the following complication is life threatening after a penetrating abdominal injury-:
A. Hemorrhage.
B. Peritonitis
C. Abdominal distention

176. which of the folloeing is the immediate treatment for punctured wound?
A. cleans the wound
B. Stop the bleeding
C. Remove the object from the wound
D. Give prophylactic anti-tetanus
177. When there is malfunction of the glomerular filter there is high secretion of-:
A. Urea
B. Creatinine
C. Potassium
D .Protein

178. In a nephritic syndrome , aside from clinical manifestation , what else can the patient
complain upon consultation?
A. Protienurea
B .Oliguria
C. albuminuria
D. polyuria

179. ECT is indicated to treat:


A. psychotic depression
B. obsessive compulsive disorder
C .neurosis

180. After taking tranquilizer ,a nurse must warn the patient that he may experience-:
A. nausea
B. hallucination
C .drowsiness

141
181. One of the signs and symptoms of paralytic ileus is-:
A. increased peristalsis
B. increased gas formation
C. absence of bowel sounds

182. The most important nursing responsibility following head injury is-:
A. monitor vital signs and record
B. monitor GCS– Glasgow coma scale
C. observe for restlessness

183. Post –operative prevention of thrombophebitis-:


A. leg exercise in bed
B. passive exercise by nurse to patient
C. early ambulation

184. nursing management which is contraindicated for thrombophelbitis-:


A. encourage early ambulation
B. deep breathing and coughing exercise
C. massage the operative site i.e. leg calf muscle

185. what will you observe following spinal anesthesia?


a. nausea and vomiting
b. restlessness
c. hypotension and headache

186. second degree burns involve-:


A .skin , dermis
B. dermis , subcutaneous , muscle
C. epidermis , dermis , subcutaneous

187. within 48 hours of burns , management is focused on -:


A. prevention of infection
B. hydration

188. best diagnostic test for suspected leukemia is-:


A. CBC
B. Blood chemistry
C. Bone marrow aspiration

189. Nursing responsibility on leukemia-:


A .Ensure safety
B. Prevent infection
C. Obtain blood samples regular

142
190. Immediate management for reaction during blood transfusion-:
A. Slow down the rate
B. Stop blood transfusion
C. Change blood transfusion with a new pack

191. How many liters of oxygen can be given with nasal canula?
A. 1-2 liters
B. 1-10 liters
C. 1-6 liters.

192. destruction of bacterial is called-:


A. medical asepsis
B .surgical asepsis
C. disinfections.

193. first management of unconscious victim is-:


A. administer shock
B. determine unresponsiveness
C. administer oxygen.
D. give cardiac massage.

194. Nursing responsibility before giving Digoxin-:


A. Check BP
B. Check apical rate
C. Instruct patient to do deep breathing

195. Treatment of scurvy-:


a. Vit.b supplement
b. Vit.c supplement
c. Iron supplement
196. Atropine so4 is given pre-operative to-:
a. relax the patient
b. decrease the secretion
c. prevent intra-operative bleeding

197. when the patient is diagnosed with wilm,s tumor , the nurse should-:
a. always keep bed rails up
b. avoid palpation of abdomen
c. observe foe nausea and vomiting

198. a patient with asthma is receiving intravenous aminophylline .the adverse reaction for which
the nurse should observe is-:
a. oliguria
b. braducardia

143
c. hypotension
d. hypertension

199. The force with which the blood is pushing against the arterial walls when the ventricles are
contracting is called-:
a. pulse pressure
b. pressure gradient
c. systolic pressure
d. diastolic pressure

200. acide-base balance refers to the regulation of the concentration of-:


a. hydrogen ions
b. sodium
c. bicarbonates ions
d. plasma protein

201. carbohydrates are stored in the body in the form of-:


a. glucose
b. glucagons
c. glycogen
d. glucose 6 phosphate

202. the period in which communicable is most contagious is the-:


a. incubation period
b. period of illness
c. convalescent period
d. all are equally contagious

203. spinal fluid for culture-:


a. can be stored in the refrigerator for 48 hours
b. can survive refrigeration for only 4-6 hours
c. must be examined immediately
d. will survive in a holding medium for 72 hours

204. an enjection the gluteal site must be given in which quadrant of the buttocks-:
a. upper – inner quadrant
b. upper-outer quadrant
c. lower – inner quadrant
d. lower – outer quadrant
205. the most important rout of drug excretion for nonvolatile substances is-:
a. lungs
b. kidneys
c. feces
d. liver

144
206. atropine is capable of producing which of the following effects-:
a. dilated pupils
b. decreased bronchial secretion
c. blocked stimulation of the vagus nerve
d. all of the above

207. the sulfonamide drugs remin the treatmint of the control of -:


a. respiratory infections
b. skin infections
c. urinary tract infections
d. gastrointestinal infections

208. The drug of choice for treatment of an angina pectoris-:


a. nitroglycerin
b. quinidine
c. epinephrine
d. dopamine

209. any one using antihistamine should be cautioned that antihistamine may cause-:
a. drowsiness
b. hypertinsion
c. tachycardia
d. anaphylaxis

210. the most frequent causes of death in pneumonia patients are shock and-:
a. pulmonary embolism
b. pulmonary edema
c. pulmonary consolidation
211. the most important factor in the promotion and maintenance of wound healing both during
surgery and the postoperative period is-:
a. adequate fluid intake
b. proper administration of antibiotics
c. strict asepsis
d. frequent cleansing of the wound

212. the major complications of inhalation anesthesia which the nurse should be constantly aware
of-:
a. circulatory depression only
b. circulatory and respiratory depression
c. respiratory and renal depression
d. renal and circulatory depression

145
213. if the skin must be shaven prior to surgery . ideally the preoperative shave be done-:
a. the night before
b. early in the morning of the surgery
c. no more than one hour before surgery
d. by the patient before entering

214. which of the following amounts of water per day should be ingested by the average person
to maintain hydration-:
a. 500cc
b. 1000cc
c. 1200cc
d. 2500cc

215. in the internal environment , fluids make up what percent of body weight-:
a. 10 %
b. 30 %
c. 70 %

216. in which of the following ways can ascorbic acid be administer?


a. orally
b. IM
c. Diluted intravenous fluids
d. all of the above

217. the type of burn in which all the dermis and epidermis , is destroyed and there is involvement
of underlying structures is called-:
a. superficial or first degree burn
b. partial thickness or second degree burn
c. full-thickness or third degree burn
d. fourth degree burn

218. while teaching diabetic patient to give himself insulin , you should stress that injections
should not be given in any one spot more often than every-:
a. 36hours
b. one week
c. two weeks
d. month

219. which of the following dietary restrictions are usually indicated to decrease edema in
nephritic-:
a. high – protein , low – calorie , on sodium restrictive diet
b. high – protein , high – calorie , low on sodium diet
c. low– protein , low – calorie, high vitamin diet
d. low– protein, high – calorie , on sodium diet

146
220. following a sigmoidoscope , the patient should be observed for signs of hemorrhage and -:
a. fluid loss
b. performation
c. flatus

221. to avoid the possible development of heart diseases following rheumatic fever the patient
must guard himself against infection-:
a. until the fever is gone
b. for one year
c. until sedimentation rat is normal
d. for the rest of his life

222. hypertension is a persistent elevation of systolic and diastolic pressure above-:


a. 90/140 .mmhg
b. 190/150mmhg
c. 100/160mmhg
d. 110/160mmhg

223. After below knee amputation-:


a. keep the patient in semi fowler’s
b. stump to be evaluated
c. watch the site for bleeding

224. to collect urine c/s specimen , the most accurate method-:


a. catheterize the patient
b. mid stream urine
c. provide sterile bedpan and collect urine

225. the nurse should observe the patient receiving decradon-:


a. urinary stasis
b. hypotension
c. infection
d. weight loss
226. signs of septic shock-:
a. cool & clammy skin
b. bradicardia
c. warm & dry skin

227. oral contraceptive used for-:


a. once daily for 21 days
b. once daily for 18 days
c. once daily for 28 days

147
228. normal arterial O2 saturation-:
a. %85
b. %92
c. %97
d. %100

229. valve which control the blood flow from right atrium to right ventricle-:
a. bicuspid valve
b. tricuspid valve
c. mitral valve
d. pulmonary artery

230. the common complication after operation-:


a. fever
b. pulmonary edema
c. pulmonary embolism

231. digitalis is given-:


a. with meals
b. before meals
c. 3 hours after meals
d. bedtime

232. diet in cirrhosis of liver without ascitis and swelling-:


a. low protein with multivitamins
b. high protein with vitamins B supplement
c. high protein with sodium restriction
d. high protein with diet.

233. persistent sweating , hypotension , tachycardia with loss of fluid on blood . nurse would
suspect-:
a. hypovolemic shock
b. cardiogenic shock
c. septic shock
d. neurogenic shock

234. before the collection of blood culture , the site should be cleansed-:
a. isopropyl alcohol%70
b. isopropyl alcohol %100
c. povidone iodine

235. blood transfusion reaction occurs , the nurse should-:


a. slow down the rate
b. notify the physician
c. shut off the transfusion

148
236. signs of hyperglacemia-:
a. polyuria , polydepsia , loss of appetite
b. acetone breath , flushed face , polyuria

237. if O2 is ordered as midication , which method the nurse would choose?


a. nasal canula
b. nasal catheter
c. O2 mask
d. O2 tent

238. pre-op medication is given-:


a. 2hours before operation
b. 45min. before operation
c. previous night of operation
d. as physician ordered

239. temp. 98.6 f , is equal to-:


a. 37 .C
b. 37.5C
c. 36.8C
d. 37.3C

240. complication of chemotherapy-:


a. bon marrow depression
b. liver damage

241. ECT is given for-:


a. psychotic depression
b. obsessive compulsive neurosis
c. hysteria

242. Age group more prone to get rheumatic fever-:


A. 5-15 years
B. 5 years
C. above 65 years

243. iron is mostly absorbed in-:


A. large intestine
B. small intestine
C. liver

244. early signs of respiratory arrest-:


A. thread pulse
B. cyanosis
C. orthopnea.

149
245. one fluid ounce equal to-:
A. 30cc
B. 15cc
C. 60cc
D5 .cc
246. most effective , easy and un-expensive method of sterlization is-:
A. heat
B. pressure
C. drying
D. antiseptic

247. in nephritic syndrome , apart from clinical manifestation the patient would complain of-:
A. proteinuria
B. oliguria
C. hematuria

248. in active stage of osteomelitis the child should be-:


A. continue in bed
B .allowed to sit in a wheel chair
C. walk up and down

249. an enlarged movable lymphnode is a sign of-:


A. inflammation
B. malignant
C .normal lymph node
D. any of the above

250. to prevent irreversible brain hypoxia CPR should be commenced within-:


A. 2-3min
B. 4-6 min
C 8-6 .min
D. 8-10 min

251. post-operative patient to do deep breathing & change of position every-:


A. 2 hourly
B. 4 hourly
C. in each shift
D. hourly

252. cystitis patient should be encourage to pass urine-:


A. 2-3hourly
B. 2-3 hourly 2 & times in the night
C. 2-3 times round the clock
D. any one of the above according to the patient’s schedule

150
253.mode of transmission of hepatitis A-:
A. oral
B .parental
C. blood transfusion

254. reacting of penicillin-:


A .anaphylactic shock
B. vomiting
C. nausea.

255. Disease without vaccination-:


A. small pox
B. measles
C. polio
D. chicken pox
256. health teaching to a diabetic patient will be on-:
A. diet , oral hypoglycemic , weight loss , feeding
B. diet , insulin , exercise , feeding
C. diet , hypoglycemic , exercise , feeding
D. about complications

257. surgical asepsis means-:


A. destroying bacteria from articles.
B. removing the bacteria which is in contact with the patient
C. destroying the bacteria before entering in to the body
D .hand washing

258. ECG done for a patient with chest to assess-:


A .stress of heart rate
B. change the rhythm
C. the decreased blood supply to the particular part of the heart

259. the nurse is expecting fracture of bone for patient , the sign would be-:
A. absence of normal activity
B. tenderness
C. loss of sensation
D. all of the above

260. the medicine which will pass through placenta to the fetus-:
A. antibiotic
B. narcotics
C. sedatives
D. all of the above

151
I- Choose the most appropriate answer:

1. All of the following are normally present in urine result EXCEPT:


a) Urea.
b) Createnin.
c) Albumin.
d) Sodium.

2. All of the following are etiologic factors of fluid volume deficit except:
a) Vomiting
b) Diarrhea
c) Renal failure
d) Excessive sweating

3. All of the following are most common complications postoperatively except:


a) Hemorrhage
b) Tonsillitis
c) Wound infection
d) Pain

4. All of the following are clinical manifestations of fluid volume excess except:
a) Edema
b) Oligurea
c) Distended neck veins
d) Increased CVP

5. Apnea is medical term means:


a) Rapid pulse
b) Increase body temperature
c) Stop breathing
d) Low blood pressure

6. All of the following are nursing actions during preoperative nursing care except:
a) Observe incisional site
b) Remove hairpins and hairpieces
c) Remove jewelry and prostheses
d) Assess all vital signs

7. Signs and symptoms of Hypovolemic shock are all of the following except:
a) Tachycardia
b) Hypertension
c) Pallor and cyanosis
d) Tachypnea

8. Normal Saline 0.9% solution is considered as which of the following:


a) Isotonic solution
b) Hypotonic solution
c) Hypertonic solution
d) Non of the above

152
9. When blood sugar level is above normal range, this means that patient has:
a) Hypotension
b) Hypoglycemia
c) Hyperglycemia
d) Bradycardia

10. The most important thing should be done after any nursing action is:
a) Documentation
b) Nursing diagnosis
c) Planning
d) All of the above

11. All of the following are assessment sites for body temperature except:
a) Oral Site.
b) Rectal Site.
c) Axillary Site.
d) Apical

12. Mr. Ashraf aged 35 years old, his pulse rate is found to be 120 bpm. His heart rate could be
described as:
a) Tachypnea.
b) Tachycardia.
c) Bradypnea
d) Bradycardia.

13. Which of the following factors are affecting body temperature:


a) Food intake.
b) Age and Gender
c) Climate.
d) All of the above

14. The condition in which a person is aware of his or her own heart beats is called:
a) Palpitation.
b) Dysrhythmia.
c) Pulse rhythm.
d) Arrhythmia.

15. The condition in which the body temperature is above the average normal is called:
a) Bradypnea
b) Fever.
c) Hypertension.
d) Hypothermia.

16. Which of the following best describes the technique of palpation:


a) The use of the sense of hearing to listen for sounds.
b) The use of tapping on a particular part of the body to produce sounds.
c) The use of the senses to evaluate general appearance of the patient.
d) The sense of touch to feel the body by slight or deep pressure.

153
17. Inhalation or inspiration means:
a) Stop breathing.
b) Difficult breathing.
c) Breathing out.
d) Breathing in.

18. Normal body temperature is ranging from:


a) 35.8 – 37.4 C
b) 34.5 – 36.5 C
c) 35.0 – 38.0 C
d) 36.5 – 38.5 C

19. Pulse pressure is defined as which of the following:


a) Difference between systolic and diastolic pressure.
b) Expansion of the artery as blood moves through it.
c) Difference between arterial and venous pressure.
d) Difference between venous and systolic pressure.

20. Sleep is defined as:


a) A state of arousable unconsciousness.
b) A state of emotional rest and excessive sleepiness.
c) A state of drowsiness and decreased activity.
d) A state of decreased activity and mental stimulation.

21. Unoccupied bed making means:


a) Changing linen when the patient remains in the bed.
b) Changing linen when the bed empty.
c) State in which a person is relieved of distress.
d) None of the above.

22. The advantage of use head to toe approach when you assess the patient:
a) It is difficult to detect the disease or problem.
b) It helps to prevent overlooking some aspect of data collection.
c) It increase the number of position changes.
d) It takes more time.

23. The most accurate time for measuring pulse rate is:
a) 30 seconds.
b) 15 seconds.
c) 60 seconds.
d) 45 seconds.

24. Normal respiratory rate at rest per minute in adults is ranging from:
a) 10-14.
b) 12-20.
c) 20-25.
d) 20-30.

154
25. Practices that promote health through personal cleanliness is called:
a) General appearance.
b) Self image.
c) Insomnia.
d) Hygiene.

26. All of the following are therapeutic bathes except:


a) Shampooing bath.
b) Whirlpool bath,
c) Medicated bath.
d) Sitz bath.

27. Orthopnea is defined as:


a) Breathing facilitated by sitting or standing up.
b) Absence of breathing.
c) Rapid breathing.
d) Difficult breathing.

28. Which of the following is a form of radiograhpy:


a) Fluoroscopy.
b) Contrast medium.
c) C.T. Scan
d) All of the above

29. The examination that indicates physical inspection of the vagina and cervix with palpation of
uterus and ovaries is called:
a) A Pap test.
b) Electrocardiography.
c) Pelvic examination.
d) Paracentesis.
30. Procedure that involves the insertion of a needle between lumber vertebra in the spine but
below the spinal cord itself is called:
a) Lumber puncture.
b) Paracentesis.
c) Pelvic Examination.
d) Electromyography.
31. Which of the following is considered as a benefit of bathing:
a) Improving self image.
b) Eliminating body odor.
c) Stimulating circulation.
All of the above.

32. The patient is reclining on back with the feet elevated in metal support, this position is called:
a) Knee-Chest position.
b) Lithotomy position.
c) Modified Standing position.
d) Sims position.

155
33. Exchange of oxygen and carbon dioxide when taking air in and out of the chest is:
a) Respiration.
b) Circulation.
c) Blood pressure.
d) None of the above.

34. Ventilation means:


a) Movement of air in and out of the chest.
b) Movement of air out of the chest.
c) Movement of air in the chest.
d) Non of the above.

35. All of the following are common factors that invalidated examination or test results except:
a) Inadequate specimen volume.
b) Failure to send the specimen in a timely manner.
c) Correct diet preparation.
d) Insufficient bowel cleansing.

36. All of the following are physical assessment techniques except:


a) Inspection.
b) Percussion.
c) Puncturing.
d) Palpation

37. All of the following are post procedural nursing responsibilities except:
a) The nurse has to attend the patient for comfort and rest.
b) Care of specimens.
c) Assist the examiner.
d) Record and report of information.

38. A patient is prepared for hemodialysis. He receives heparin before therapy before primarily to
help:
a) Relieve discomfort
b) Prevent blood clotting
c)Maintain blood pressure
d) Stimulate the production of urine

39. Another name for a stone in the urinary tract is:


a) Calix
b) Calculus
c)Calcemia
d) Calcitonin

40. Urine that remains in pt's bladder after he voids is called:


a) Reflux urine
b) Over flow urine
c)Retention urine
d) Residual urine

156
41. For a normal person the urine specific gravity is ranged between:
a) 1.000 and 1.010 c) 1.025 and 1.050
b) 1.015 and 1.025 d) 1.050 and 1.070

42. Which of the following conditions does not predispose the development of renal failure:
a) Mismatched blood transfusion
b) Sever dehydration
c)Septicemia
d) Cerebrovascular accident (CVA)

43. Which of the following terms is used to indicate total urine out put of less than 50 ml in 24
hours:
a) Anuria
b) Oliguria
c)Dysuria
d) Polyuria

44. Which of the following terms refers to a disorder characterized by proteinuria, edema,
hypoalbuminuria, and hyperlipidemia.
a) Pyelonephritis
b) Interstitial cystitis
c)Nephrotic syndrome
d) Glomerulonephritis

45. The name of the procedure for the removal of urinary bladder stone is:
a) Cystolithiasis
b) Cystolithectomy
c)Cystometry
d) Cystoextraction

46. Patients who are receiving hemodialysis for ESRD (end stage renal failure), are especially prone
to develop:
a) Peritonitis
b) Renal calculi
c)Bladder infection
d) Serum hepatitis

47. After the IVP, the nurse should anticipate incorporating which of the following measures into
the client plan of care:
a) Maintaining bed rest.
b) Encouraging adequate fluid intake.
c) Assessing for hematuria.
d) Administering laxatives.

48. The nurse should instruct the female to reduce the recurrent UTI as the following:
a) Cleanse around the perineum with front to back cleansing.
b) Wash genitalia in tub bath.
c) Drink alittle amount of fluid.
d) Non of the above.

157
49. Acetyl calsilic acid (Aspirin) contraindicated to be given in:
a) Pt. with hemorrhage .
b) Severe headack.
c) Heart disease
d) Rheumatic fever

50. Gentamicin is:


a) Broad spectrum antibiotics .
b) Antimetabolite .
c) Aminoglycocides
d) A and c

51. Paracetamole is:


a) Analgesic .
b) Antipyretic .
c) Antibiotic
d) A and B

52. All of the following are analgesics except:


a) Diclofin
b) Flagyl
c) Acamol
d) Aspirin

53. Pt. with head trauma should have all the following treatment except:
a) IV line catheter
b) IV fluid
c) Sedatives
d) Oxygen

54. You should instruct the Pt. when taking Prothiasin:


a) To take it after 2 hours of meal
b) To void after 2 hours
c) Not to drive his care
d) Don’t worry about vomiting

55. Diclofin is:


a) Antipyretic
b) Non steroidal anti-inflammatory drug
c) a and b
d) Non of the above

56. Insertion of a tube through the nose into the stomach called:
a) Orogastric intubation.
b) Nasogastric intubation.
c) Nasointestinal intubation.
d) Transabdominal tube.

158
57. Lavage is:
a) Giving nourishment by tube.
b) Surgical opening in the stomach.
c) Removing substances from the stomach.
d) Non of the above.

58. Normal color of urine is:


a) Urine has no color. C) Light yellow.
b) Bright red color. D) Light green.

59. Urinary elimination process means:


a) The process of re-absorption of water and minerals in the kidney.
b) The process of urine collection in the human bladder.
c) The process of releasing excess fluid and metabolic wastes.
d) The process of urine assessment by the lab technicians.

60. Measurement of the NGT as:


a) From neck to nose to epigastric region.
b) From shoulder to nose to zephoid process.
c) From ear to nose to zephoid process.
d) From mouth to nose to zephoid process.
61. Substances associated with childhood poisoning include all of the following except:
a) Drugs.
b) Cleansing agents.
c) Fresh food.
d) Cosmetics.
62. For all the following poisoning situation induce vomiting and gastric lavage recommended
except:
a) Food poisoning.
b) Drug poisoning .
c) Caustic and corrosive poisoning .
d) Plant poisoning.
63. Pain transmition begins by stimulation of:
a) Nerve plexus.
b) Nerve receptors.
c) Spinal cord.
d) Skin.
64. Blood pH normal range:
a) 7.25 – 7.30
b) 6.40 – 7.00
c) 7.75 – 8.00
d) 7.35 – 7.45
65. Promotion of oxygenation could be occur by:
a) Maintain air way open
b) Positioning (sitting or fowler)
c) Oxygen supply.
d) Breathing techniques.
e) All of the above.

159
66. Peri-operative period means:
a) Total period before surgery
b) Total period after surgery
c) Total period of (before, during and after surgery)
d) Non of the above

67. All the following factors are considered when selecting a syringe and needle except:
a) Depth of the tissue
b) Family history
c) Type of the drug
d) Size of the patient

68. All the following routs of administration considered parenteral routs except:
a) Intradermal
b) IV
c) SC
d) Otic
e) Intra-joint
f) Ophthalmic
g) Both d and f

69. IV bolus administration means:


a) Given drug IV in short period
b) Given drug IV in along period
c) Given drug IV by infusion
d) Given drug IV quickly without dilution

70. Z-Tract technique used in:


a) Deep intramuscular injection
b) Irritant drugs
c) Both a and b
d) Neither a nor b

71. All the following IM injection sights except:


a) Dorsogluteal
b) Forearm
c) Ventrogluteal
d) Vastus lateralis
e) Deltoid

72. In case of meningitis, the CSF examination will be :


a) Low protein level.
b) Low glucose level.
c) Clear appearance.
d) Decrease CSF pressure.

160
73. All the following are potential complication of meningitis Except:
a) Seizures.
b) Cerebral infarction or abscess.
c) Myocardial infarction.
d) Subdural effusion.

74. All the following is clinical manifestation of a cute glomerulonephritis Except :


a) Brown colored urine
b) Weight loss.
c) Malaise.
d) Hypertension

75. 11- All of the following are considered congenital a cyanotic heart disease Except:
a) Ventricular septal defect (VSD)
b) Tricuspid atresia.
c) Coarctation of aorta
d) Patent ductus arterioses.

76. Tetorolgy of fallot is characterized by all of the following Except:


a) Right ventricular hypertrophy.
b) Pulmonary stenosis
c) Ventricular septal defect.
d) Atrial septal defect.

77. Kerning's sign is manifestation for:


a) Wiliam's tumor Encephalitis
b) Acute Rheumatic Fever.
c) Encephalitis
d) Meningitis

78. All of the following are cardinal signs of nephrotic syndrome Except:
a) Hypoalbuminemia
b) Proteinuria
c) Hypocholesterolemia
d) Edema

79. A full-term pregnancy normally ranges between:


a) 37-40
b) 38-42
c) 38-44
d) 36-40

80. Morning sickness is caused by:


a) Vitamin deficiency
b) Exhaustion
c) Pregnancy Hormones
d) Twins pregnancy

161
81. During what phase of the menstrual cycle is an egg (ovum) released?
a) Ovulation
b) Gestation
c) Luteal
d) Mittelschmerz
e) Incubation
82.
What does amenorrhea mean?
a) Painful menstruation
b) Menstrual cycle where ovulation does not occur
c) Absence of menstrual period
d) Heavy bleeding during menstruation

83. When the foetus is growing inside the uterus it needs nutrients. What provides these
nutrients?
a) Placenta
b) Amniotic sac
c) Uterus
d) Oviduct
84. Why is breast milk important to a newly born baby?
a) It is the right temperature for the baby
b) It is made by its own mother so it is matched to its needs
c) It contains antibodies that protect against common microorganisms
d) It tastes better than bottle milk

85. HCG is an acronym for what?


a) Human cloning gene
b) Human cell gamete
c) Human chorionic gonadotropin
d) Human corpus growth

86. The family structure consisting of parents and their dependent children living together is
known as a(n):
a) Binuclear family
b) Reconstituted family
c) Nuclear family
d) Extended family
87. Adolescents are at a greater risk for injury from:
a) Poisoning and child abduction.
b) Automobile accidents, suicide, and substance abuse.
c) Home accidents.
d) Physiological changes of aging.
88. The patient lying face down is called:
a) Supine
b) Prone
c) Fowler's
d) Lateral

162
89. Which of the following has had the greatest impact on reducing infant mortality in the United
States?
a) Improvements in perinatal care
b) Decreased incidence of congenital abnormalities
c) Better maternal nutrition
d) Improved funding for health care

90. Which of the following is FALSE concerning gestational diabetes in mothers-to-be?


a. The diabetes usually disappears after birth
b. Their babies may experience hypo-glycemia after birth
c. Their babies are commonly born with breathing difficulties
d. Their newborns tend to be underweight

91. What is polyhydramnios?


a. Excessive vomiting during pregnancy
b. Excessive maternal hair growth
c. Excessive maternal swelling
d. Excessive fluid around the fetus

92. Preeclampsia is another possible complication of pregnancy. Which of the following is NOT
characteristic of preeclampsia?
a. protein in the urine
b. high blood pressure
c. vaginal bleeding
d. swelling

93. Pregnancy Lasts on average


a. 280 days from last period
b. 37 weeks
c. 42 weeks
d. 40 weeks
e. a & c

94. Which nutrient has been proven to prevent birth defects?


a. Folic acid
b. Pantothenic acid
c. Manganese
d. b. Riboflavin

95. Which of the following vitamins helps the body absorb iron?
a. Vitamin C
b. Riboflavin
c. Vitamin E
d. Vitamin A

163
96. When will the baby first recognize his mother visually?
a. From birth
b. 3 to 4 weeks
c. 3 to 4 months
d. 6 to 8 months

97. Antibodies in milk protect the baby against problems with:


a. Vision
b. Gastrointestinal tract
c. Respiratory system
d. B and C

98. You're most fertile when one of your ovaries releases a mature egg. This process, called
ovulation, occurs:
a. Halfway through your menstrual cycle
b. 12 to 16 days before your next period starts
c. Right before you get your period

99. The Nurse Midwife is an advanced practice nurse who:


a. Provides care to child-bearing women during preconception, prenatal, labor and
delivery, and postpartum periods.
b. Transports and cares for sick and injured patients in emergency situations and
during the transport of patients from one patient care facility to another.
c. Performs pharmaceutical, medical, and nursing research.
d. Helps create public health policies.

100. A newborn loses heat through:


a. Evaporation
b. Conduction
c. convection
d. radiation
e. insensible water loss through skin, feces, urine, respiration
f. all of the above

101. 8-Premature separation of the normally implanted placenta is


a. placenta accrete
b. placenta previa
c. placenta percreta
d. none of the above

102. Treatment with hormone replacement therapy


a. Should always include a progestagen.
b. Reduces the incidence of cardiovascular disease in post-menopausal women.
c. Is not beneficial unless given within one year of the menopause.
d. Should not result in vaginal bleeding.
e. Is of no benefit unless given for longer than ten years.

164
103. Immediate post-partum hemorrhage is associated with
a. Retained placenta.
b. lacerations or tears of the cervix.
c. Ritodrine administration.
d. Uterine atony.

104. Placenta previa is:


a. Premature separation of the normally implanted placenta before the birth of the
fetus.
b. The abnormal implantation of the placenta in the lower uterine segment, partially
or completely
c. A placenta that attaches itself too deeply and too firmly into the wall of the uterus.
d. Placenta that bleeds all the pregnancy period

105. Which of the following is not considered a fat soluble vitamin?


A. Vitamin A
B. Vitamin B
C. Vitamin K
D. Vitamin E

106. Which of the following is not considered a water soluble vitamin?


A. Vitamin B
B. Vitamin C
C. Vitamin D
D. Vitamin B12
107. Which of the following is not an underlying cause of hypercalcemia?
A. Paget's disease B. Hyperparathyroidism
C. Hartnup disease D. Sarcoidosis

108. Which of the following is caused by a B5deficiency?


A. Ectopic pregnancy B. Nausea
C. Dermatitis D. Fever

109. Which of the following is caused by a B6 deficiency?


A. Excessive irritability B. Nonproductive cough
C. Dry mouth D. Depression

110. Which of the following is caused by a B12 deficiency?


A. Glossitis B. Fever
C. Hypertension D. Edema

165
111. Which of the following is caused by a Vitamin C deficiency?
A. Fever B. Anemia
C. Headaches D. Nausea

112. Which of the following is caused by a Vitamin D deficiency?


A. Edema B. Anemia
C. Lupus D. Rickets

113. Which of the following is caused by a Vitamin K deficiency?


A. Bruising B. Optic Nerve degeneration
C. Anemia D. Hemorrhage (infants)

114. Another name for Vitamin B1 is ____ .


A. Thiamine B. Riboflavin
C. Pyridoxine D. Cobalamin

115. Which of the following foods is not high in potassium?


A. Oranges B. Bananas
C. Tomatoes D. Turnips
116. 1 gram of protein is equal to ____ of energy.
A. 3 kcals B. 4 kcals
C. 5 kcals D. 6 kcals

117.A protein restricted diet requires only ______ grams/day.


A. 20-40 B. 30-50
C. 40-60 D. 50-70

118. Which of the following is not considered a food reaction for infants?
A. Diarrhea B. Hypertension
C. Skin rash D. Difficulty breathing

119. Which of the following vitamins will be the most common in: oils from cereal seeds, salad
oils, margarine and shortenings?
A. Vitamin A B. Vitamin D
C. Vitamin E D. Vitamin K
120. Which of the following vitamins will be the most common in: leafy green vegetables, egg
yolk and soy oil?
A. Vitamin A B. Vitamin D
C. Vitamin E D. Vitamin K

166
121. Which of the following vitamins will be the most common in: fish liver oils, milk, and egg
yolk?
A. Vitamin A B. Vitamin D
C. Vitamin E D. Vitamin K

122. Another name for Vitamin B12 is ____ .


A. Pantothenic Acid B. Cyanocobalamin
C. Niacin D. Riboflavin

123. Which of the following does not contain a high concentration of Niacin?
A. Yeast B. Meat
C. Liver D. Corn

124. Which of the following does not contain a high concentration of Vitamin A?

A. Strawberries B. Oranges
C. Green Vegetables D. Yellow Vegetables

167

Das könnte Ihnen auch gefallen